Vous êtes sur la page 1sur 79

February 2014 Medicine Solved Paper

192 MCQs (Mixed Paper A & B)


1. High Pressure in Glomerular capillaries is due to
a. Afferent is long b. Afferent is short c. Efferent is low resistanced.
Efferent is down stream
2.Regarding SLE followin are true except
a. ANA is +ve
b. Fibrinoid necrosis is common histological feature
c. Glomerulonephritis is commonest cause of death
d. more common in women
e. risk is increased by inheritance of HLA B27
3.Edema in nephrotic syndrome
a. Na retention
b. Hypoproteinemia
c. Hyperlipidemia
4.Bronchopulmonary segment
a. each has its own blood supply
b. different number on both sides
5.Recently diagnosed fatal disease, most appropiate
a. tell family but nt patient
b tell patient but nt family
c. tell family and patient as soon as knwn
d. crisp, logical and evidence based accurate information to the patient
and family accordin to demand
6.Young lady having anemia, MCV 78 , MCH 25, MCHC 29
a. Hypochromic
b.Hypochromic and Normocytic
c.Hypochromic and microcytic
d. normocytic and normochromic
7.WHICH OF THE FOLLOWING SHIFTS THE O2-HGB
DISSOCIATION CURVE TO RIGHT:
A.ACIDOSIS B.ALKALOSIS C.INCREASE pH D. NO E. FETAL
HEMOGLOBIN
8.A 75 year old man with an acute MI suddenly dies 6 days after the
infarct . At autopsy, there is a
large amount of blood in the pericardial sac . The most likely cause of
death is ;
A. Cholesterol pericarditis B. Amyloidosis C. Cardiac tamponade D.
Codsackie B pericarditis E. Constrictive pericarditis
9.most active gluconeogenesis results from
metabolism of ?
a.fatty acid b.protein c.cholesterol d.glycogen e. TG
10. Severe magnesium deficiency
a. Hypocalcemia b. Hypercalcemia c. Hypokalemia
d.Hypophosphatemia
11. Langerhans giant cells found in
a.Tuberculosis b.Sarcoidosis c.Wegners granulomatosis
D)Syphlis
12. parasympathetic inervation
a.sweat gland b.salivary gland
13. A pt has multiple polyps on colonoscopy.His father died of same
disease.These types of polyps are
a-Hemartomatous
b-Adenomatous polyp
c-Metaplastic
14. Dysarthria is due to a lesion in?
a.Brocas area b.Cerebellum c.Thalamus d.Sensory motor cortex
e.Hypothalamus
15.Hypothyroidism
a. Increased cholesterol
16. During 2nd week which is appropriate test for
Typhoid fever ?
a) Widal+Blood culture
b) Blood culture only
c) Stool culture only
d) Bone marrow
17. A woman havin fever for few days developed chest pain which
aggravated by lying down . Reason
is?
a.Costochondral junction
b.Mi
c. pericarditis
d.due to plurea
18. purpura is most likely asaociated with

a. leukemia
b. von wilbrand disease
c. hiv
19. a lady using diethylesterbestrol what should be most likely in her
daughter
a. squamous cell carcinoma
b. clear cell carcinoma
20. Lady with osteoporosis,having family h/o cardiac disease and
breast ca. Treated by
a. Calcium plus vit D b. Raloxifene c. bisphosphonates d. HRT
21. Which disease involves a carrier stage ?
a.Measles b.Mumps c.Polio d.Typhoid 22. H+ ion
secretion in proximal tubule is related with?
A. Bicarbonate reabsorption B. Bicarbonate excretion C. K secretion D.
Na secretion
23. Hamartoma is ;
A. A Metaplasia B. A neoplasia C. Has a capsule D. Same as
cholestoma
E. Totally benign
24. 40 yr old woman, middle class,3 kids ,pap smear shows cervical
dysplasia..cause
A HPV B IUCD
25. 10% Formaldehyde is used for sending Biopsy
specimens,Why?
a. Sterilisation b.To prevent autolysis c. TO stain cells
d. TO denature proteins e. to cause lysis of cell
26. Metastatis occur in all except
a.Liver b. Brain d. Spleen d. lung
27.Fast, sharp pain from mechanical origin is mediated by which fibers;
A. A alpha B. A delta C. C fiber D. High myelinated
28. Fat necrosis occurs in ; A. Acute pancreatitis B. Heart C. Liver
29. Which tumor is derived from all the 3 germ layers;
A. Adenoma B. Teratoma C. Rhabdomyosarcoma D. Chancroid
30. A 20yr old male has dyspnea on lying down. The structure most
likely to cause this is ;
A. Enlarged isthmus of thyroid B. Puberty C. Retrosternal goiter
31. A FISHER MAN PRESENTED WITH SLOWLY DEVELOPING
LETHARGY, EASY FATIGUE AND
PALPITATIONS. HIS DIET COMPRISES OF FISH AND RICE
MOSTLY PHYSICAL EXAM SHOWED PALLOR
AND LOSS OF TOUCH SENSATION IN BOTH FEET AND LOWER
LIMB. HIS CBC SHOWED HB 7.5G/DL MCV
132, PLATELETS 110 AND WBC 3450 . MOST LIKELY CAUSATIVE
PARASITE:
A.ANKYLOSTOMA DUODENALE
B.DIPHYLLOBOTHRIUM LATUM
C.STRONGYLOID STERCOLIS
D.ECHINOCOCCUS GRANULOSIS
E. TAENIA SAGINATA
32. Long term elevated BP is due to
a.cardiac output
b.renal function
c.TPR
33. The vitelline duct forms;
A. Ductus venosus B. Duodenum C. Meckel, s diverticum D. Portal
vein E. Urachus
34. Which is correct Regarding Axillary artery
a-It Lies behind pectoralis minor
b-Extend upto the lower border
of pectoralis major
35. a. the components of the cell containing microtubules in there
structure are;
a. cell membrane b. centrioles c. endoplasmic reticulum d. lysosomes
e. mitrochondria
36. a chronic smoker from 30 years working in a tyre factory presented
with painless hematuria n on
histopathology transitional cell carcinoma of bladder causative agent is
..?
a.aromatic amines b.tobaco smoking c.nitrites/nitratesd.shistosomia
hematobium
37. most common manifestation of autoimmune
disease ;
a. Fever b.arthralgia c.haematologic symptoms. d.fatigue
38. ACTH effectively controls ?
a. Aldosterone
b. Adrenal androgens
c. Adrenal estrogen
d. Hydrocortisone

39. what defines the acute inflammation ?


1.local response of living tissue to injury
2.systemic response of living tissue to injury
3.migration of leukocytes
40. aortic pressure lowest during which phase of cardiac cycle??
a.atrial systole
b.isovolumic contraction
c.isovolumic relaxation
41. The vision lost is left temporal and right nasal field of both eyes, the
visual pathway involved in
this lesion is
A. Rt optic nerve B. Left optic tract C. Rt optic tract D. Left optic nerve
42. the affect of L dopa in parkinson pt dcreease gragually due to
1.antibodies develop against l dopa
2.gradual degeneration of neurons in sunstantia nigra
43. Glucose transport across a membrane is directly proportional to
concentration graddient . Type of
transport is ;
A. Simple diffusion B. Facilited C. Active D. Na countertaransport
44. sebacous glands are absent on
1.corner of lips 2.palms 3.glans of penis
45. Blood transfusion reaction is likely to occur when a person having
blood group A+ is transfused
with blood of group;
A. A+ B. A- C. AB- D. O+ E. O46. C0mplicti0n of diabetic?
a.Wet gangren b. dry gangren c. ranyod phen0mena d. thrmbroagints
47. true regarding genetic inhertance of SLE is ;
a. high incidence in monozygotic twins
b. incidence is high in females
c. if its in family, more chances to occur to all family members
48. Langhans giant cells are found in;
a.Tuberculosis
b.Sarcoidosis
c.Wegners granulomatosis
d. Leprosy
49. most active gluconeogenesis results from metabolism of ?
a.fatty acid b.amino acids c.cholesterol d.glycogen/carbohydrates
50. Esophago gastric junction competence is
maintained by
A. lying supine B. increased intra abdominal pressure C. diaphragm
paralysis D. use of metoclopramide E. use of morphine
51. Diminished jaw jerk due to lesion of
A. Mesencephalic nucleus of V nerve B. Facial motor nucleus C.
Glossophyrngeal injury D. Vagus injury
52. Co-trimoxazole action in.hiv pt on pnumocystc carini?
a.Inhibit cellwall syn b.Inhibit folic acid synth c. inhibit DNA synth
53. Which of the following structure doesn't pass posterior to the flexor
retinacum
A. Ulnar nerve B. Median nerve C. Flexer digitorum superficialis D.
Flexer digitorum profundus
54. Cardiac surgeon during open heart surgery found bleeding vessel
accompanying LAD in anterior
interXventricular groove . Most likely involved vein is ;
A.Great cardiac vein
B. Anterior cardiac vein
C. Oblique vein
D. Small cardiac vein
E. Marginal vein
55. An upper motor lesion of facial nerve will result in ;
A. Excessive lacrimation
B. Hyperacusis
C. Loss of taste sensation in ant 2 /3 of tongue
D. Ability to wrinkle the forehead on the affected side
56. SQUAMOUS CELL CARCINOMA
a.Actinic keratosis b. marjulian ulcer c. Bowen Disease
57. Regarding histology of gastrointestinal tract, which is inappropriate
;
A. Peneth cells contain Eosinophilic granules in apical cytoplasm
B. Peyer's patches are present in submucous layer of duodenum
C. Parietal cells of human stomach secretes intrinsic factor
D. Striated brush border is composed of microvilli
E. The longitudinal coat of muscularis externa is arranged in 3 bands in
colon
58. Which one.is.malignant
a.Glioma b.Meningioma c.Psedolymphoma d.Chondroma
59. which cranial nerves r parasympathetic
A. III, VII, IX, X B. V, VIII, II, XII

60. Severly dehydrated man comes in er what to gv


a.5percent glucose
b.Ten precent glucose
c.Hypertonic albumin
d. 0.9% nacl
61. Fibrous dysplasia commonly occurs in
a.Maxilla b.Occipital bone c.Frontal bone d.Temporal bone
62. Weight of prostate
a.14 gr b.18 gr c.30 gr d.40 gr
63. Edema is caused by;
A. hydrostatic pressure B. osmotic pressure C. Lymphatic
obstruction
64. Growth hormone secretion is increased ;
A. Exercise B. Hperglycemia
65. Drug action wont b affected bound to alpha
glycoprotein in
a. Neoplasm b.Trauma c. Hepatic disease d. MI
66. Clinically GFR is measured by
a. Creatinine B. Inulin
67. 16 yr child with cola color urine since one day , Hb 6 , high grade
fever , was given antimalarial 1
day back ..
a.g6pd b.falciparum(blackwater fever)
c. PNH
68. The cause of death in gas gangrene is due to ;
A. Bacteremia B. Toxic shock
C. Pneumonia
69. which is not the content of mediastinum...?
a.lungs b.heart c.trachea d.vagus nerve e.large vessels
70. Diseases transferred thru vertebrate animals to
humans:
a)zoonosis b)zonoarthosac c)Zoncosis d) anthropozoonosis
71. about calcitonin...
a.must b given parenteral b.may b used in intoxication of vid D
c.hypoparathroidism
d.a polypeptide of 32 aminoacid
72. most common conginital heart dsz?
a)asd b)vsd
73. A man with AIDs who has no symptom with CD4 count 800
presented with suppurative otitis
media, causative organism is:
A. Mycobacterium B. Strep pneumoniae C. Pseudomonas aeruginosa
d)stap aureus
74. A pt can survive without mechanical ventilation, if the lesion is ;
A. Above C2
B. At C3
C. At C4
D. Below C5
75. Preganglionic sympathetic nerve fibers release ;
A. ACH b. Norepinephrine C. Dopamine
76. 5th Post intercostal artery
a. Thoracic aorta
77. Which one of the following is a COX II inhibitor ;
A. Aspirin B. Indomethacin C. Meloxicam
78. Fasting blood sugar of pt is 6.8 mmol (122 mg/dl) after 1 hr of
OGTT it is 10.9 mol and later after 2
hr postprandial it was 10.7 mol ( 194 mg/ dl ) what is the diagnosis
A. DM B. impaired glucose tolerance C. Long lag phase
79. Not recorded on ECG normally
A. SA NODE b.LEFT ATRIUM c.AV NODE d.RIGHT ATRIUM
80. After 48 hr of starvation, wht will be broken down to provide
energy,;
A. Carbohydrate B. Fat C. Protein
81. Which of the following causes increased interstitial pressure;
A. capillary permeability
B. colloid osmotic pressure
82. Post op wound causes greenish pus discharge . Which organism is
involved ;
A. Psudomonas B. Klebseilla C. E. Coli
83. Infection spreads retroperitoneally, which will b infected
a) Spleen b) Jejunum c) Transverse colon d. Ascending colon
84. Sprain of ankle leads to eversion of ankle, which ligament is
damaged .
a. Deltoid b. Talofibular c. peroneal d. tendocalcaneous
85. A pt, wd DK didnt respond to treatment and died, Autopsy show
inflammatory lesion in kidney
brain and lung, with some fungus. Most likely organism is ;
A. Mucormycosis B. Candida

86. Person naked in room, temperature 21 degree centigrade, humidity


present, loss of heat by ;
A. Insensible perspiration B. Sweating C. Conduction and radiation
87. Microscopic feature of polyarteritis nodosa is ;
A. Granuloma B. Fibrinoid necrosis C. Associated with hepatitis c
88. Which of the following structure damages in thyroidectomy ;
A. Recurrent laryngeal nerve damage B. External laryngeal nerve
damage C. Laryngeal oedema D. Sore throat E. Vagus nerve damage
89. ACTH effectively controls ?
a.Aldosterone b.Adrenal androgens c.Adrenal
estrogend.Hydrocortisone e.NE
90. which cannot b measured by spirometry
1.residual volume 2.expiratory reserve volume 3.inspiratory reserver
vol 4.tidal vol.
91. the following diseas occur in iv abuser
1)infective endocarditis 2)llibmansack endocarditis
92. Normal FEV1/ FVC ratio is ;
A. 0.6 B. 0.8 C. 1 D. 5
93. Pt unable to void after fall which of the following section of spinal
cord is affected;
A. L2, L3, L4, L5
B. L1, L2
C. S1, S2, S3
D. S2, S3, S4
94. osmotic diuretic effects on which part of nephron
a.proximal tubule b.collecting duct c.asc loop of henle d.glomerular
memb
95. After rapid loss of 2 liter of fluid . person drinks plain water . it ll
lead to
A. Increase in interstitial fluid only
B. Increase in intracellular fluid volume
C. Plasms volume increase
96. A patient known smoker in laryngoscopy found laryngal nodule
what is most unlikely
1 neoplasm
2 hyperplasia
3 keratosis
4 atrophy
97. secondary center of ossification
a.epiphysis b.epiphyseal plate
c.diaphysis
98. .liq uefactive necrosis.
a.brain b.kidney c.spleen
99. 1st heart sound
a.isovolumetric contraction
100. A 40 year old man has pain felt deep in the face and nose
between the mouth and orbit to the
ear and temporal region . the pain may be due to the lesion of the ;
A. auricular nerve b. facial nerve c. hypopharyngeal nerve d. maxillary
branch of the trigeminal nerve e. opthalmic branch of the trigeminal
nerve
101. EBV
nasopharyngeal carcinoma
102. inverse stretch reflex(resulting relaxation)
golgi tendon organ
103. fast sharp pain
A delta fibers
104. 3rd ventricle
bounded laterally by 2/3 thalamus (Rabia Ali MCQ)
105. glycogen breakdown
aerobic and anaerobic break down to pyruvate
106. fatty acids co enzyme
biotin
107. %age
units/total units X 100
108. The diagnostic lab test for the infection by streptococuss beta
haemolyticus is ;
A. ASO titre B. Blood culture C. ESR D. Leukocytes culture E. Urine for
bile pigment
109. disclosing pt's secret info
A. only with pt. consent B. Insurance Claim
110. Most common cause of inc bleeding time
a.thrombocytopenia
b. Aspirin
111. opsonin
C3b
112. plasma cells produce
antibodies

113. black water fever


a.p.falciparum
114.Sulphonylurea?? Dont remember the options but question was
asking about mechanism of action
115. alkalinize urine for removing
a.phenobarbitone
116. which one is true for parasympathetic sys on Heart
a.inc the AV nodal delay b. Decrease K ion outward current c.
Increase Ca++ inward Current d. Decrease Inward Na+ current
117. .parasympathetic action
a. sweat glands b.salivary glands
118.ectopic pregnancy most common site
a. uterine tube
119. both central n peripheral chemoreceptors respond to
a..arterial pCO2 b. Increase pH c. Decrease pO2
120. meckel's diverticulum
a.vitelline duct
121. .insulin dec by
a.beta blockers
122. most sensitive cardiac marker
a.trop T
b.Ckmb
123. Hypothyroidism
a. increased TSH
124. left adrenal vein drain into
a.left renal vein
125. fastest B.P regulation
a.baroreceptor
126. ESR inc in a.infections b. polycythemia
127.local factor delaying wound healing a.foreign body b. local
infection
128.actin covered by a.tropomyosin
129. turner a.44xo
130.klinefilters syndrome a.46XXY
131. S2,S3,S4 lesion a.rectal incontinence b.no pain in labour c.
anorgasmia
132. pain referred to shoulder a.c2c3 b.C3,C4,C5
133. compression of S1a.dec ankle jerk
134. not related to rt. kidney a.descending colon
135. external carotid a.lateral to retromandibulr vein
136.symphisis pubis a.sec. cartilaginous joint
137.fibrous and parietal pericardium supplied by a.phrenic
138. corona radiata a.granulosa cells
139. .cimetidine with sucralfate a.decrease metabolism b. Sucralfate
decreases cimetidine absorption, give it after 2 hours
140. Cimetidine may significantly prolong the prothrombin time and
causes bleeding in pt who has
been treated with warfarin by ;
A. decreasing the hepatic clearance of warfarin
B. Displacing warfarin from plasma protein C. Increasing the oral
bioavailability of warfarin D. Inhibiting renal tubular secretion of
warfarin
141. .sternocleidomastoid supplied by a.accessory nerve
142.regarding collagen a.present in all types of connective
tissueb.unique protein
145.organ for metabolism a.liver b kidney
146. Regarding Portal vein following statement are correct ; Except
A. Develops partly from the cranial part of right vitelline vein
B. is connected to veins of anterior abdominal wall through
paraumbilical vein C. Is formed by union of superior and inferior
mesenteric vein D. lies in the free margin of lesser omentum E.
tributaries include left gastric and superior pancreatoduodenal vein
147. compact bone... a.Regularly arranged lamelae
148. An initial left to right shunt before reversal is most commonly a
feature of ;
A. Coarction of the aorta B. Ebstein' s malformation C. Eisenmenger's
complex D. Transpostion of the great arteries E. Truncus arteriosis
149. Digoxine unlikly 1.inc Ca intracellular 2.inc K intracellular3.inc Na
intracellular 4.inc sympathetic to heart
150. u wave is associated with
A..atrial depo B .atrial repo C. ventricular repor 4. ventricl depo 5. slow
repolarization of papillary muscles
151. A 25year old sprinter developed acute leg pain while running. the
next day he noticed
ecchymosis around ankle . he can stand on his toes through it hurts .
the most likely cause is ;
a. acute arterial embolism
b. deep vein thrombosis

c. herinated lumbar disc


d. ruptured Achilles tendon
e. ruptured plantaris tendon
152.A 30 year male came to a consultant clinic with history of
progressive right facial numbness and
headach for 3 months, on examination, facial nerve palsy was
confirmed on right side along with VIII
nerve involvement of same side, and right sided Intention tremors, the
most likely site of lesion is ;
A. Cerebello pontine angle B. Pons C. Rt lobe of cerebellum D. Rt side
of mid brain
153. Wilsons disease
A. Ceruloplasmin
154. 16 yr old boy with diarhea,biopsy showin villous atrophy, not
improved by gluten free diet, most
likely diagnosis
a. Celiac disease b. Giardiasis c. Whipples disease
155. IM ketorolac given in
a. Renal failure b. Pregnancy
c. Suspected drug addicted complaining of pain
156. Water and salt reabsorption is maximum at
a. Duodenum b. Stomach c. jenunum d. Ileum e. Colon
157. wht happens in patient gettin TPN
a. Hyperglycemia
b. hyperphospatemia
158. epinephrine given slowly???
a. Decrease in TPR b. Increase in diastolic pressure
159. Premalignant oral lesion
a. Lichen planus b. Submucosal fibrosis
160. Question from biostatistics (Asked about Wrong Statement)
a.statistical data is of no significance b. Chi square
161. Nerve loops around arch of aorta
A. recurrent laryngeal nerve
162. Hemolytic anemia question?? Scenario was given and Correct
Answer was Hemolytic anemia
163. Regarding atonic bladder, it is caused by
A. damage to parasympathetic efferent pathway B. Damage to
sympathetic efferent pathway C. Cerebral cortex damage D. Pudendal
damage
164. Toxoplasmosis is inection caused by
a. Parasite b. Virus C. Fungus D. Bacteria
165. transmitted from eating undercooked meat a. tapeworm b. Liver
fluke
166. false statement about Carbimazole a. increases T4 to T3
conversion
167. lady with exophthalmos, thyrotoxicosis a. Graves b. MNG
168. Mitral stenosis, absent waves are a. c waves b. a waves
169. What wil decrease in hypotension a. Carotid sinus discharge
170. RBC fragility
a. 0.35 % NaCl cause complete hemolysis b. Hypotnonic cause
swelling c. Hypertonic causes shrinkage
171. Standing from 1 hour a. Decrease CVP
172. something about Gamma motor neuron
a. Relaxes muscle b. controls fiber 1a afferent c. contract the intrafusal
fibers
173. Arteriolar dilation leads to
a. increase in arterial blood volume b. increase in venous volume..c.
something about capillaries
174. Cause of Squamous Cell Carcinoma bladder
a. Shchistosoma hematobium
175.Post Commnicating artery
a. Internal carotid and Post Cerebral artery
176. Sjogrens Syndrome
a.dry mouth and eyes b. Dry mouth only c. Dry nasal mucosa
177. Which one is not epithelial tumour
a. Adenocarcinoma b. Liposarcoma c. Squamous cell Ca
178. Diphtheria toxoid given n thn aftr 2 weeks skin rashes , most likely
reaction
a. Type 1 b. type 2 c. Type 3 d. Type 4
179. Cardiac output low in
a. beri beri b. Cardiac failure c. AV fistula
d. hyperthyroidism
180 Labetalol
a. Beta blocker b. Alpha blockr c. Alpha plus Beta blocker
181.Monocytes likely
A. Can transfer into large multinucleated giant cells in chronic infection
B. Increase in allergy C. Produce IgM D. Formed from precursor cells

in lymph nodes E. Do not migrate across capillary wall unlike


granulocytes
182. not true abt surfuctant a.increase surface tension
183.Lipid diet a. Cholecystokinin
184. Reflex involving cerebrum
a Light reflex b. Consensual light reflex c.Accomodation reflex
d.Visual body reflex
185.Ocular myasthenia a. Decreased postsynaptic receptors
186.Oral anticoagulant
a.PT b. aPTT
187.Cushing syndrome a.ACTH
188.Renal excretion of drugs??? a.Something about Plasma proteins
189.graft rejection a. Acute Cellular
190.Ventilation of lungs a.increased at base
191. Failure of aorta to separate from Pulmonary artery
a. Truncus Arteriosus b. TOF
192. Peptic ulcer pain
a. Greater splanchnic Nerve
b. lesser splanchnic nerve
MOCK TEST 2014 (held on 22
nd
Oct 2014)
1. The papillae present on margins of the tongue is A) fungiform
papillae B) filliform papillae C) vallate papillae D) folate papillae Ans: A
exp: fungiform present on the margins filiform are spread all over the
dorsum of the tongue va,llate papillae are present anterior to sulcus
terminalis
2. Deep veins accompany arteries except in the A) abdomen B) thorax
C) cranium D) lower extremities Ans: C exp: deep veins usually
accompany arteries except in the cranium superficial veins also do not
accompany the artery
3. Cartilagenous joint present in the median plane of a body are called
as A) symphysis B) synchondrosis C) syndesmosis D) schindylesis
Ans: A exp: symphysis are the secondary cartilageous joint present in
the median plane of the body each body surface has a plate of hyaline
cartilage finally attached to it which in turn blends with a thick strong
deformable plate of fibrocartilage these joints allows restricted but
appreciable movement
4. Following arteries supplying a long bone arises from arteries
supplying muscles attached to the bone are A) metaphyseal arteries B)
epiphyseal arteries C) periosteal arteries D) nutrient arteries Ans: C
exp: at bone surfaces cortical capillaries make connections with
periosteal plexus which are formed by arteries from neighbouring
muscles contributing vascular arcades to the fibrous peritoneum
5. Following are the branches of lateral cord of brachial plexus except
A) ulnar nerve B) musculocutaneous nerve C) lateral pectoral nerve
D) lateral root of median nerve Ans: A exp: ulnar nerve is a branch of
medial cord of brachial plexus
6. Following are the tributaries of great saphenous vein except A)
superficial external pudendal B) superficial epigastric C) superficial
circumflex iliac D) deep circumflex iliac Ans: D exp: deep circumflex
iliac is a tributary of external iliac vein and not the saphenous vein
7. Compare with a person who ingest 2L of distilled water a person
with water deprivation will have a A) higher free water clearence (Ch20) B) lower plasma osmolarity C) lower circulating level of ADH D)
higher rate of H20 reabsorption in the collecting ducts Ans: D exp: the
person with water deprivation will have a higher plasma osmolarity and
higher circulating levels of ADH these effects will increase the rate of
H20 reabsorption in the collecting ducts and create a negative free
water clearence (C-H20)
8. Hypoxemia produces hyperventilation by a direct effect on the A)
carotid and aortic body receptors B) medullary chemoreceptors C) J
receptors D) lung stretch receptors Ans: A exp: hypoxemia stimulates
breathing by a direct effect on the peripheral chemoreceptors in the
carotid and aortic bodies medullary chemoreceptors are stimulated by
Co2 or H ions J receptors and lung stretch receptors are not
chemoreceptors

9. Regarding histology of gastrointestinal tract, which is inappropriate ?


A. Striated brush border is composed of microvilli
B. Peneth cells contain Eosinophilic granules in apical cytoplasm C.
chief cells of stomach secretes intrinsic factor D. Peyer's patches are
present in illeum E. The longitudinal coat of muscularis externa is
arranged in 3 bands in colon
Exp: Parietal cells secrete intrinsic factor
10. Regarding the axilla, one is inappropriate..
A. The axillary artery terminates at the lower border of Teres major. B.
In the neck, the plexus is closely related, posteriorly, to the fascia of
the scalenus medius C.The axillary lymph nodes receive 75% of the
lymphatics of the breast. D The third part of the axillary artery is related
to the cords of the brachial plexus. E Anterior fold of the axilla is higher
than the posterior fold.
Explanation: The second part of the axillary artery is related to the
cords of the brachial plexus.
11. Second part of dudenum is crossed anteriorly by? A. Right vein B.
IVC C. transverse colon D. Psoas muscl E. portal vien
Explanation: All others are posterior relation
12. Synthesis of secretory Vesicles takes place in? A Golgi apparatus
B Lysosomes C Ribosomes D Endoplasmic reticulum E Mitochondria
Explanation: It occur in E.R while packaging nd modification occur in
golgi bodies
13. Patient develops sensory loss over left side of body. After few days
his behavior for pain is changed & he become angry after touching.
Where could be the lesion in brain A. Internal capsule B. Thalamus
C.Cerebral cortex
D, Basal ganglia E. Midbrain
Explanation: he has Thalamic Pain Syndrome
14. Otic ganglion lies under? A. Foramen spinosum B. Foramen ovale
C. Maxillary nerve D. Stylomastoid foramen
15. The increase level of 5-hydroxyindolacetetic acid in urine is most
commonly found in? A. Phenylketonurea B. Alkaptonurea C. Carcinoid
syndrome D. Malignant melanoma E. DIC EXPLANATION: Urinary 5HIAA is the best initial test for Carcinoid syndrome.
16. Regareding Azathioprine ? A. Its in active form B. xanthine
oxidase is active metabolite form C. Belongs to the chemical class of
pyrimidine analogue D. The main adverse effect of azathioprine is
vasculitis E. It is an immunoproliferative drug
Explanation: . Xanthine oxidase is active metabolite form. its a prodrug,
The main adverse effect of azathioprine is bone marrow
suppression,and belongs to the
chemical class of purine analogues.
17. Which part of blood test should be sent for matching histo
compatibility of the recipient and donor A. WBC B. RBC C. Bone
Marrow D. Serum E. Platelets
Explanation: HLA = Human Leukocytes Antigen, Leukocytes = WBCs.
18. Difference between mean of two samples is tested by ? A. Chi
square test B. Pie test C. T test D. F Test E. Correlation analysis
19. "Menopause"" occur before "Menarche"..?? A. Patau.. B. Edwards
C. Turners D. Polycystic ovarian Diseases E. Angelamans syndrome
20. Major form of CO2 in the blood is as..?????? A. As dissolved state
in plasma B. As carboxy hemoglobin C. As carbnic Anhydrase in RBC
D. As Carbonic anhydrase in WBC.. E. In WBC cytoplasm
EXPLANATION: CO2 transport in blood
Bicarbonate 70% Carbaminohemoglobin 23% Dissolved CO2 7%
21. Nicotinic receptors sites includes all of the following EXCEPT; A.
Bronchial smooth muscles. B. Adrenal medullary Cells. C.
Parasympathetic Ganglia. D. Skeletal muscles. E. Sympathetic ganglia
Explanation: M3 receptors (Muscarinic NOT Nicotinic) & beta 2
receptors.
22. The earliest hormonal deffficeincy clinically evident on
hypopituatarism is??? A. ACTH B. LH and FSH C. Prolactin. D.TSH.
E. None of above

EXPLANATION: Thats why these pts will come with Amenorrhea/Loss


of Libido etc.
23. A localized lesion in which of the following produces prolong
coma???? A. Nucleus tractus solitarius.. B. Locus Ceruleus C. Right
frontal lobe D. Both Frontal lobes. E. Periaquiducatal region at top of
mid brain
[Reference Ganongs Nervous System Physiology Chapter]
24. When thoracotomy incision was made to enter the pleural cavity
following structure were incised except which one???
A. Skin and subcutaneous tissue
B. Pectoral muscle and serratus ant: muscle
C. Latissimus Dorsi muscle
D. External intercostals &inner most intercostal muscle
E. Internal intercostals
Explanation: Latissimus dorsi is a sheet of muscle that cover the back
of thoraeic cage and inserts into the floor of bicipital groove of the
humrus .It is located too far posterior to be damage by the exploratory
thoracotomy.
25. A patient shifts to the A/E Department with a penetrating injury to
the 5
intercostal space at mid axillary line which structure is unlikely to be
damage?
A Sup: vana cava
B. Liver
C. Right atrium
D. Upper lobe of Right lung
E. Right pulmonary arty
Ans .C thExplanation. As question asking unlikely damage structure
due to right 5
I/C injury Right atrium most unlikely to damage b/c right atrium of
heart which extends free 3
costal cartilage to 6th costal cartilage just to the right of the sternum
not in mid axillary line.
26. Atrophy of the short muscle of thumb mainly which nerve is
interpolated
A. Deep branch of radial nerve
B.Median nerve
C.Deep branch of ulnar nerve
D.Posterior interossus nerve
Ans.B
Explanation. Short Muscle of thumb is
Abductor pollicis brevis flexor pollicis and oppones pollicis these are
innervates by median nerve and Adductor pollicis oblique and
transverse head innervates by deep branch of ulnar nerve main nerve
is median
27. 25 years old office secretary wearing high heeled shoes inverts
and sprains her ankle while running down a flight of stairs which of the
following ligament did she most likely injured?
A. Anterior tibial &posterior tibial
B.Deltoid
C.Medial collateral and lateral collateral
D.Calcaneo_tibial
E.Calcano fibular and Anti: talofibular
Ans.B
Explanation. The most common type of ankle sprain is lateral which
occurs as a result of excessive inversion of the foot and dorsiflexion of
the ankle the calcano fibular and anti: talofibular ligament may tear
producing marked swelling and pain these two ligament combined to
the posterior talofibular ligament constitute the lateral ligament of ankle
28. 40 years old male come to the physician with complainant of some
think was protruding out from his anus after defecation 2 year history of
blood stained stools on proctosopic examination showed that mucous
membrane above the level of anal valves the swelling was Mucous
membrane contained large enlargement of vein beneath the
surface???
A. External hemorrhoid
B.Perianal abscess
C.Chronic anal fissure
D.Internal Hemorrhoid
E.Complete rectal prolapsed
Ans.D

Explanation .Internal Hemorrhoid are dilation of the tributaries of


superior rectal vein they are covered by mucous membrane in the
upper half of anal canal protruded down due to enlargement of veins
and pressure of defecation the dilated vein comes out from anus
29. 20 Years old biker has RTA come to A/E department with
compliant of unable to extend the wrist and figures which nerve
probably damage???
A. Median nerve
B.Ulnar nerve
C.Radial nerve
D.Musculo_cutaneous nerve
Ans .C
Explanation. Radial nerve lesion commonly result the fracture of the
mid shaft of the hummers which injures the nerve in the spiral groove
the patient unable to extend the wrist and fingers and there is wrist
drop.
30. Femoral sheath is formed by???
A. Fascia transversalis
B.Fascia transversalis & iliac fascia
C.Pectineal fascia
D.Lacunar ligament
E.Inguinal ligament
Ans.B
Explanation. The femoral sheath (crural sheath) is formed by
prolongation downward behind the inguinal ligament the tranversalis
fascia being continued down in front of the femoral vessels and the
iliac fascia behind then.
31. The highest rate of occurs of carcinoma of tongue is that of
A. Tip
B. Ventral surface
C.Dorsal surface
D.Lateral margin
E.Posterior surface
Ans.D
Explanation: The tongue is the most common intra_oral site of
carcinomas most tumors are the lateral margin extending into the
ventrum.
32. Lymph from the nail bed of the big toe drains into the which
lymphatic group???
A. Horizontal group of superficial inguinal nodes
B. Pre-sacral nodes
C. Popliteal nodes
D. Internal iliac nodes
E. Vertical group of superficial inguinal nodes
Ans. E
33. 3 years old children H/O avascular necrosis of head of femur what
is the chief arterial supply to the head of femur they compromise and
cause the avascular necrosis???
A. Obturator artery
B. Internal pudendal artery
C. Branch from medial and lateral circumflex femoral artery
D. Deep circumflex iliac artery
E. Superficial circumflex artery
Ans-A Explanation: the nutrient artery which is a branch of obturator
artery reaches the femoral head in children along the ligament of the
head and enters the bone at the fovea capitis
34. 40 YEARS OLD WOMEN TAKING ANTI-DEPRESSENT DRUG
HER
HUSBAND RECENDLY PASSED AWAY SHE DIAGNOSED AS
DEPRESSED AFTER START MEDICATION HER COMPLAINING OF
DROWSINESS, CONFUSION, AND FATIQUE DEPLETION OF
WHICH OF THE FOLLOWING ECTROLYTES MAY CAUSING THESE
SYMPTOMS??
A. ClB. Mg+
C. PSO4
D. K+
E. Na+ Correct answeExplanation: hyponateremia is associated more
frequently with selective
Serotonin reuptake inhibitors
Hyponatremia should be considered in all patients who develop
drowsiness .confusion, convulsion while taking anti-depressant

35. A 56 YEARS OLD LADY IS ADMITTED WITH 30/MINTS


HISTORY OF HEAVY CENTRAL CHEST PAIN ASSOCIATE WITH
NAUSEA AND SWEATING HER ECG SHOWS ST ELEVATION IN
LEADS V1.V2.V3&V4 WHICH OF THE
FOLLOWING
CORONARY ARTERY IS MOST LIKELY TO BE OCCLUTED ???
A. Circumflex artery
B. Left anterior descending artery
C. Marginal artery
D. Right coronary artery
E. Posterior descending artery
Correct answer B Explanation: anteroseptal M-l is due to an infarct in
the territory of the left anterior descending artery
36. AN 18 YEARS OLD BOY PRESENTED WITH TWO WEEK
HISTORY OF DYSURIA AND PURULENT PENILE DISCHARGE
GRAM STAIN OF URETHERAL SWAB SHOW GRAM NEGATIVE
INTRACELLULAR DIPLOCOCCI WHAT SPECIFIC TREATMENT
SHOULD HE RECIVE ???
A. Cefixime
B. Cephradine
C. Ciprofloxacin
D. Co-amoxiclav
E. Crystalline penicillin
Correct answer A
Explanation: N.gonorrhoea occurs in young age adult gram stain +ve in
25%
Culture +ve 50 % current guidelines recommend 1
line treatment with cefotaxime, cefixime or spectinomycin may give (as
stat dose)
Fluroquinolones are no longer used as 1
st
st
line due to the high rate of resistance
37. THE MANTOUX RACTIONS IS AN EXAMPLE OF WHICH TYPE
OF HYPERSENSITIVTY REACTIONS???
A. Type -1 H.S.R
B. Type-2 H.S.R
C. Type 3 H.S.R
D. Type 4 H.S.R
E. Humoral immune response
Correct _D EXPLINATION: tuberculin test is exp: of type 4 HSR or
delayed type of hypersensitivity reaction (DTH) this reaction develops
when primed Th1 cells encounter their specific antigens an
inflammatory response evolves over 24- 72 hours
38. ABGs OBTAINED & SHOWS A PH.7.51 AND PCO2 OF 48mmHg
ON BLOOD SAMPLE
Na+ =136meg/L K+ 3.2 meq/L Cl_= 100 meg/L HCo3 =37 meq/L
WHAT IS THE MOST LIKELY DIAGNOSIS???
A. Pt with vomiting
B. Pt with diabetic ketocidosis
C. pt with diarrhea
D. Pt with primary Aldosteronism
E. pt with hyperventilation syndrome
Correct answer A Explanation: the patient has an alkalemia (ph. More
then 7.45)
Since PCO2 is elevated (more 40 mmhg) the pt is primary metabolic
acidosis with respiratory compensation an elevation of HCO3 could
come from loss of acids as well as loss of intravascular volume
39. THE MEASURE OF CENTRAL TENDANCY THAT IS MOST
AFFECTED BY EXTRME SCORE IN A SAMPLE DISTRIBUTION IS
THE???
A. MEDIAN
B. MODE
C. STANDERD DEVIATION
D. MEAN
E. VARIAN CORRECT ANSWER D EXPLINATION; mean is the
measure of central tendency that most affected by extreme score
neither the standard deviation and no variance is A measure of central
tendency
40. REGARDING THE MYOPATHIES ALL ARE THE FOLLOWING
ARE TRUE EXCEPT???
A. duchenn myopathy is X-link dominant
B. Beckers myopathy is X-link recessive
C. Facioscapulo humeral dystrophy is autosomal dominant
D. Limb girdle dystrophy is autosomal recessive

E. Myotonic dystrophy is Autosomal dominant


Correct answer A
Explanation: Duchenne muscular dystrophy is inherited X-link
recessive disorder not Xlink
Dominant
41. Vertebral artery traverses all of the following EXCEPT
A. Foramen magnum
B. Subarachnoid space
C. Intervertebral foramen
D. Foramen transversarium
Ans. C
42. Which of the following elements is known to influence the body's
ability to handle
oxidative stress? A. Calcium B. Iron C. Potassium D. Selenium
Ans. D. Selenocysteine is part of enzyme glutathione peroxidase which
has antioxidant
action
44: The most common site of carcinoma of the colon is
a.Cecum
b.Ascending colon
c.Transverse colon
d.Splenic flexure
e. Rectosegmoid
[Reference Goljan: 50% cases]
43. Pulmonary compliance is not decreased in? A. COPD B.
Decreased surfactant production C. Pulmonary congestion D.
Pulmonary fibrosis
Correct answer : A. COPD Pulmonary compliance is increased in
emphysema (a type of chronic obstructive pulmonary disease). In the
other 3 conditions ( Decreased surfactant production, Pulmonary
congestion and Pulmonary fibrosis), the pulmonary compliance is
decreased.
45. What is the single most important test in diabetic ketoacidosis
(DKA)? a. Glucose level b. pH c. Ketone and/or acetone levels d.
Serum osmolarity Answer b. pH The glucose level is not as important
as knowing if the patient is acidotic. Glucose levels can fluctuate wildly
from high to medium, but if the patients pH on an ABG or serum
bicarbonate on chemistry is near normal, it does not matter. The same
is true of ketones, acetone, beta-hydroxybutyrate, or acetoacetate. The
level of these ketone bodies is not as important as the level of
accumulated acid.
46. The best initial test for Acromegaly is?
a. Insulin-like Growth Factor (IGF)
b. GH
c. MRI head
d. Glucose suppression test EXPLANATION: They are asking best
INITIAL test which IGF & most accurate test is Glucose suppression
test. NEVER start with radiology in Endocrinological diseases. First do
biochemical test to ensure that disease really exist then scan.
47. The earliest finding in hyperkalemia on an ECG is?
a. P wave disappear b. QRS widening c. AV block d. Peaked T wave
e. T wave flatting Explanation: Initial change on ECG is Peaked T
wave, then widening of QRS complex
etc.
48. 70 years old man admitted with palpitation to ER, ECG reveals
tachycardia rate of
170 bpm and blood pressure 120\80, which is ideal to give a.
Amiodarone b. Lidocane with central line
c. Procanamide d. Verapamil e. Defibrillate
Explanation: Pt in question is Hemodynomically stable (BP : 120/80)
so there is NO Point in defibrillating him. For Rate control the best
INITIAL drug choice is Amiodarone with central line. VERAPAMIL is
wrong. The descending order of medication (which will be used first)
Amiodarone > Lidocain > Procanamide.
49. Which of the following is the only hormone made continuously? a.
Cortisol b. Testosterone c. T4 d. Growth hormone (GH) e. ACTH
Answer c. T4 All hormones are made in pulsatile fashion except for T4
and T3, which are made continuously. They are all subject to feedback
inhibitionafter production, they go back and shut off their own

stimulatory hormone. Cortisol does feedback inhibition on the pituitary


and hypothalamus to shut off ACTH and corticotropin-releasing
hormone (CRH).
50. What receptor is affected most by exercise? a. GLUT1 in brain b.
GLUT2 in kidney and bowl c. GLUT3 in neurons d. GLUT4 in skeletal
muscle Answer d. GLUT4 in skeletal muscle Exercise does not affect
any of the glucose transporters except the one in skeletal muscle
51. Venous return of lower limb on standing from the supine posture
depends on all except
A) Deep fascia sleeve
B) Arterial pressure
C) Valves of perforators
D) Calf muscle contraction
EXPLANATION: All others options do has effect. ONLY arterial
pressure has NOTHING to do with venous return when pt change from
supine to standing position.
52. The dorsal roots of all spinal nerves contain
a) sensory neuronal processes.
b) sensory and autonomic neuronal processes.
c) motor neuronal processes.
d) motor and autonomic neuronal processes.
EXPLANATION: Dorsal roots only contain sensory neuronal processes
of spinal nerves. All ventral roots contain motor processes and the
thoracic, lumbar and sacral spinal nerves also autonomic nerve
processes.
53. Parasympathetic preganglionic axons leave the CNS with the
a) cervical spinal nerves.
b) thoracic spinal nerves.
c) lumbar spinal nerves.
d) sacral spinal nerve
Explanation: Parasympathetic axons exit the central nervous system
only with sacral spinal nerves and four of the cranial nerves.
Sympathetic preganglionic axons exit the CNS only with the thoracic
and upper lumbar spinal nerves
54. Which of the following endocrinopathies is associated with high
prolactin level?
a. Estrogen deficiency
b. Addison disease
c. Cushing syndrome or hypercortisolism
d. Hypothyroidism
e. GH deficiency
Answer d. Hypothyroidism
It is reasonable to exclude hypothyroidism in any person with
hyperprolactinemia. Increased estrogen directly stimulates the pituitary
to release prolactin. This is the mechanism in pregnancy of how breast
glands normally grow. The placenta makes estrogen. Estrogen makes
prolactin. Prolactin makes the breast glands grow.
55. Absolute refractory period in the heart
a. corresponds to the duration of relaxation
b. lasts till half of cardiac contraction
c. shorter than refractory period in skeletal muscle
d. lasts till cardiac contraction
56. Great saphaenous vein does NOT: A. lie behind medial malleolus
B. lies 1 hand's breadth behind medial border of patella C. drains into
femoral vein 3.5 cm below & lateral to pubic tubercle D. contains up to
20 valves E. most valves are found below the knee level
Explanation: it lies anterior to medial malleolus.
57. The hypothalamus:
a. forms part of the mid-brain.
b. forms part of the roof of the third ventricle.
c. is responsible for hormones and enzymes regulation.
d. contains the tuber cinerueum.
e. has nerve connections with the anterior lobe of the
pituitary gland.
58. The superior cervical ganglion: all are true except
a. contains both parasympathetic and sympathetic fibres
b. gives off branches that supply the heart

c. is the only cervical ganglion that supplies the cranial


nerves
d. gives off branches that supply the sweat glands of
the face
e,The SCG is located opposite the second and third cervical
vertebr.
59. Sterilization can be achieved with all, except
a. Pasteurization
b. gamma irradiation
c. moist heat at 1340C for 3 minutes at 202kPa
d. ethylene chamber
e. Tndylization
60. Normally (Physiological) Hemolysis is
a. extravascular
b. intra vascular
c. extra + intravascular
d. interiovascular
e. exterio interior
EXPLANATION: Normal hemolysis (i.e destruction of RBCs after 120
days) take place in SPLEEN and Extravascular hemolysis mean
Spleen & Liver. INTRAVascular hemolysis is ALWAYS pathological.
61. The relationship of the heart to lung is?
a. superior
b. lateral
c. dorsal
e. medial
d. medio lateral
EXPLANATION: Heart is surrounded by Lungs on either side. So it is
medial to lung.
62. Regarding histology of cervix, which of following is true?
a. squamous epithelium layer forms mucous secreting cells over
endocervix
b. transformation zone is a region where over time more fragile
squamous cells replaces columnar epithelia cells.
c.cells of ectocervix will nt collected in pap smear
d.In parous cervix appear as small slit like opening in center.
e. during pre adulscent endocervix is located in vaginal portion of
cervix
63. Regarding Blood Flow dynamics All are true, except
a. generally blood flow is laminar
b. ascending aorta have turbulent blood flow
c. turbulence decreases the perfusion pressure
d. turbulence increases with increased raynolds numbers
e. turbulence can generate murmurs
EXPLANATION: Turbulence will INCREASE perfusion PRESSURE.
64. Tall boy 14 yr old secretion from mammary gland, x-ray skull show
larged pituitary. Histological exam of pituitary is likely to show.
A. Dec number of pituicytes
B. Inc numbr of basophil cells
C. Inc numbr of chromophobes
D. Inc number of eosinophils
65. ESR is increased with decreased in?
a. fibrinogen
c. globulin
d. albumin
e. infection
b. age
EXPLANATION: ESR has DIRECT relation with Fibrinogen & Globulin
(will increase with increase in these two) while INVERSE relationship
with Albumin (will Increase with Decrease in Albumin)
66. Morphologically hallmark of apoptosis is, except?
a.condensation of nucleoli
b.karyorrhexix
c.shirinkage of cytoplasm
d.proteolytic cleavage
e. mitochondria damage is hall mark
Explanation: Mitochondria will INTACT in Apoptosis.
67. All of the following are features of apoptosis EXCEPT: a. Cell
swelling
b. Chromatin condensation c. Formation of cytoplasmic blebs d. Lack
of inflammation e. Phagocytosis of apoptotic bodies
EXPLANATION: Check Question 71 explanation.

68. All of the following are characterized by granulomatious


inflammation except
a) Sarcoidosis
b) Tuberculosis
c) Histoplasmosis
d) Diphtheria
e) Leprosy
EXPLANATION: You dont need it (explanation)
69. Silicosis is most often complicated by
a.Asthma
b.Carcinoma of lung
c.Mesothelioma
d.Tuberculosis
e. bronchi alveolar carcinoma
[Ref: Goljan Respiratory Chapter]
70. Condition is not associated with sinus bradycardia?
A.Brucellosis
B. Leptospirosis
C.Hypothyroidism
D.Advanced liver disease
E. Typhoid fever
EXPLANATION: Leptospirosis is associated with Tachycardia.
Contrary to our common belief that ONLY Typhoid is associated with
Bradycardia there are many other conditions (3 are given here) like
Falciparum Malaria etc.
71. All of the following are features Irreversible cell injury, Except?
a.acidosis
b.lysosomal swelling
c.mitochodreal swelling
d.damage to plasma membrane
e.plasma membrane blebbing & blunting
EXPLANATION: Reversible cell injury: cell swelling, detachment of
ribosomes from granular e.r. and dissociation of polysomes into
monosomes. Fatty change encountered in cells invloved in fat
metabolism (hepatocyte, myocardium). Histologically characterized by
pallor, hydropic change, vacuolar degeneration. EM: plasma
membrane blebbing, blunting, villous distortion, myelin figures,
mitochondrial swelling, rarefaction, nuclear disaggregation of granular
and fibrillar elements.
Irreversible cell injury: mitochondria swell, lysosomes swell, damage to
plasma membrane and lysosomal membranes leads to enzyme
leakage; acidosis somewhat protective by inhibiting enzymatic
reactions.
72. Serious adverse effect of morphine?
a) Extreme sedation
b) Increased intracranial pressure
c) Decreased respiration
d) Decreased myocardial conductivity
e) Decreased blood pressure
EXPLANATION: Respiratory depression is one of the most serious
adverse effect. You have to start Nalaxone to such pts.
73. Glucose Transported from mother to fetus by:
A. Primary active transport B. Facilitated diffusion C. Simple diffusion
D. Na co-transport E. None of the above
Ans: BBBB [Glucose via Facilitated (mainly), Amino-acids via Na- Cotransport]
74. A young female developed nausea vomiting and jaundice &
suddenly (within hours) she died which is most likely in this patient?
A. HAV B. Hep B C. Hep C D. Hep D E. . Hep E
Ans: Hep D (She is NOT pregnant, so E is unlikely and the HIGHEST
mortality is associated with Hepatitis D virus)
75. Transmission of which type of infection is most likely to occur after
platelet transfusion?
a. Syphilis.
b. Malaria.
c. Hepatitis B.
d. HIV.
e. Bacterial.

Explanation: Platelet are stored at room temperature and at room


temp. BACTERIAL transmission is common as their growth is
promoted.
76. What is the greatest stimulant to saliva production?
a. Acetylcholine
b. Epinephrine
c. Norepinephrine
d. Nicotinic receptors
Answer a. Acetylcholine
EXPLANATION: Saliva production is the only part of the body in which
the sympathetic and parasympathetic nervous system both lead to
increased production of a substance. The greatest stimulant, however,
by far, is acetylcholine. The acetylcholine receptors in the salivary
glands are muscarinic receptors not nicotinic receptors
77. Which test is most likely to be positive in SLE?
a. Antinuclear antibody (ANA)
b. Anti-dsDNA
c. Anti-Smith
d. RF
e. Anti-Jo
Answer a. Antinuclear antibody (ANA)
Although the ANA test has no specificity for SLE, it is the test most
commonly found to be positive. Ninety-five percent of patients with
SLE should be positive for ANA, this means the ANA has an
enormously strong negative predictive value for the disorder.
78. What is the single most specific test for multiple myeloma?
a. SPEP
b. Bone marrow biopsy
c. Urine immunoelectrophoresis
d. Bence Jones protein
e. Peripheral blood smear
Answer b. Bone marrow biopsy
Looking for >10% plasma cells on bone marrow biopsy is the single
most specific test for myeloma. When combined with a monoclonal
spike on SPEP and lytic lesions, this establishes the diagnosis of
myeloma. Some cancers can give lytic lesions. The monoclonal
gammopathy of unknown significance (MGUS) accounts for 99% of
IgG spikes on SPEP. You can have Bence Jones proteinuria alone
without myeloma. The only thing that gives >10% plasma cells on bone
marrow biopsy is myeloma.
79. Which of these can be associated with target cells?
a. Autoimmune hemolysis b. Iron deficiency c. Folic acid deficiency d.
Vitamin B12 deficiency EXPLANATION: Any of the microcytic anemias
can be associated with target cells. The most likely of all the anemias
to cause target cells is thalassemia. Both alpha- and betathalassemia
cancause target cells and thalassemia can also be caused by
thalassemia trait. Autoimmune hemolysi causes spherocytes not target
cells.
80. What will change first in response to iron replacement in Fe
deficiency anemia therapy?
a. MCV b. Reticulocyte count c. Hemoglobin level d. RDW
Answer b. Reticulocyte count
Reticulocytes are the measure of new cell formation. Reticulocyte
concentration has to increase first for any of the other features of a
CBC to change.
81. What test should be done to confirm the presence of vitamin B12
deficiency?
a. Methylmalonic acid level
b. Anti-intrinsic factor/antiparietal cell antibodies
c. Schilling test
d. Homocysteine level
Answer a. Methylmalonic acid level
Methylmalonic acid levels build up in vitamin B12 deficiency.
Homocysteine level increases in both vitamin B12 and folate
deficiency. Vitamin B12 deficiency can have a
low normal level in 10% to 20% of patients routinely. Anti-intrinsic
factor, antiparietal cell antibodies, and the Schilling test are not used to
discover whether there is a vitamin B12 deficiency. They are used to
identify the etiology of the deficiency.
82. 65 years old man with Hx of hypertension & epilepsy is noted to
have gingival hyperplasia on examination in the cardiology clinic.
Which one of the following is most likely to be responsible?

a. Sodium Valporate
b. Lisinopril
c. Atrovastatin
d. Nifedipine
e. Carbamazipine
EXPLANATION: Anti-psychotic drug which causes gingival hyperplasia
is Phenytoin NOT Valproate or Carbamezipine. Calcium channel
blockers (especially Nifedipine can also cause gingival hyperplasia)
83. What is the physiologic basis of a barrel chest?
a. Increased fibrosis of lungs
b. Increased residual volume
c. Decreased compliance
d. Decreased FEV1
Answer b. Increased residual volume
As air becomes trapped in the lungs, the diaphragm flattens and the
chest enlarges. These increased volumes are all residual volume, or
the part of the air that cannot be exhaled. This is all a sign of increased
total lung capacity, but not usable lung. The usable part of lung
volume, the FEV1 and FVC, both diminish.
84. The Most common cause of death in Adult polycystic kidney
disease is?
A. Berry Aneurysm B. Renal Cell Carcinoma C. Chronic Renal failure
D. Splenic Rupture E. Liver Cirrhosis
Answer: CRF (Ref: Goljan Kidney Diseases Chapter)
85. All of the following will cause Right shift of Oxygen dissociation
curve, EXCEPT:
A. Increase pCO2 B. Increase Temp C. Decrease pH D. Chronic
Acidosis
E. Increase 2.3-GDP Answer: D (In Option D will remain Same No
shift, as initial acidosis will cause Right shift but with pessage of time
the level of 2.3-GPD inside RBCs will decrease which move AWAY
Curve toward LEFT and the result will no NO Shift )
86. A 41-year-old woman was taken to the emergency department
after being involved in a motor vehicle accident. Neurologic
examination showed that the papillary light reflex was absent in both
eyes when light was shone in the left eye, but was normal in both eyes
when light was shone in the right eye. At which location could a lesion
account for this pattern of deficit in the pupillary light response? A.
Left occulomotor nerve (CN III) B. Left optic nerve C. Right EdingerWestphal nucleus D. Right visual cortex E. Superior colliculi
B: The absence of both the direct and consensual light reflex response
when light is shone in the left eye indicates a problem with the afferent
part of the pupillary light reflex pathway on the left. A normal response
on both sides when light is shone in the right eye demonstrates that
the efferent parts of the reflex pathway, including the Edinger Westphal
nucleus and cranial nerve III, are unaffected. The superior colliculus
and visual cortex are not involved in the pupillary light reflex.
87. Why does Mycoplasma not appear on Gram stain?
a. No cell wall
b. Intracellular
c. Too small
d. No stainable DNA
Answer a. No cell wall
Mycoplasma does not have a true cell wall. Mycoplasma is encased in
a cell membrane that does not pick up Gram stain. Chlamydia and
viruses are intracellular. Gram stain does not stain DNA, it stains the
cell wall
88. Which of the following parts of the eye has the greatest
concentration of rods? A. Ciliary body B. Iris C. Optic disk D. Fovea
E. Parafoveal region
Answer: EEEE (Ganongs MCQ)
89: Which of the following is the principal buffer in interstitial fluid? A.
Hemoglobin B. Other proteins C. Carbonic acid D. H 2 PO 4 E.
Compounds containing histidine
Answer: CCCCC (Ganongs MCQ)
90: Compared to hepatic bile, gallbladder bile contains a reduced
concentration of which of the following?
A. Bile acids B. Sodium ions C. Protons D. Glucose E. Protons
Answer: BBBBB (Ganongs MCQ)
91. Which of the following parameters is decreased during moderate
exercise? A. Arteriovenous O
2

difference B. Heart rate C. Cardiac output D. Pulse pressure E.


Total peripheral resistance (TPR)
Answer: EEEEE
92. Which of the following is the major immunoglobulin in human
serum, accounting for 80% of the immunoglobulin pool?
A) IgA B) IgD C) IgG D) IgM E) IgE
Answer: CCCC
Below are the some unique characteristics of each antibody:
(1) IgG = Smallest (in size) antibody, Highest concentration in serum,
One that crosses placenta and antibody that is known as a Warm
antibody.
(2) IgM = Largest in size, antibody that is mainly Intravascular, Earliest
antibody to be synthesized and Cold antibody.
(3) IgE = Reagenic antibody, antibody with Minimum life, Heat labile
antibody.
(4) IgA = Antibody that protect surfaces, antibody that is present in
body fluids like Saliva, Breast Milk
93. Pulmonary Arterial Hypertension may be seen in each one,
EXCEPT?
A. Hepatitis B
B. Eisenmengers Syndrome
C. Sickle cell disease
D. Sarcoidosis
E. HIV
EXPLANATION: Hep B is NOT associated with Pulmonary HTN.
94. Which one of the following is least associated with small-cell lung
carcinoma?
A. Cushings syndrome
B. Worst prognosis
C. Hyponatremia
D. Lambert Eaton Syndrome
E. PTH related protein
EXPLANATION: Small cell is associated with Cushings (due to ectopic
ACTH) & SIADH (due to ADH production). PTH-rp with squamous cell
carcinoma.
95. Preganglionic autonomic fibers are which type of fibers?
A. A Delta fibers B. A gamma fibers C. A beta fibers D. C fibers E. B
fibers
[Ref: BRS Neurophysiology Chapter table: Kindly do memorize this
table]
96. Receptors of all of the following are rapidly adopting receptors,
EXCEPT?
A. Pressure B. Velocity C. Vibration D. Tapping
[Ref: BRS Physio, Neurophysiology chapter]
97. Which one of the following hormone has 2 different mechanisms
action on its receptors? A. Insulin B. Angiotensin II C. ADH D. Vitamin
D E. ACTH
EXPLANATION: Insulin: Tyrosine kinase, Angiotensin II: IP3, Vitamin
D: steroid hormone mechanism, ACTH: cAMP mechanism, ADH (V1:
IP3 mechanism, V2: cAMP mechanism]
98. 55 years old woman had hysterectomy presents for advice about
HTR (Hormone Replacement Therapy). Which one of the following
would result from the use of a combined OCPs compared to an
Estrogen only:
A. Decrease risk of thromboembolism
B. Increase risk of stroke
C. Increase risk of breast cancer
D. Increase risk of endometrial cancer
E. Better control of symptoms
Explanation: Adding progesterone increases the risk of breast cancer
while decrease the risk of endometrial cancer
99. Patient with suspected COPD, which of the following is least
relevant?
A. X-ray chest B. Smoking C. CBC D. Peak expiratory flow E.
Spirometry
EXPLANATION: As per recommendations Peak expiratory flow has
NO value in COPD management. CBC is helpful in the sense that it
will EXCLUDE secondary polycythemia.
100: Which of the following Cephalosporin is effective in MeticillinResistant Staphylococcus Aureus (MRSA) infections?
A. Cefixime B. Ceftaroline C. Ceftazidime D. Ceftriaxone E. Cephalexin
Explanation: The ONLY cephalosporin with MRSA coverage is
Ceftaroline. MRSA is
resistant to ALL others cephalosporin.

MEDICINE & ALLIED 4


th
JUNE 2014 (Paper A & B Mixed)
1. MRSA sensitive to:
aztreonam
co amoxiclav clotrimoxazole metronidazole
2. primary hepatocellular carcinoma : aflatoxin b1
hep E hep C
alpha 1 antitrypsin deficiency
3. Braodman area 3 1 2
1 recieves all somatic sensation 2 special sensory
3 granular cortex something
4. serology not helpful in a) toxoplasmosis
b) ambisis c) giardiasis
d) ameobic liver abscess
6 Byssinosis occurres in people working in 1)garment factory
2)lead factory 3)textile industry 4)cement factory
7. Virus causes cancer via 1 proto oncoogene
2 oncogene
3 nucleic acid
4 promotr gene
Byssinosis occurres in people working in 1)garment factory
2)lead factory 3)textile industry 4)cement factory
OPD , 2 year old boy comes with fever, bowel disturbance... mcv mch
mchc all raised ... detected by
serum vit b12 levels
intrinsic factor antibodies
8.
9. 1st line phagocyte in blood neutrophil
monocyte macrophage
old aged Person living at 12000ft had a hx of of MI 6 year back...both
lower limbs swelling and bluish discoloration ...cause
A. cardio-pulmonary failure B. venous thrombosis
C. CO Poisoning
10. a lady delivered 34 weeks baby who was at 30th percentile of
weight and height, cleft lip cleft palate, microcephaly , 6 fingers in both
hands ... axial something ... whats the diagnosis :
a: 45X b: 46XXy
c: 74 XY+13 d: 74 XY + 18
11. Esophageal stage of swallowing most effected by :
myasthenia
cerebral cortex stroke just above lateral gyrus
Scleroderma
12. cancer cells combatted by :
natural killers (BOTH NK cells & CD8 are Anti-Neoplastic) cd8 and t
lymphos
cell mediated
13. TCA.... what is false:
A. sedation
B. alpha blocker
C. Sympthomimetic
D. beta agonist activity
14. DIC is stimulated by?? tissue thromboplastin
15. Thiazides causes.. Hypokalemia Hyponatremia Hypocalcemia
16. Hyperpolarization is caused by.. Na+ influx
K+ influx Cl- influx
17. Pt of chronic renal failure going to dialysis..normocytic
normochromic anemia .. Drug given:
Erythropoietin Frolic acid
Vit b6 Vit b12
18. Tricuspid auscultation area? A. 2nd right ic space
B. 2Nd left ic space C. 3Rd left ic space D. Mitral area
E. Lower half of left border of sternum
19. Parotid ducts opens into (Asim MCQ)
Second upper molar teeth between cheeks and teeth
20. Histamine releasing opioid analgesic.
A. Morphine
Tramol
butyrophenie
fentanyl
21. vitamin b12 def can be found out by urine test of
methylmalonic acid pyruvic acid

22. a patient known case of lambago comes with sudden left side
chest pain.. Cardiac enzymes r normal.. No retrosternal pain also not
radiated.. But tenderness at 5,6 intercostal spaces at posterior side..
MI Costochondritis Aortic aneurysm Pleuritis
23. There is lesion of upper trunk.. All wil occur except (Asims MCQ)
adduction of arm
medial rotation of arm Sensory loss over forearm Elbow extension
Supination
24. Typhoid diagnose in 2nd week best by :
widal blood cs
blood cs plus widal only widal
stool cs
25. Pt with nose lesion , granuloma, proteinuria leprpsy wegner's
26. Which hormone not required for growth of sperm:
GH bHCG Prolactin FSH
27. Steatorrhea due 2 chronic pancreatitis occur due to deficient:
1,amylase 2.lipase
28. Female with lush push bodyline , well develped brest ,,
complained amenorrhoea since puberty , short vagina , uterus not
palpable, testes in inguinal region.
male psuedohermaphrodite female pseudoherma testicular
feminization
29. Dissection of aorta:
medial necrosis (Cystic Medical Necrosis Rubins Pathology)
30. Subclavian artery arches over: (Asims MCQ)
clavicle 1st rib 2nd rib
31. Whats the hallmark of aids:
progressive immune-depression opportunistic infections Proliferation
of virus inside Tlymphocytes
32. in tractus solitarius A. 1st order neuron of taste (ant 2/3 tongue)
B. 2nd order neuron of taste (ant 2/3 tongue)
33. wound healing
ascorbic acid
34. Autosomal dominant. whats wrong:
50% offspring effected complete penetrance 50% females effected
Variable expression
35. radiation causes cancer usually in : A. 5-10 yr
B. more than 10 yr36.
what white blood cells are raised in acute viral infection A.
Neutrophils
B. Lymphocytes
37. protein fat carbohydrates deposition by A. GH
B. glucagon C. insulin
70 yeard old lady...vision not clear A. Cataract
B. Stabismus C. Astigmatism
inappropriate about adrenal gland
neonate adrenal is same the size of adult adrenal gland
Artificial respiration is not necessary if lesion occur at.. A.c2 B.c3
C.above c5 D.below c5
An immunocompromize pt developed s/s of pneumoia BAL fluid
revealed boat shaped cyst organism (Nov 2013 Medicine paper MCQ
= repeated) Pt unable to invert his foot while the lateral rotation is
intact structure
involved
A. tibialis ant & tibialis post
B. tibials ant & fallicus H longus
38. Appendix ..pain in flexion & Medial Rotation of hip. (Asim &
Shoaib MCQ) A. Pelvic
B. Retrocecal C. Preileal
D. Post ileal
39. body utilizs nutrients in what order:
CHO, fats , protein fat, prot CHO
cho Prot fat
40. myasthenia gravis Pt, Dr gv some medication due to which Pt
conditions improved. Whats mode of action of medication:
anticholinesterase chlonergic anticholinergic
41. Skeletal muscles
A. are innervated by somatic nervous system B. are non straited
C. intercalated disc
42. Shock characteristic: tissue hypoxia
low BP tachycardia
43. Scenario about nerve supply to tip of the nose Opthalmic
division of trigeminal nerve
44. Neurogenic shock mediated by:
loss of vasomotor tone
heart pumping decrease
45. Scenario on lacrimation while eating like that Facial nerve

46. something about secondary active transport (Amino Acid


absorption in PCT)
47. what sound comes on rapid filling of lft ventricle
48. S3 S2
49. If Ejection fraction increase (Rabia Ali MCQ with wrong key AND
BRS Physio MCQ)
EDV decrease
ESV decrease
50. Woman with borderline lab values of MCV, MHCH etc:
microcytic normo
macrocytic macro micocytic hypochromic
51. Lumber puncture ...predominates lymphocytes, HIGH Protein &
Normal Glucose (Asims & Chandkians MCQ)
A. TBM
B. Bacterial meningitis
52. Nutrient artery supplies:
A. epiphyseal plate D. flat plate
C. epiphysis
D. Diaphysis (Supply Diaphysis & Central portion of Metaphysis.
Peripheral portion of Metaphysis is supplied by separate artery
Metaphyseal artery)
E. metaphysic
what white blood cells are raised in acute viral infection A. Neutrophils
B. Lymphocytes
53. Vitamin K deficiency, defective clot formation i guess
A. fibrogen
B. fibrin
70 yeard old lady...vision not clear A. Cataract
B. Stabismus
C. Astigmatism
inappropriate about adrenal gland
neonate adrenal is same the size of adult adrenal gland
Artificial respiration is not necessary if lesion occur at.. A.c2 B.c3
C.above c5 D.below c5
An immunocompromize pt developed s/s of pneumonia BAL fluid
revealed boat shaped cyst organism (Nov 2013 Medicine paper MCQ
= repeated)
Pt unable to invert his foot while the lateral rotation is intact structure
involved
A. tibialis ant & tibialis post
B. tibials ant & fallicus H longus
Vomiting center situated Medulla oblongata
Lower motor neuron lesion A. Decrease motor tone
B. spasticaortic opening aneurysm structures compressd...azygous
vein and thoracic duct
3. thrombin
4. thromboplastin 5. Prothrombin
54. Osteogenic imperfecta Defect in synthesis of collagen
55. liver in its place coz of
attachment with IVC ligaments
56. Jugulodiagastric lymph node drains into
Palatine tonsils and tongue
57. Right adrenal gland
Pyrimidal in shape
58. oblique fissure in lung extends from : (Asims MCQ)
t3 - t6 t2 t6 t4 t6
59.
60. factor 9 def, aptt high PT low
what white blood cells are raised in acute viral infection A. Neutrophils
B. Lymphocytes
Christmas disease factor 8 def
70 yeard old lady...vision not clear A. Cataract
B. Stabismus
C. Astigmatism
inappropriate about adrenal gland
61. sixty year old lady having unilateral severe head ache...temporal
atery biopsy
A. Giant cell artiritis B. Takayasu
neonate adrenal is same the size of adult adrenal gland
Artificial respiration is not necessary if lesion occur at.. A.c2 B.c3
C.above c5 D.below c5
An immunocompromize pt developed s/s of pneumonia BAL fluid
revealed boat shaped cyst organism (Nov 2013 Medicine paper MCQ
= repeated) Pt unable to invert his foot while the lateral rotation is
intact structure
involved

A. tibialis ant & tibialis post


B. tibials ant & fallicus H longus
Vomiting center situated Medulla oblongata
Lower motor neuron lesion A. Decrease motor tone
B. spastic
aortic opening aneurysm structures compressd...azygous vein and
thoracic duct
62. bile acids
absorbed in ileum
63. bile acids helps fat digestion by:
Facilitate absorption of Lipid
Hydrolysis of LIPID
64. stone in the parotid duct (Asims MCQ)
65.
Apoptosis
necrosis degeration
66. epithelium changes in endocervix Metaplasia
dysplasia
67. Squamous cell Carcinoma
marjolin ulcer bowen's disease
68. Peroxisomes
Long chain fatty acid
what white blood cells are raised in acute viral infection A. Neutrophils
B. Lymphocytes
69. Pt for renal transplant , what investigation is not necessary in the
donor: (Asims MCQ)
Serum Renin UCE
HLA CBC
70 yeard old lady...vision not clear A. Cataract B. Stabismus C.
Astigmatism
inappropriate about adrenal gland
neonate adrenal is same the size of adult adrenal gland
Artificial respiration is not necessary if lesion occur at.. A.c2 B.c3
C.above c5 D.below c5
An immunocompromize pt developed s/s of pneumonia BAL fluid
revealed boat shaped cyst organism (Nov 2013 Medicine paper MCQ
= repeated)
Pt unable to invert his foot while the lateral rotation is intact structure
involved A. tibialis ant & tibialis post
B. tibials ant & fallicus H longus
Vomiting center situated Medulla oblongata Lower motor neuron
lesion A. Decrease motor tone
B. spastic
aortic opening aneurysm structures compressd...azygous vein and
thoracic duct
70. Hormone involved in gluconeogenisis Glucagon
71. Inhibit gastric acid secretions (Asims MCQ)
Secretin
72. Naked Person with 80% humidity in air & 21 degree temp: heat
loss by (Ganongs MCQ: Ch# 17 MCQ#2)
A. conduction and radiation B. By vaporization of sweat
C. Elevated metabolism
D. Respiration
E. Urinatio
73. Derived from neural crest (Asims MCQ) autonomic ganglia
74. Rapidly adopting fibers... A. baroreceptors, B. pacinian
corpuscles, C. meissner
75. Nutritional deficiency..scaly dermatitis,alopecia Zinc
76. Not supplied by vagus nerve (Asims MCQ) A. stomach
B. ascending colon C. Jejunum
77. Which will shift the O2.HB curve to the extreme left...
A. CO poisoning
B. fetal Hb ,
C. acidosis,
D. inc 2,3 bis
78. Secretary vesicles found in organelle
A. Golgi apparatus (Transport vesicle from ER to Golgi Modification
of Transport vesicle inside Golgi Secretary Vesicle)
B. ER
79. Cristanee concavity toward nucleus Golgi apparatus
80. Inappropriate about atropine..
A. Increase CUTANEOUS BLOOD FLOW
B contraindicated in inc intra-ocular pressure
81. Widest part of epidural space A. L2
B. C7
82. About collagen.... most abundant protein in the body
83. High pressure oxygen for 24 hrs (Scenario of Some Smoker).
Spontaneous Pneumothorax

84. Warfarin given for DVT, what is Appropriate?


Not given in pregnancy
Rapidly antagonized by FFP
85. 5th week what happens?
A. Heart has definite form (NOT True as Upper as it occurs at the end
of 8
week ) B. Limb buds appear (NOT True as Upper limb buds in 4
In These options (If other 3 were wrong, then Option B is appropriate.)
week & Lower limb in 5
86. Neural tube is formed in which week A. 3
B. 4
C. 5
87. 2*2 TABLE
Chi-square testing
88. Sample which is age based grouped then random people taken
from each group, IT IS? (Rabia Ali MCQ Page 484)
A. stratified sampling B. simple sampling
89. Meningitis after pyogenic lung abcess organism? (Asims,
Chandkians & Rabia Ali MCQ)
A. Staph
B. Pneumococci
90. Endometrial hyperplasia A. inc estrogen
B. dec estrogen
C. inc progesterone
91. PYOGENIC PEROTINITIS A. Bacteroids
B. E.Coli
Reference: GOLDMANs Cecil Medicine 24
thed and Harrisons Principles & Practice of Medicine 18
92. Organ with high atriovenous (AV) 02 difference at rest?
A. Heart
B. Kidney
C. Brain
D. Skeletal muscle
th
Ed.
93. Hormone that stores carbohydrates, lipids and proteins in cells
is also called hormones of abundance
A. GH
B. Cortisol
C. Thyroid hormone D. Estrogen
E. Insulin
94. Starling law of heart (Ganongs MCQ: Ch#30, Q# 6) A.
Explain the increase in cardiac output that occur when venous return is
increase
B. explain the inc in heart rate by exercise.
C. does not operate during exercise.
D. explains the increase in cardiac output when the sympathetic
nerves supplying the heart are stimulated.
E. does not operate in the failing heart.
95. Trigeminal ganglia
A. Completely covered by dura B. Freely bathed in csf
C. Lies in MCF on the side of cavernous sinus
96. Short limb..short height...intelligence normal (case of
achondroplasia) A. Autosomal dominant
B. Autosomal recessive
97. Drug which enhances the hypoglycemic effect of sulphonylurea?
A. Phenylbutazone, (Ref: Textbook of Diabetes Table 24-6)
B. thiazide.
C. OCP
98. Tx of hypercalcemia
A. Calcinomimetic
B. Bisphosph C. Furesimide
99. 45x0/46xy....A short stature n prominent crease line...
A. BRACHECEPHALY AND MENTAL RETARDATION
B. GONADAL DYSGENESIS WITH FOLLICULAR APLASIA
100. Neostigmine action: inhibit Ach esterase
101. Venous return increased by A. intrathorcic pressure
B. Valve in veins
C. Muscle contraction of leg
102. A LADY HAD AN ROAD ACCIDENT AND GOT HEAD
INJURY.SHE LATER
DEVELOPED complaint of polydypsia and polyuria..lab findings were
given in which urine osmolarity is low and plasma osmolarity high.
After giving vasopressin urine osmolarity increased and plasma
osomalilty decreasedwhat is diagnosis???
A. Central Diabetes Insipidus
B. SIADH
C. Nephrogenic Diabetes Insipidus

103. Chrons disease diff from Ulcerative colitis Perianal lesions


104.
what white blood cells are raised in acute viral infection A. Neutrophils
B. Lymphocytes
A) Toxoplasma gondii
B) H. influenza
70 yeard old lady...vision not clear A. Cataract
B. Stabismus
C. Astigmatism
C) Pneumocystis carinii
inappropriate about adrenal gland
neonate adrenal is same the size of adult adrenal gland
Artificial respiration is not necessary if lesion occur at.. A.c2 B.c3
C.above c5 D.below c5
An immunocompromize pt developed s/s of pneumonia BAL fluid
revealed boat shaped cyst organism (Nov 2013 Medicine paper MCQ
= repeated)
Pt unable to invert his foot while the lateral rotation is intact structure
involved
105. Hormone inhibited by feedback from hypothalamus? Prolactin
106. Pneumothorax
Ipsilatrral lung collapse and chest wall spring out
107. A patient burning and recovered completely what factor
determines that
complete recovery will occur?
,A. underlying Granulation tissue B. local edema C. Blood supply
108. Typhoid test (Asims MCQ) A. 1:160 O antigen
B. 1:160 H ag C. 1:80 O ag
D. 1:160 O and H antigen
.110. What happen in lower motor neuron lesion..?? A. muscle
wasting.. B hypertonia..
113. Chorea, athetosis region involved
Vomiting center situated Medulla oblongata
Lower motor neuron lesion A. Decrease motor tone B. spastic
aortic opening aneurysm structures compressd...azygous vein and
thoracic duct
70 yeard old lady...vision not clear A. Cataract
B. Stabismus
Vomiting center situated Medulla oblongata
Lower motor neuron lesion A. Decrease motor tone
114. a lady have difficult in going down stairs examination shows head
tilted to left and rite eye was upward ocular musle involved?
A. right sup oblique.. B. inf rectus.. C sup rectus
115. Area of the brain most activated before performing skilled motor
activity
A) Precentral gyrus
B)spinal motor neuron
C) cerebellum.
D) Cortical association areas (Ganongs MCQ: Ch: 12 Q: 5)
116. pupillary light reflex carried by..
A. 2nd & 3CN B. 3rd and 4rd Cranial Nerves th
117. Some Scenario of UNILATERAL MUSCLE ATROPHY WITH NO
SENSORY DEFICIT? like that..dont rememeber exactly...lesion?
A. upper motor neuron B. Lower motor neuton
C. Lower motor neuron with ant horn of spinal cord
118. Asthma scenario.. Dec FEV 1/FVC 75
119. Pt with chemotherapy induced vomiting drug to b given
A. metoclopramide
B. domeperidon C. OndaNsetron
120. Artery in TRUE PELVIS Middle rectal artery
121. Estrogen is released by?
A. ovary
B. uterus
122. Which is malignant?
A. Glioma
B. Meningioma
C. H. mole
123. Clostridium perfingens exotoxin action by..
A. coagulase .
B. phopholipase
124. Pt with bronchial ca present with hemoptysis due to?? (Asims
MCQ)
A) Erosion of bronchial vessel
B) pulm vein
C) pulm artery.

125. Cavernous sinus thrombosis A. superior opthalmic vein B. inferior


opthalmic vein
126. Inappropriate about hcg..
A. It causes ovulation..
B. Little similar as thyroid,
C. it causes LEUITINIZATION OF FOLLICLES
127. major part of energy utilized during breathing is to overcome
(Asims MCQ)
A. Elastic recoil of lungs
B. Resistance of chest wall C. Large airway resistance
128. Side effect of chlorpromazine.. A. dystonia
129. Immunological function is significantly reduced in
A. dec basophils
B. ACTH
C. dec neutrophis
D. liver failure
130. Patient came with complaint of hirsutism .muscle wasting .skin
pigmentation and truncal obesity, hyprtension hyprglycemia...
A.Cushing disease
B. Primary inc in ACTH
131. Decrease in PR interval
A. 1
st
degree Heart Block
B. 2nd degree Heart Block
C. WPW syndrome
132. Slow Pain transmitted by unmyelinated c fibers..
A. substance p
B. serotonin
C. Ach
133. Which hormone decrease gastric motility..??? A. CCK .. B
secretin.. C. gastrin
134. Minimal change nephrotic syndrome...microscopic finding
A. mesengial deposition
B. sub epithelial deposition C. sub endothelial deposition
D. Linear IgG deposition on IMMUNOFLORESCENCE
(Confusion is in Qs stem. If it was Goodpastures Syndrome then
Option D is Correct)
135. Fracture behind humeral medial epicondile loss of sensation in
A) medial/3palmer aspects
B) Medial 1/3 palmer & dorsal aspect
C) Lateral 1/3
136. A sluggish pendular knee jerk due to..?
A. Anterior horn neurons damage
B. upper motor neuron lesion
137. what white blood cells are raised in acute viral infection A.
Neutrophils
138. Microaneurysm feature of AIDs dont include..
A. hard exudates
B. Vasculitis
140. Aids not associated with..
A squamous cell carcinoma..
B lymphoid series of bone marrow (Leukemia is Unlikely in AIDS.
Option B is correct NOT bcoz of Lymphoma but because of BONE
MARROW, Lymphoma can occur in HIV pt, in-fact CNS Lymphoma is
the 2
nd
most common cancer in HIV pts )
141. Lymph nodes not present in?
A. Spleen
B. Thymus
142. Basic drugs bind to..?? (Chandkians MCQ with wrong key
answer)
A. alpha 1 glycoprotein..
B. albumin
143. Thrombocytopenia APPROPRIATE
A. inc clotting time causing dec platelet count
B. thrombocytonpenia impairs APTT pathway
C. DIC occurs due to Dec in platelet count/ DIC causes decrease in
platelet count
144. A child with thrombocytopenia after acute viral infection due
to....
A. Anti-platelet antibody (Explanation: In ITP Body own cells attack on
Platelets The reason why We give Steroids or do Splenectomy is
because it is Autoimmune process and SPLEEN is the main culprit
site. Antibodies Formed in SPLEEN BY Plasma Cells). B. immune
complex formation.
C. consumptional coagulopathy

145. Minimal change in blood flow to an organ during exercise


A) Heart B) BrainC) Skeletal muscles D) Kidneys. E) Lung.
146. Congenital anomaly associated with urothelial CA is
A. bladder extrophy (Adenocarcinoma has association with B. Extrophy
Conns Current Therapy 2013)
B. unilateral agenesis of kidney C. double ureter
D. Horse shoe kidney
147. Lipoprotein with highest cholesterol content (Asims MCQ) A) HDL
B) LDL C) IDL E) Chylomicrons
148. One question about leprosy drugs (Asked about Inappropriate
statement):
Answer = CLOFAZIMINE IS NOT GIVEN IN PT RESISTENT TO
DAPSONE
149. Galactorrhea (Asims MCQ with little modification of question
stem)
A. Chromophoes are smallest B. chromophils are 50% of total
150. Scenario of turner syndrome.
A. bracheceephale with mental retardation short stature with
prominent simian crease B. gonadal dysgenesis with follicular aplasia
151. Inappropriate about dopamine.
A: contraindicated in septic shock...
B. causes increase myocardial infarct area..
C. 10 microgram per kg dose of dopamine causes vasoconstriction
a. Hyernatremia A. heart failure
B. excessive ADH
152. A vertical lesion in optic chiasma causes..
A bitemporal hemiaopia..
B. right sided homonomous hemianopia
153. Warfarin causes decrease in..
A. Fibrinogen
B. Factor 13
C. Prothrombin
154. Essential amino acid is.. Phenylalanine
155. Atomic bomb,... release of Stroncium-90 which affect milk
products..so it causes
A.osteosarcoma B.nasopharyngeak CA 3.leukemia
156. Vertical artery along face, deep to submandibular gland is..?? A.
Facial B. Maxillary C. Lingual
157. Tumor which also invade neural tissue
A.adenoCA B.hemangiopericytoma
158. which one is NOT autosomal recessive
A. ataxia talengiectasia B. carcinoma ovary 159. good patient and
docrelationship...active listening
160. One glucose give ATP.. A. 38 (8 in Glycolysis & 30 from Krebs
Cylce) | some folks are using FA as reference that it is 32 BUT it is
NOT Correct. According to Modern P/O (Phosphate/Oxygen) ratio 1
NADH2 = 2.5 ATP & 1 FADH2 = 1.5 ATP. The sum of ALL ATPs,
NADH2 & FADH2 produced by 1 molecule of glucose is equal to 31
ATPs (NOT 32) So again 32 is NOT correct. ALL Major Biochemistry
Texts still follow the Traditional (Old) P/O ratio (NADH2 = 3ATP &
FADH2 = 2 ATP) So Correct answer is 38
161. Digoxin don't cause
A. inc intracellular Ca
B. inc intracellular Na
C. inc intracellular K
162. inferior thyroid artery is branch of thyrocervical trunk
163. Pt with gum bleed in petechia advised bone marrow likely DX
A) Lymphoma B) Leucopenia C) Leukemia
164. fluid having specific gravity of 1.006 cause is A. CCF B. Ca lung
C. pneumonia D. TB
165. a patient came in semi comatose state,,shifting dullness was
positive..liver biopsy taken and there were mallory bodies n fibrosis of
liver : Alcoholic Hepatitis
166. A lady after abdominal surgery presented with dyspnea susp DVT
cause (Asims MCQs)
A) Endothelial injury
B) Endothelial injury & stasis
C) Stasis & hypercoagulablity.
167. Broadmans Area 3 2 1,,,
A. agranular cortex..
B. Primary Somatosensory Cortex
168. secondary center of ossification (Asims MCQ) EPIPHYSIS
169. acclimatized...rapid ascent
a) 2 3 DPG b) PO2 is normal c) PO2 is lower than normal d)
Pulmonary edema

170. Pregnant lady with normocytic normochromic Anemia. Cause?


plasma volume expansion
171. Adrenal insensitivity syndrome which is inappropriate test
A. Serum LH FSH B. Serum progesterone C. US abd
172. Endocervical epithelium of lady reveals sq epi cause? A)
Dysplasia B) Anaplasia C) metaplasia
173. Water Intoxication causes??? (Asims MCQ) A) Slow pulse B)
Thirst C) Altered behavior.
174. In complete heart block
A. Atrial rate is lower than ventricular rate B. Ventricular fibrillation is
common
C. Fainting occur cuz atrial unable to pump blood into ventricles
D. Fainting occur because of prolong periods during which ventricles
fail to Contract
175. Burning hot sensation occurs when temp of the surrounding is
A.28
B.32 C.42 D. Above 45 (Thermal nocioreceptors activate when
Temperature Falls Below 5 C COLD or shoot Above 45 C)
176. Slow growing Tumor/Carcinoma of ThyroidA. Follicular B.
Papillary
177. Low molecular weight heparin
A) can cause thrombocytopenia
B) produced by fragmentation n degradation of heparin
178. Clostridium tetani main site of action is? A. cerebral cortex B.
Exotoxin produces lethal effects
179. Pt having Hypertension 210/120 and Creatinine 7, defect is in?
(Rabia Ali MCQ)
A. JG apparatus B. Mecula Densa
180. Gastrectomy after 3 months anemia
A. microcytic hypochromic B. megaloblastic
181. Drug contraindicated in impaired renal failure: Gentamicin
182. Solubility of drug is increased by which factor A. increase lipid
solubility
183. Gastric secretions are inhibited by what. A. secretin B. Ach
184. In slow wave sleep what is unlikely.
A. Inc muscle tone B. Dreams unable to recall C. Delta waves
185. Bradykinin ... A. kallikren
186. Alpha adrenergic stimulation causes
A.. Vasodilation B. Lipolysis C. Pupillary dilation
187. GFR Clinically Measure by: A. Creatinine B. Inulin C. PAH
188. Inappropriate regarding Azygous Vein Pass through Esophageal
Opening
189. Rh Agglutinins A. present of RBC surface B. present in month of
Rh -ve mother when give birth to rh +ve fetus
190. Artery involved Anterior wall MI LAD artery
191. Taste fibers through. tractus solitarius
192. After giving Glucocorticoids, there will be? (Asims MCQ)
Decrease Lymphocytes Decrease Neutrophil
193. Multiple long bone fracture...B.P 90/, Pulse 115. Initial
management? (Asims MCQ) A. Fracture stabilization B. Volume
replacement C. Tracheostomy
194. Atrial fibrillation A. Pulses deficit
195. The process by which DNA is transferred from one bacterium to
another by a virus is known as A. TRANSDUCTION
196. Fracture of Long Bones Fat Embolism
197. Thirst is Reduced By? (Asims MCQ)
A. Angiotensin II B. INC Volume of ECF C. Stimulation of
Hypothalamus

Controversial Mcqs Solved By Admin Team With final Answers)


Credits : Chief Admins (Umbreen Hashim And Mehtab Alam )
1)patient on ETT, heat loss is by
evaporation
..
2)..rubella may cause cardiac defect in embyo at
8 to 9 wks s0 select the option nearest to it
3)nipple and areola of multiparouse womans breast are usually at the
level
fifth intercostals

4)pt when asked to look left,his left eye showed internal squint indicate
injury of
trochlear
5)pathogenesis of venouse thrombosis in a60 year old female after
major abdominal
operation
endothelial injury and stasis
6)tumor with longest recurrence period is
malignant melanoma of uveal tract
7)tumor which s not apudomas
Gastrinoma
8)..common biological agent for carcinoma of bronchus
cmv
9)which drug dont cause gynecomastia
androgen
(If androgen could cause Gynecomastia then every man will have it)

-GENERALLY CAUSE OF PID------------- CHLAMYDIA >


GONORRHEA
-PURPURAL SEPSIS---------------------- (If group B strep is NOT there in
options then)
BACTERIOIEDS
-POST OP: INF------------------------ E.COLI
-ABDOMINAL SURGERY-------------- E.COLI
-MOST COMMON CAUSE OF PERITONITIS (Pyogenic or
Spontaneous) ---- E. Coli
-Net tubular reabsorption of sodium
Aldosterone and ANP
-Mole for mole Na excretion
Aldosteron
-Just Na reabsorption------ Aldosterone

10).oblique fissure
t3 to t6
11).gastric motuility increased by
Ach
12)Muscle forming inf relation wd hip joint
.obturator externus
13)Radiationz usualy cause cancer aftr
B. 5- 10yrz
14)LP shud b done at which level..
L3 -4 (BOTH L3/L4 or L4/L5 are correct but L3 is landmark, so select
the option with L3)
15)Brain abscess common in
Frontal > temporal (Answer: FRONTAL)
16)synapses are absent in
dorsal root ganglion
17)Sperms live in females body for
24-48 hrz (Controversy is there, Kaplan physio says 48-72 hours, but
Moore Parsaud says
sperm live upto 48 hours. The thing is Sperm live for 4-5 days but its
capability to fertilize
ovum is upto 48 hours. BOTTOM Line is: Select 24-48 hours but 48-72
hours option is also
correct)

-Just Na excretion-------- ANP


-growth hormone is max rise in----------- sleep
-Differnce between first & second heart sound is-------- Frequency (
pitch)
-In cell cycle chromosomes replicate in-------- Interphase
-Splenic infarction--------- CML
-Philedelphia chromosome seen in cell------- Granulocytes
(If question specify Cell Select Granulocytes, If question asks Site
Select Bone
Marrow)
-Tearing of pelvic diaphragm durng child birth will paralyse------ Levator
ani muscle
-Ca of rectum metastasize to ? Liver, duodenum, spleen , kidney?
ans. LIVER
-Extravaseted urine from penile rupture, spreads to---------- Scrotum
-On vaginal examination, structure palpable is -------- Ureters
-Broncheal cartilages derived frm----- mesoderm

18)which cells are involved in AIDs --------- Th2

-Broncheal epithelium and glands derived from----- endoderm

19)Wht is the energy source after 48 hrs of starvation??

-Large amount of dextrose water will inhibit--------- ADH

Fat (Triglyceride)

-Erythropoietin release frm----- peritubular capillaries

20)Most Commonly injured nerve in thyroidectmy--------Extrrnal laryngeal N

-Hormonal therapy in metastatic carcinoma, will cause.. Apoptosis


-withdrawal reflex mediated by---------- nociceptors

21)After lung abcess cause of pneumonia iz--------- Staph aureus


-Typical presentation of Diabetese------------ Wt Loss & Polyuria
-IUCD user female------------ ATINOMYCOSIS ISRAELI
-TUBOOVARIAN ABSCESS PID-------- GONORRHEA

-The diagnostic lab test for the infection by streptococuss beta


haemolyticus is ---------blood culture.

(Remember:
For Strep INFECTION Culture
For Strep infection COMPLICATIONS (Post-Strep GN or Rheumatic
Fever) ASO titer
-Pus contains------- Neutrophils
_Enzymes release from Neutrophils ( lysosomal enzymes).

5. Parasympathetic stimulation caused:


a. Bronchodilation
b. Decreased gut motiilty
c. Opens intestinal sphincters
d. Constipation
e. Tachycardia

- Negri bodies =-------- in Rabies


Ans: c
- Guarnieri bodies = -------in Small pox
- Henderson-Peterson bodies in--------- Molluscum
contagiosum
- Bollinger bodies = Fowl pox

6. Which of the following is the site of fusion of binocular vision?


a. Optic chiasma
b. Lateral geniculate bodies
c. Retina
d. Visual cortex
e. Optic nerve

- Prowazek Bodies = - Trachoma--Halberstaedter


- Cowdry type A = in Herpes simplex virus and
Varicella zoster virus and
CPSP BCQs with original key
DrKhalid Khan
1. Blood brain barrier is formed by:
a. Capillary endothelium
b. Continuous basement membrane
c. Pericytes
d. All of above
e. None of above

Ans: d
7. A newborn baby with hydrocephalus has a swelling in lumbosacral
spinal region, which contains neural tissue in it. What could be the
probable diagnosis of this patient?
a. Spina bifida
b. Meningocele
c. Meningomyelocele
d. Meningoencephalocele
e. Meningohydroencephalocele

Ans: c
8. Which of the following is most strong antioxidant?

Ans: a
2. Total power of the eye is 59 diopters. Main role is maintenance of
this power as performed by:

a. Glutathone
b. Vitamin - E
c. Vitamin - C
d. Ceruloplasmin
e. Catalase

a. Anterior surface of cornea


b. Lens
c. Vitreous Humor
d. Retina
e. Posterior surface of cornea

Ans: a

Ans: a
3. In patient with increased bleeding time, what could be the cause of
bleeding
a. Prothrombin Deficiency
b. Hemophilia A
c. Vitamin - K deficiency
d. Protein - C deficiency
e. Von Willebrand's disease
Ans: e
4. What is the nerve supply of tip of nose?
a. Ophthalmic nerve
b. maxillary nerve
c. Mandibular nerve
d. Fascial nerve
e. Cervical plexus
Ans: a

9. Most early diagnosis of Vitamin-A deficiency is:


(a) Bilot spots in cornea
(b) Night blindness
(c) Keratomalacia
(d) Chielosis
(e) Hyperkeratosis
Ans: b

10. A patient presents with recurrent epistaxis. Investigation receal


decreased factors II, VII, IX, X & protein-C which of the following is
most likely diagnosis of this condition?
(a) Christmas disease
(b) Hemophillia
(c) Vitamin-K deficiency
(d) Von willebrands disease
(e) Liver dysfunction

Ans: c
11. On H & E staining a student sees the hallow structure around the
nucleus. What it could be:
(a)Golgi apparatus
(b) Lysosomes
(c) Ribosomes
(d) Endoplasmic reticulum
(e) Mitochondria

Option (a) Infection


Option (b) Diabetes mellitus
Option (c) High mobility
Option (d) Foreign body in wound
Option (e) Vascular insufficiency

Ans: e
18. In a patient with sympathetic stimulation, what effect will occur on
his heat?

Ans: b
12. Bitemproral vision loss is most commonly caused by:
(a) Piuitary tumor
(b)Crainiopharyngioma
(c) Adenoma of hypothalamus
(d) Section of optic tract
(e) Section of optic nerve

Option (a) Decreased rate


Option (b) Coronary vasoconstriction
Option (c) Hyperpolarization of SA node
Option (d) Decreased AV nodal delay
Option (e) Decreased stroke volume

Ans: d

Ans: a

19. Which of the following is diagnostic of granuloma?

13. A patient has finger like projection on upper lid. Hostopathological


report shows epithelial cells along with fibrous element. What is most
probable diagnosis?
Option (a) Pappiloma
Option (b) Basal cell carcinoma
Option (c) Squamous cell carcinoma
Option (d) Chalazion
Option (e) Cellulites

Option (a) Giant cells


Option (b) Epithliod cells
Option (c) Multinucleated cells
Option (d) Fibroblasts
Option (e) Caseation

Ans: a
14. Parotid gland supplied by:
Option (a) GVA
Option (b) GVE
Option (c) SVE
Option (d) SVA
Option (e) GSA

Ans: b
20. Medial orbitotomey is done in a patient with tumor in orbit. Now he
is complaining of numbness at upper part of head up to vertex &
medial part but medial part is intact. Which of the following nerves is
damaged?
Option (a) Supraorbital nerve
Option (b) Supratrochlear nerve
Option (c) Superior orbital nerve
Option (d) Inferior orbital nerve
Option (e) Fascial nerve

Ans: b
15. Primary malignant melanoma of the choroids most commonly
metastasizes to:
Option (a) Liver
Option (b) Lungs
Option (c) Brain
Option (d) Kidneys
Option (e) Breast

Ans: a
21. In inferior wall Myocardial Infraction, which artery should be
blocked
Option (a) :Left marginal artery
Option (b) :Diagonal artery
Option (c) :Right coronary artery
Option (d) :Right marginal artery
Option (e) :Posterior Interventricular artery

Ans: a
16. A patient develops sensory loss over left side of body. After few
days his behavior for pain is changed & he become angry after
touching. Where could be the lesion in brain

Ans: d

Option (a) Right thalamus


Option (b) Internal capsule
Option (c) Cerebral cortex
Option (d) Basal ganglia
Option (e) Midbrain

Option (a) :Hypovolumic shock


Option (b) :Anemic Hypoxia
Option (c) :Hypoxic Hypoxia
Option (d) :Congestive Heart Failure
Option (e) :Acidosis

Ans: a

Ans: c

17. A middle aged male has non-healing wound in patient for long time
in presence of regular dressing random blood sugar is 130 mg/dl. What
could be the cause of delayed healing in this patient

22. Oxygen level in the blood will decrease in:

23. A male patient has antibodies against FSH RECEPTORS. Which


of the following will be lower than normal in this patient?

Option (a) :LDL cholesterol


Option (b) :Hemoglobin
Option (c) :Sperm count
Option (d) :Blood Sugar
Option (e) :Triglycerides

Ans: c

29. Which of following structures in the mouth is derived from


Ectoderm?
Option (a) :Epithelium of the tongue
Option (b) :Submandibular gland
Option (c) :Mylohyoid muscle
Option (d) :Epithelium of parotid gland
Option (e) :Tonsil

24. Brucellosis is Transmitted by


Ans: d
Option (a) :Files
Option (b) :Ras Fish
Option (c) :Unpasteurized Milk
Option (d) :Air Borne
Option (e) :Blood Transfusion

Ans: c
25. week pregnant women has Irregular Ulterine Contractions. Which
of following drugs will be beneficial for this lady.
Option (a) :Progesterone
Option (b) :Estrogen
Option (c) :Prolactin
Option (d) :Oxytocin
Option (e) :LH

Ans: d
26. The Diagnostic finding on investigations of Metabolic Alkalosis is:
Option (a) :HCO3 more then 24meq/L
Option (b) :CO2 more then 24meq/L
Option (c) :PH less than 7.4
Option (d) :Decreased O2 in blood
Option (e) :Increase serum carbonic anhydrase

Ans: a

30. A young patient's blood pressure is 150/95. his serum Rennin level
is higher than normal. Which of the following is the STIMULUS for this
increased level of rennin?
Option (a) :Essential hypertension
Option (b) :Increased delivery of sodium to Renal tubules
Option (c) :Increased sympathetic stimulation via Renal nerves
Option (d) :Vasodilatation of Renal arterioles
Option (e) :Increased blood flow to the kidneys

Ans: c
31, A young women, complains of Dry mouth & Dry eyes. Which
investigation will give clue to her diagnosis
Option (a) :ANA
Option (b) :Anti-ANA
Option (c) :RA factor
Option (d) :Anti - SS A/B
Option (e) :ANCA

Ans: d
32. Regarding SA Node:
Option (a) :Its resting membrane potential is - 65 to -85 volts
Option (b) :It automatically generates impulses, creating rhythmic
heartbeat
Option (c) :Its membrane is impermeable to Na ions
Option (d) :It is supplied by left coronary artery
Option (e) :It lies in the septum

Ans: b
27. A 6 cm Lump in the breast removed. Four Lymp nodes & skin were
involved. Biopsy declared invasive Dectal Carcinoma. This tumor is
called HIGH-GRADE because:
Option (a) :Lymph node involvement
Option (b) :Skin involvement
Option (c) :Size of tumor
Option (d) :Pleomorphism
Option (e) :Duct involvement

33. During the stretch in the Skeletal Muscle, which of the following
changes will occur in the Nuclear Bag fiber?
Option (a) :They increase impulse generation
Option (b) :They remain static
Option (c) :They decrease impulse generation
Option (d) :They increase oscillation
Option (e) :They inhibit muscle contraction

Ans: a
Ans: d
28. Which of the following is the characteristic of lgM?

34. which of the following structures is produced in 3rd week of


development?

Option (a) :It provides mucosal barrier


Option (b) :It is in highest concentration in serum
Option (c) :It is smallest molecule
Option (d) :It is largest Molecule
Option (e) :It crosses the placenta

Option (a) :Thyroid gland


Option (b) :Parathyroid gland
Option (c) :Genital ridge
Option (d) :Heart tube
Option (e) Tonsil

Ans: d

Ans: d

41. Patients comes with deviation of tongue to right side. Decreased


sense of touch and vibrations, the artery commonly involved in brain is:

35. Antibodies are produced by:


Option (a) :Lymphocytes
Option (b) Plasma cells
Option (c) T-Cells
Option (d) :Neutrophils
Option (e) Endothelium

Ans: b
36. Pulmonary Artery pressure increases in:
Option (a) :Exercise
Option (b) :Hypoxia
Option (c) :Anemia
Option (d) :Hypovolumic Shock
Option (e) :Hypertension

Option (a) :PICA


Option (b) :AICA
Option (c) :Anterior Spinal
Option (d) :Posterior Cerebral
Option (e) :Superior Cerebral Artery

Ans: c
42. Most common site of malignancy in patients suffering from nuclear
outbreak
Option (a) :Haematopoietic
Option (b) :Thyroid
Option (c) :Lung
Option (d) :Breast
Option (e) :Bones

Ans: a
Ans: b
37. Defect in the formation of Bulbus Cordis result in all of following
EXCEPT?
Option (a) :ASD
Option (b) :VSD
Option (c) :Hypertrophy of right ventricle
Option (d) :Congenital cyanosis
Option (e) :Transposition of great vessels

43. Least common site for ectopic pregnancy would be at:


Option (a) :Ovaries
Option (b) :Pouch of douglus
Option (c) :Greater omentum
Option (d) :Fallopian tubes
Option (e) :Cervix

Ans: a
44. Most sensitive cells to hypoxia are

Ans: b
38. Regarding the vibration Sense all are correct EXCEPT:
Option (a) :It is lost in Diabetes
Option (b) :Its fibers are traveling in the dorsal columns
Option (c) :Its receptor is Pacinian Corpuscle
Option (d) :Its receptor is Meissener's corpuscle
Option (e) :Is highly correspond to the bony portions

Option (a) :RBCs


Option (b) :Neurons
Option (c) :Nephrons
Option (d) :WBCs
Option (e) :Platelets

Ans: b
45. Following is not a tumor marker:

Ans: d
39. A patient with aplastic anemla is given Anti Lymphocytic Globulin
(ALG). One week later he develops Skin rashes, mechanism for
presentation?
Option (a) :Arthus reaction
Option (b) :Type - I Hypersensitivity
Option (c) :Type - II Hypersensitivity
Option (d) :Type - III Hypersensitivity
Option (e) :Type - IV Hypersensitivity

Ans: d
40. The most common site of Fertilization in humans is:
Option (a) :Ovary
Option (b) :Uterus
Option (c) :Fallopian tube
Option (d) :Cervix
Option (e) :Peritoneal cavity

Ans: c

Option (a) :PALP


Option (b) :CEA
Option (c) :bHCG
Option (d) :AFP
Option (e) :Acid Phospatase

Ans: e
46. tyrosine derivative does not include:
Option (a) :TSH
Option (b) :Adrenaline
Option (c) :Nor adrnaline
Option (d) :Prolactin
Option (e) :Dopamine

Ans: d
47. Organ having least chances of infarction:
Option (a) :Lungs
Option (b) :Heart
Option (c) :Kidneys
Option (d) :Liver

Option (e) :Spleen

Ans: d

Option (a) :FAP


Option (b) :Villous adenoma
Option (c) :Tubular adenoma
Option (d) :Tubulovillous
Option (e) :Metaplastic polyp

48. Least chances of renal stones is associated with:


Option (a) :Hyperlipidemia
Option (b) :Hyper PTH
Option (c) :Hyper vit.D
Option (d) :Infections
Option (e) :Hyperurecemia

Ans: a
49. Patient with injury to left 8th cervical segment of spinal cord will not
show following sign:
Option (a) :Decreased sense of position
Option (b) :Vibration below lesion on same side
Option (c) :Extensor plantar on left side
Option (d) :Dec power of muscles below the lesion on same side
Option (e) :Dec sense of pain and temperature below the lesion on
same side

Ans: e
50. Patient with bone pains having normal Ca, inc Alkaline
phosphatase. Most likely suffering from:
Option (a) :Pagets disease
Option (b) :Hyper PTH
Option (c) :Hyper Vit D,
Option (d) :Bone mets
Option (e) :Osteomalacia

Ans: a
51. Bile salts are reabsorbed from the:

Ans: c
55. Man in suffering from testicular Carcinoma, the lympahtic drainage
of testicle is into
Option (a) :Para aortic Lymph nodes
Option (b) :Pre aortic
Option (c) Superficial inguinal
Option (d) :Internal iliac
Option (e) :External iliac Lymph nodes

Ans: a
56. Mesothelioma is associated with:
Option (a) :Vinyl chloride
Option (b) :Silica
Option (c) :Asbestos
Option (d) :Copper dust
Option (e) :Carbon

Ans: c
57. Foetal period starts after which week:
Option (a) :11th
Option (b) :8th
Option (c) :12th
Option (d) :16th
Option (e) :21st

Ans: b

Option (a) :Duodenum


Option (b) :Jejunum
Option (c) :Ileum
Option (d) Colon
Option (e) Rectum

Ans: c
52. In Turner syndrome, the genotype would be:
Option (a) :XX
Option (b) :XO
Option (c) :XXY
Option (d) :XY
Option (e) :XYY

58. Soldier comes with heavy bleeding. The ideal fluid replacement
would be:
Option (a) :Packed RBCs
Option (b) :Crystallines
Option (c) :Colloids
Option (d) :Whole blood for 3 days
Option (e) :Whole blood for 18 days

Ans: b
Ans: d
53. Presence of pancreatic tissue in gastric mucosa is termed as:
Option (a) :Hamartoma
Option (b) :Metaplasia
Option (c) :Neoplasia
Option (d) :Choriostoma
Option (e) :Dysplasia

Ans: d
54. Patient with old history of adenocarcinoma of colon operated for
polypectomy, on histologic evaluation pathologist labelled it as benign
growth with no chances into malignant transformation, it would be:

59. Vertebrae is derived from :


Option (a) :Myotome
Option (b) :Sclerotome
Option (c) :Dermatome
Option (d) :Ectoderm
Option (e) :Endoderm

Ans: b
60. Largest total cross-sectional and surface area is of:

Option (a) :Artery


Option (b) :Arterioles
Option (c) :Capillaries
Option (d) :Venules
Option (e) :Veins

67. Somatostatin dec sec of which hormone


Option (a) :ADH
Option (b) :Insulin
Option (c) :Oxytocin
Option (d) :Prolactin
Option (e) :Thyroid hormone

Ans: c
61. Esophagus histology

Ans: b

Option (a) :Covered by squamous ep


Option (b) :Has str. sq in upper 1/3
Option (c) :Has str. sq in lower 1/3
Option (d) :Has 3 layers of muscles
Option (e) :Esophageal opening acting as anatomical sphincter

68. Digoxin toxicity


Option (a) :Inc K
Option (b) :Inc Mg
Option (c) :Dec Ca
Option (d) :Dec Na
Option (e) :Alkalosis

Ans: b
62. Structures entering thorasic inlet
Option (a) :Accessory nerve
Option (b) :Aorta
Option (c) :Superior vena cava
Option (d) :Azygous Vein
Option (e) :Left recurrent laryngeal nerve

Ans: b

Ans: e
69. Following is true regarding sterilization:
Option (a) :Repeated heating denatures polyvencyl tubings
Option (b) :Autoclaving is heating objects at 121 degC at 15 psi for
3min
Option (c) :Radiation (UV light)
Option (d) :Dry heat
Option (e) :Formaldehyde

63. Example of carrier mediated counter transport:


Option (a) Na - glucose transport
Option (b) :Active transport
Option (c) :Passive transport
Option (d) :H transport
Option (e) :Diffusion

Ans: a

Ans: b
70. Pt complains of chest pain for more than 30 min. ECG shows
changes in V1-V4. It denotes:
Option (a) :Anterior wall MI
Option (b) :Anterolateral MI
Option (c) :Inferior wall MI
Option (d) :Lateral wall MI
Option (e) :Inferior wall MI

64. Alpha recptors effects


Option (a) :Inc HR
Option (b) :Lipogenesis
Option (c) :Midriasis
Option (d) :Piloerector contraction
Option (e) :Bronchodilator

Ans: c

Ans: a
71. Depolarization of cell is maintained by:
Option (a) :Na influx
Option (b) :Ca efflux
Option (c) :Ca influx
Option (d) :K influx
Option (e) :Na efflux

65. Hypoglycemia increases


Ans: a
Option (a) :SSK
Option (b) :Somatostatin
Option (c) :Gastrin
Option (d) :VIP
Option (e) :Secretin

72. Spinal cord ends at the level of lower border of:


Option (a) :L2 vertebra
Option (b) :L3 vertebra
Option (c) :L4 vertebra
Option (d) :L5 vertebra
Option (e) :S1 vertebra

Ans: e
66. Cushing's triad:

Ans: a

Option (a) :Inc ICP, HTN, Bradycardia


Option (b) :Inc ICP, hypoTN, Bradycardia
Option (c) :Inc ICP, hypoTN, Tachycardia
Option (d) :Inc ICP, HTN, Tachycardia
Option (e) :Dec ICP, HTN, Tachycardia

73. Hyperoxia. All true except 1.

Ans: a

Option (a) :Retrolental fibroplasia


Option (b) :Atelectasis
Option (c) Dec surfactant prod
Option (d) :CV depression
Option (e) :Anemia

Ans: e
74. Pacinian corpuscles related to
Option (a) :Touch
Option (b) :Vibration
Option (c) :Pressure
Option (d) :Rapidly adapting pain
Option (e) :Slowly adapting pain

Ans: b
75. Drug that inc extracellular K (moves K out of the cell):
Option (a) :Angiotensin
Option (b) H2CO3
Option (c) :Carbonic anhydrase
Option (d) :pH
Option (e) :Exercise

Option (c) :Mucor


Option (d) :Aspergillus
Option (e) :Blastomycosis

Ans: c
81. A patient has non-reactive HBs Ag, non reactive HBe Ag & reactive
anti HBc Ab. This stages of patient indication what?
Option (a) :Acute infection
Option (b) :Carrier
Option (c) :Transient resolving stage of hepatitis
Option (d) :Immunized
Option (e) :Chronic active disease

Ans: c

Ans: a
76. Thirst is decreased by:
Option (a) :ADH
Option (b) :Aldosterone
Option (c) :Ag II
Option (d) :Baroreceptor efferent
Option (e) :Inc Hematocrit

Ans: d
77. Muscle of quiet inspiration
Option (a) :Diaphragm
Option (b) :Rectus abdominis
Option (c) :Internal intercostals
Option (d) :innermost intercostals
Option (e) :External intercostals

Ans: a
78. Dry mouth increases all except
Option (a) :Thrist
Option (b) :ADH
Option (c) :Ag II
Option (d) :Plasma osmolarity
Option (e) :Plasma vol

82. A Patient has enlaged parotid gland with pain in this region. Which
nerve is carrying pain fibers from the parotid gland?
Option (a) :Auricuolotemporal nerve
Option (b) :Glossopharyngeal nerve
Option (c) :Fascial Nerve
Option (d) :Temporal nerve
Option (e) :Greater auricular nurve

Ans: a
83. A Patient has anemia, hyper-segmented Neutrophils on peripheral
blood examination & neurological manifestations. Which type of
anemia he is suffering from?
Option (a) :Folic acid deficiency anemia
Option (b) :Iron deficiency anemia
Option (c) :Pernicious anemia
Option (d) :Thalasemia
Option (e) :Autoimmune hemolytic anemia

Ans: c
84. A young child has increased BP in upper limbs while decreased BP
in lower limbs. Pulse in also week. What is the probable diagnosis?
Option (a) :Preductal coarctation of aorta
Option (b) :V.S.D
Option (c) :Patent ductus arteriosus
Option (d) :Postductal coarctation of aorta
Option (e) :Transposition of great vessels

Ans: e
79. Gastric acid is stimulated by:
Option (a) :Gastrin
Option (b) :CCK
Option (c) :Secterin
Option (d) :GIP
Option (e) :VIP

Ans: d
85. In case of typhoid fever of 06 days. Which investigation is of first
choice?
Option (a) :Widal test
Option (b) :Blood culture
Option (c) :Typhi dot test
Option (d) :Bone marrow culture
Option (e) :Urine culture

Ans: a
80. Which of the following fungi produce life threatening infection in
patients with diabetics ketoacidosis?
Option (a) :Candida Albicans
Option (b) :Histoplasmosis

Ans: b
86. In a patient with diabetes, which is the best antihypertensive drug
Option (a) :Captopril
Option (b) :Diuretic
Option (c) :Varapamil
Option (d) :Propranolol

Option (e) :Diltiazim

Option (c) :Gallium Scan


Option (d) :CPK
Option (e) :Ultrasound

Ans: a
87. Urine Examination of a patient with diabetes demonstrates.
Ketone-bodies. What is the mechanism of formation of these ketonebodies?
Option (a) :Insulin deficiency
Option (b) :Hyperglycemia
Option (c) :Defective fat metabolism
Option (d) :Hypoalbubinemia
Option (e) :Defective glucose metabolism

Ans: a
88. After giving blood transfusion, a patient develops hypersensitivity
reaction. Which type of hypersensitivity reaction is this?
Option (a) :Type I Hypersensitivity
Option (b) :Type II Hypersensitivity
Option (c) :Type III Hypersensitivity
Option (d) :Type IV Hypersensitivity
Option (e) :A.D.C.C

Ans: c
93. In a patient diarrhea are corrected after the fasting. Which is most
likely type of diarrhea?
Option (a) :Infectious
Option (b) :Psychogenic
Option (c) :Osmotic
Option (d) :Metabolic
Option (e) :Toxic

Ans: c
94. A diabetic Patient is advised fasting blood glucose level. At what
value doctor is confusing & needs further evaluation with glucose
tolerance test?
Option (a) :04 m mol / 1
Option (b) :05 m mol / 1
Option (c) :07 m mol / 1
Option (d) :10 m mol / 1
Option (e) :15 m mol / 1

Ans: b
89. A Patient develps lesion in the Caudate nucleus. Which is most
common clinical feature Indicating this lesion?
Option (a) :Chorea
Option (b) :Intentional Tremor
Option (c) :Resting Tremor
Option (d) :Hemiplagia
Option (e) :Nystygmus

Ans: a

Ans: c

95. A patient has Aphasia & Facial nerve palsy on same side. Which
artery is most likely blocked?
Option (a) :Anterior cerebral artery
Option (b) :Posterior cerebral artery
Option (c) :Middle cerebral artery
Option (d) :Anterior division of middle cerebral artery
Option (e) :Posterior division of middle cerebral artery

90. Heme Binds with


Option (a) :Albumin
Option (b) :Globulin
Option (c) :Hemopexin
Option (d) :Heptoglobin
Option (e) :Macroglobulin

Ans: d

91. Best pulmonary function test for the asthma is:


Option (a) :FEV - 1
Option (b) :Residual volume
Option (c) :Tidal volume
Option (d) :Vital capacity
Option (e) :Inspiratory capacity

Ans: a
92. A pregnant lady develops pain few hours after delivery. E.C.G.
shows S1, Q3 and T3. What is another investigation most appropriate
for the diagnosis of this case?
Option (a) :RF
Option (b) :X-Ray

Ans: c
96. A young female is having Goiter. She feels difficulty in breathing
while lying down. Which type of the goiter she is having?
Option (a) :Retrosternal goiter
Option (b) :Malignant goiter
Option (c) :Toxic goiter
Option (d) :Diffuse goiter
Option (e) :Simple goiter

Ans: a
97. Regarding L & D sugars
Option (a) :D sugar has - OH on right
Option (b) :L sugar has - OH on right
Option (c) :D sugar has - OH on left
Option (d) :D sugar has no - OH
Option (e) :L sugar has no OH

Ans: a
98. Councilman bodies are formed in the process of
Option (a) :Infection
Option (b) :Apoptosis

Option (c) :Trauma


Option (d) :Necrosis
Option (e) :Atrophy

Ans: b

104. Prenatal chromosomal abnormality can be detected at:


Option (a) :Booking
Option (b) :10-12 weeks
Option (c) :14-18 weeks
Option (d) :After 20 weeks
Option (e) :In second trimester

Ans: c
99. A middle aged male presents with joint pain since one week. His
serum Uric Acid level is 156 mg/dl. What is the best investigation for
confirmation of the disases?
Option (a) :R.F
Option (b) :Synovial fluid for polarized light
Option (c) :Synovial fluid for Culture & Sensitivity
Option (d) :X-Ray of joint.
Option (e) :E.S.R

Ans: b
100. A patient comes to you in ill condition. His blood pH is 7.3 PCO2
is 44 mmHg & HCO3 is 17 meq/L. What diagnosis you are thinking of
this patient?
Option (a) :Metabolic alkalosis
Option (b) :Metabolic acidosis
Option (c) :Respiratory acidosis
Option (d) :Compensatory acidosis
Option (e) :Compensatory alkalosis

Ans: b
101. During general anaesthesia Halothane is given in combination
with which of the following drugs?
Option (a) :Thiopentone
Option (b) :Nitric oxide
Option (c) :Enfluran
Option (d) :Phenobarbital
Option (e) Ketamine

Ans: b
102. Most common cause of the pelvic inflammatory disease is?
Option (a) :Gonococcus
Option (b) :Chlamydia
Option (c) :Anaerobes
Option (d) :Candida albicans
Option (e) :E-coli

105. Secondary Hyperaldosteronism can occur due to:


Option (a) :Increased rennin
Option (b) :Decreased rennin
Option (c) :Increased potassium
Option (d) :Decreased potassium
Option (e) :Hypertension
Ans: a
106All of the following mechanisms are involved in development of
diabetic gangrene EXCEPT?
Option (a) :Trophic changes due to peripheral neuritis
Option (b) :Arterioscierosis leading to reduced blood flow to foot
Option (c) :Decreased neutrophil motility
Option (d) :Excess sugar in tissues leads to reduced resistance to
infection especially fingal infection
Option (e) :Anaerobic infection is commonly encountered in this
disease
Ans: b

107. A patient presents with dislocation of hip after an accident. Which


of the following muscle group is involved?

Option (a) :Gluteus Maximus + Minimus + Medius


Option (b) :Only Gluteus medius
Option (c) :Only Gluteus Maximus
Option (d) :Gluteus Maximus + Medius
Option (e) :Only Gluteus Minimus

Ans: a
108. A patient has history of recurrent abortion. Which of the following
enzyme analysis will give help to diagnosis?
Option (a) :Estrogen
Option (b) :Progesterone
Option (c) :L.H & F.S.H
Option (d) :H.C.G
Option (e) :Testosterone

Ans: b
Ans: b
103. A lady received DES during pregnancy for prevention of the
abortion. Her baby will be on the risk of:

109. A patient presents with secondary amenorrhea. One year back


she has history of P.P.H for that she received six pints of blood what is
most probable caused of her menstrual problem?

Option (a) :Small cell of carcinoma


Option (b) :Squamous cell carcinoma
Option (c) :Carcinoma of endometrium
Option (d) :Clear cell carcinoma of Vagina
Option (e) :Uterine fibroid

Option (a) :Pituitary Tumor


Option (b) :Sheehan's syndrome
Option (c) :Addison's disease
Option (d) :Adrenal cortex adenoma
Option (e) :Psychological problem
Ans: b

Ans: d

110. A patient presents with recurrent Rhinitis with Urticaria & Rashes.
Which of the following is most probably diagnosis?

Option (d) :inferior vena cava


Option (e) :inferior mesenteric artery
Option (a) :Type - I Hypersensivity
Option (b) :Type - II Hypersensivity
Option (c) :Viral Infection
Option (d) :Arthus Reaction
Option (e) :Cell mediated reaction

Ans: e
117. Which of the following is characteristic of polycystic ovary
disease:

Ans: a
111. A 60 years old women is brought with bleeding per vagina. She is
diabetic & hypertensive & one year back she was diagnosed as having
carcinoma of breast. What can be most probable diagnosis for this
bleeding p/v?

Option (a) :Increased L.H


Option (b) :Increased F.S.H
Option (c) :Decreased L.H
Option (d) :Decreased F.S.H
Option (e) :No change in hormones
Ans: a

Option (a) :Carcinoma of colon


Option (b) :Carcinoma of endometrium
Option (c) :Endometrial Hyperplasia
Option (d) :Carcinoma of cervix
Option (e) :Cervical erosion
Ans: b
112. A Patient is suffering from carcinoma of rectum. He develops pain
in the posterior aspect of thing due to involvement of nerve. Which
nerve supplies the posterior aspect of thigh & that can be involved in
the malignancy of rectum?

118. A 20 - Week pregnant lady has blood pressure 140/95. She


complains of vomiting & headache. What is the probable diagnosis of
the condition?
Option (a) :Appendicitis
Option (b) :Pregnancy induced hypertension
Option (c) :Oligohydramnios
Option (d) :Raised intracranialpressure
Option (e) :Pregnancy induced DM
Ans: b
119. Which cranial nerves carry parasympathetic fibers?

Option (a) :Lumbosacral trunk


Option (b) :Sacral nerves
Option (c) :Obturator nerve
Option (d) :Superior hypo gastric plexus
Option (e) :Inferior hypo gastric plexus

Option (a) :Third, Fourth, Ninth, Tenth


Option (b) :Third, Fourth, Fifth, Ninth
Option (c) :Second, Fourth, Fifth, Ninth
Option (d) :Third, Seventh, Ninth, Tenth
Option (e) :Fifth, Sixth, Tenth, Eleventh

Ans: b
113. A semiconscious patient is brought to emergency department. He
has history of taking some unknown drug. NaHCO2 reverses the
action of drug. Which drug he has most likely taken?
Option (a) :Phenobarbital
Option (b) :Phenothiazine
Option (c) :Morphine
Option (d) :Diazepam
Option (e) :Alcohal
Ans: a
114. A lady with 26 weeks gestation has uterine height of 24 weeks.
On sonography there is no fetus & snowstorm appearance. Which of
the following probably associate with it?
Option (a) :Choriocarcinoma
Option (b) :Karyotype-46XX
Option (c) :47XX
Option (d) :69XXX
Option (e) :45XO

Ans: b
115. Normal female pelvis has:
Option (a) :Deeper inlet
Option (b) :Inturned spine
Option (c) :Narrow outlet
Option (d) :Round outlet
Option (e) :Oval inlet
Ans: e
116. The ascent of horse shoe shaped kidney is prevented by:
Option (a) :External iliac artery
Option (b) :Internal iliac artery
Option (c) :Superior mesenteric artery

Ans: d

****Clear Handy Points******

Trisomy 21 = Down syndrome. (Age for Drinking is 21)

Trisomy 13 = Patau syndrome.(Age for Puberty in females is


13)

Trisomy 18 = Edward syndrome.(Age for voting in Election is


18)

Increased alpha feto protein plus increased albumin =


Anencephaly.

Vitamin A prevents squamous cell carcinoma.

Right border of heart is made by right atrium.

Trephine biopsy Indicates = aplastic anemia > ALL.

In anesthesia halothane is always given with Nitric oxide.

Most diagnostic test for TB = PCR > AFB > Caseous.

Down syndrome occurs 1/100.

Common genital vesicle is herpes simplex virus.

Inferior orbital fissure contents are maxillary nerve and its


zygomatic branch, inferior ophthalmic vein and sympathetic
nerves and this fissure communicates with pterygo palatine
fossa.

Superior orbital fissure communicates with middle cranial


fossa and it transmits lacrimal nerve, frontal nerve, trochlear
nerve, oculomotor nerve, abducent nerve, nasociliary nerve
and superior ophthalmic vein.

Thyroid is the only endocrine gland that stores its secretions


outside the cell.

Sensations from the tip of nose are carried by ophthalmic


division of trigeminal nerve.

Tensor tympani is supplied by mandibular division of


trigeminal nerve.

Stapedius is supplied by facial nerve.


Nucleus raphe synthesize serotonin.

Locus Ceruleus = Nor epinephrine.

At term CRL = 36 cm and CHL = 50 cm.

CRL used b/w 7 - 14 weeks.

BPD is used 16 - 30 weeks.

Sperm life in genital tract is 24 to 48 hours.


2nd most common cause of osteoporosis in old age is
Cushing syndrome.
Prenatal chromosome is detected at 14 -18 weeks.
Bronchial asthma plus hypertensive patient > Best drug
Verapamil.
Aphasia and facial nerve palsy > damage to middle
meningeal artery.
Diabetic plus hypertensive patient > Best drug Captopril.
Inferior wall MI > Right marginal artery block.
Epidermis of partoid gland is derived from ectoderm.
To kill spores of surgical instrument > moist heat at 160 C for
1 hr.
Food poisoning caused by Staph. Aureus is through
entertoxin.
Least positive value for Widal is 1:120.
Trigeminal ganglion is completely covered by dura.
Tactile sensation is carried by dorsal white column to medial
leminiscus.
Anti HBcAg = positive window period.
MAP = diastolic +1/3 pulse pressure.
Insulin secretion is inhibited by beta blocker.
Highest triglyceride = VLDL.
Highest cholesterol = LDL.
Highest lipoprotein = HDL.
Which is not a phospholipid = Plasmalogen.
Best way to check bone density is the scan of spine.
End break down of glucose is pyruvate.
In pre eclamptic patient hydralazine is the drug of choice
before surgery.
In down syndrome: Triple test (alpha feto protein is
decreased, b-HCG is increased and estriol is decreased)
and if we add up inhibin which is increased then it will be
called as Quadruple test.
Identification of turner syndrome is by barr bodies.
PLAP (Placental Alkaline Phosphatase) is a tumor marker in
seminoma and ovarian carcinoma.
Tubo ovarian abscess by IUCD - most causative agent is
Actinomycosis.
Major intracellular buffer is Hb.
DVT more common in popliteal vein but pulmonary
embolism is through femoral veins.
Investigation of DIC D-dimers, FDPs, Platelet count and PT
(except clotting time).
Referred pain: Cervix S2-S3 <> Ovary -T10-T11 <> Testis T10 <> Umbilicus -T10 <> Kidney T12-L2.
Trimethoprim (co-trimoxazole ) side effect > megaloblastic
anemia plus leukopenia.
Sacrospinous ligament does NOT contribute in wall of
perineum.
Lesser omentum connects with duodenum.
Sphincteric urethra is known as external urethral sphincter
and is supplied by pudendal nerves where as internal os is
supplied by inferior hypo gastric plexus.
Nerve supply of rectum is hypogastric plexus.
Uterine tube is 10 cm long.
Urachal cyst is the remanant of allantois.
Thyroid gland is derived from endoderm.
Thymus and inferior parathyroid develops from 3rd branchial
pouch.
Superior para thyroid is developed from 4th brachial pouch.
Diaphragmatic hernia occurs due to absence of pleuro
peritoneal membrane.
Superior thyroid artery is related to external laryngeal nerve
which supplies posterior cricothyroid muscle of larynx and
damage to it will cause hoarseness.
Inferior thyroid artery is also related to recurrent laryngeal
nerve which runs b/w trachea and esophagus and its most
common injury is in thyroidectomy.
In papillary carcinoma of thyroid > it occurs in young age and
involves cervical lymph nodes.
Tracheostomy is done at 2nd tracheal ring by pulling the
isthmus inferiorly.
Nonfunctional nodule or cold nodule has high chances of
malignant transformation.
Erythropoiesis in middle trimester is in the liver.
Right coronary artery supplies SA node.
Levator ani muscle is supplied by L2, L3 & L4.

Defect in Bulbus Cordis results in VSD, hypertrophy of Right


ventricle, congenital cyanosis, transposition of great vessels
(but not ASD).
Endocardial cushion is important for the formation of four
chambers of heart.
Thirst is least stimulated by blood pressure.
Important hormone involved in gluconeogenesis is Cortisol.
Apoptosis is inhibited by bcl-2 inhibition.
Low serum complement in SLE.
C3b & IgG are Opsonins & C5a is chemotactic protein.
Urea is an important indicator for muscle protein loss.
ESR is decreased when albumin is increased.
Drug which does not cause gynecomastia is Androgen &
drugs which cause gynecomastia are Digoxin, Girsoefulvin,
Cimetidine, Androgens, Spironolactone and Ketoconazole.
Tx of hirstuism is Cyproterone Acetate.
Pyruvic acid is intermediate from glucose to acetyl coA.
Epinephrine .nor epinephrine & dopamine are derived from
tyrosine.
End product of Purine is Uric acid.
RBCs have glycolytic enzyme activity.
End product of glucose gives 2 Pyruvate.
Alanine should be taken in diet.
Cisplatin is more notorious to cause renal toxicity.
Dysplasia is seen in epithelia.
MRNA has a codon.
HSV is associated with vulvar papules.
Wart on the lateral wall of introitus it is caused by HPV.
Plaque like lesion on posterior superior wall of vagina is
squamous cell carcinoma.
German Measles causes Congenital Cataract.
Anterior abdominal wall swelling with umbilical cord attached
to it in a new born baby is known as Omphalocele.
Beta-lactam acts on the Cell Wall.
Streptomycin causes ototoxity.
Isoniazid > Hepatotoxicity.
Pyrazinamide > Gout.
Rifampicin > red color of body secretions.
Opportunistic organism > E. Coli.
Pseudomembranous colitis is caused by C. difficile.
Most common organism involved in gynecological &
abdominal procedures is E.coli.
Vulvular itching = Chlamydia; fish like smell; Bacterial
Vaginosis.
Vitamin K dependent: Factors 2, 7,9,10, Protein C, Protein
S, Fibrinogen and Prothrombin.
Intrinsic and Extrinsic pathways of coagulation converge at
factor 10.
Threonine does not contain Sulfa group.
Autosomal dominant is hereditary Spherocytosis& Poly
cystic kidney disease.
Lens opacity causing drugs>Chlorpromazine, Amidarone,
Tamoxifen, Gold & Iron toxicity.
Drugs causing corneal opacity > Amiodarone, Chloroquine,
Mepacrine & Copper.
Ribosome have purple color on Eosin & Methylene blue
staining
High energy content > Starch.
High energy compound > ATP
Antidote of warfarin is vitamin K but if action is more quickly
required then FFP.
Olfactory cells are the only neurons in the body that
regenerates.
Projectile vomiting greenish in color means bilious vomiting
so it is due to duodenal atresia but if projectile vomiting non
bilious then it is hypertrophic pyloric stenosis.
Pulmonary trunk relation with the bronchus at the hilum of
the lung-mnemonic is RALSR- Right Anterior & Left
Superior.
Rhino sinusitis is caused by Strep Pneumonia, H. Influenza,
M Catarrahalis.
Homans sign is present in DVT in which if you dorsiflex the
foot there will be pain in calf muscles.
Classic triad of Pulmonary Embolism:
-Neurological manifestations.
-Petechial rash.
-Hypoxemia.

Nitrogen bubbled precipitator in ascending divers and can be


treated with hyperbaric oxygen.

CT pulmonary angiography is the best test to detect


Pulmonary Embolism.

The most common infectious agent transmitted by blood


transfusion is cytomegalovirus (CMV), which is present in
donor lymphocytes.

Before blood is transfused into newborns or patients with Tcell deficiencies, it must be irradiated to kill donor
lymphocytes. This prevents the patient from developing a
graft-versus-host reaction or a CMV infection.

Yersinia enterocolitica, a pathogen that thrives on iron, is the


most common contaminant of stored blood.

Iron is stored in bone macrophages.

Structures passing thru superior orbital fissure....


NOT-FAL
NASOCILLIARY<>OPTHALIMIC
VEIN<>TROCHLEAR<>FRONTAL
ABDUCENT<>LACRIMAL

Cranial Nerve Foramina


CN I passes through > cribriform plate
CN II > optic canal
CN III, IV, VI (ophthalmic division), VI > superior orbital fissure
(Maxillary div > foramen rotundum, mandibular div > foramen
ovale)
CN VII, VIII > internal acoustic meatus
CN IX, X, XI(cranial root) > Jugular foramen (accessory nerve
spinal root >foramen magnum)
CN XII > hypoglossal canal.
Gynea june 2014
1. A Female during develivery looses blood and requires blood. Her
blood sample is reactive
blood agglutinates with anti sera B and anti sera D. which blood is
suitable for transfusion.
a. A+
b. B RH Positive
c. O +
d. AB +
2. Following arches over left lung root.
a. Thoracic ductb. Arch of aortac. Azygus vein
3. Patient IVC is blocked just above the emergence of azygus vein,
where blood will divert.
a. Right gastric arteryb. Hepatic veinc. Hepatic arteryd. Umbilical
arterye. Splenic artery
4. Internal Jugular vein is related
a. Anterior to Carotid arteryb. Medially to Carotid Arteryc. Laterally to
Carotid arteryd. Posteriorly to Carotid arterye. Internal Carotid Artery
posteriorly
5. Rapid adaptation receptors
a. Free nerve endingsb. Meckels diskc. Pacinian corpusclesd. Ruffinis
corpuscles
6. Hypermeagnesemia causes
a. Hyperreflexiab. Dec acetyl choloine uptake
7. Best example against single organ?
a. Hashimoto thyroiditisb. PANc. SLEd. RA
8. Hormone required for brain development
a. Thyroid hormoneb. Growth hormonec. Cortisold. Androgene. Insulin
9. Ketamine used in repeated burn dressings because
a. It does not cause hypotention
b. Causes profound analgesia
c. It can be given IV
d. It does not causes addiction
e. Its not narcotic
10. Hanging causes death by
a. Fracture of odontoid process
Disclaimer: questions are incomplete and MAY NOT INCLUDE
CoRRECT ANSWERS
b. Fracture of atlas
11. Mecanisim of action of propylthiouracil
a. Inhibition Iodine pump
b. Blocks the synthesis of thyroid hormone.
c. Blocks the synthesis of thyroglobulin.
d. Inhibits hormone release
e. Decreases TSH
12. Last thing to be returned normal after haemorrhagic shock
a. Heart rate

b. Blood pressure
c. Cardiac contractility
d. RBC in peripheral smear
e. Pulse pressure
13. Antibodies are produced by
a. Lymphocytes
b. Plasma cells
c. T-Cells
d. Neutrophils
e. Endothelium
14. 15 year old male had history of right iliac fossa pain for 18 hours
and was operated for
appendicitis, histopathology will show which cell
a. Lymphocytes
b. Macrophages
c. Neutrophils
15. Which of congenital malformation is most common
a. ASD
b. VSD
c. TS
16. Male with transmural infarction died suddenly in washroom after 8
days.
a. Atrial fibrillation
b. Ventricular Fibrilation
c. Cardiac Temponade
d. Pulmonary embolism
17. After trauma Female with sudden acute chest pain, Dyspnoea,
hemoptysis and shock and
then death.
a. Fat embolism
b. Air Embolism
c. Peunomionia
18. G1P2 female after delivery develops sudden chest pain and
difficulty in breathing and
hemoptysis, ECG shows S1 Q3 T3 pattern which is most appropriate
test.
a. LDH
b. SGO?PT
c. X-Ray
d. CPK
19. In complete Glucose metabolism number of ATP
a. 30
b. 32
c. 38
d. 46
Disclaimer: questions are incomplete and MAY NOT INCLUDE
CoRRECT ANSWERS
20. Male breathing with Tidal volume 500ml and RR of 10/m what will
be the volume.
a. 1000
b. 1200
c. 2500
d. 4800
e. 5000
21. After forced expiration residual volume will be
a. 500
b. 1000
c. 1200
d. 2400
22. A female with regular cycle of 21 to 23 days what will be the
ovulation days
a. 12 14
b. 14 18
c. 7 9
23. A 46 year old farmer with history of discharging sinus on foot,
smear shows red granule
and filaments 1-3 microns, which is most appropriate
a. Actinomyces
b. Cornibacterium
c. Medurula mycetom
24. 15 year gardner with history of chest pain, no other symptoms, XRay was done which
showed Round mass which is calcified
a. Aspirgilosis
b. Tuberculosis
c. Sarcoidosis
d. Hydatid cyst

25. Young boy with 3 week history of facial weakness and


naopharyngeal sinusitis, on
examination bilateral facial weakness, with drooling on mouth
a. H.Influenza
b. Cornibacterium
26. Patient with defect in lumen of allantois
a. Mickels diverticulum
b. Umblical Sinus/fistula?
c. Urachal Fistula
d. Urachal Cyst
e. Urachal Sinus
27. 59 female Non Insulin dependent diabetes and HTN on
antihypertensive brought to the
emergency with semi collapsed state, K+ is 6.1 what is the cause?
a. ARF
b. CRF
c. Diuretics
d. Heart Failure?
28. Male brought to the ER with critical condition, ABGs showed PH
7.25 PCO2 65 and HCO3 24.
a. Acute Respiratory Acidosis
b. Raspiratory alkalosis
c. Partially Compensated Respiratory Acidosis
d. Partially Compensated Respiratory Alcalosis
e. Respiratory and metabolic Acidosis
29. Which of tumours does not metastasize
Disclaimer: questions are incomplete and MAY NOT INCLUDE
CoRRECT ANSWERS
a. Basal cell carcinoma
b. Squamous cell carcinoma
c. Teratoma
d. Choricarcinoma
30. Common pereonial nerve is related to
a. Biceps femoris medially
b. Biceps femoris laterally
31. Drug that is contraindicated in Intracranial HTN
a. Ketamine
b. Thiopental
c. Fentanyl
32. How to defrentiate water derivation from SIADH???
a. Plasma osmlarity
b. Urine Osmolarity
c. ADH Levek
33. A patient had resection of terminal ileum, what will result
a. Decreased Amino acid absorption
b. Increased water content in stool
c. Increased enterohepatic circulation
34. A sharp object touches the foot patient withdraws.
a. Multisynaptic reflex
35. Ejaculatory duct opens at
a. Prostatic urethera
36. Regarding renal tubular absorption which of the following is
passively absorbed.
a. Na
b. Cl
c. Urea
d. Ca
37. Regarding sarcoplasm of muscle fiber is between
a. Two Z-Lines
b. Transverse tubules
c. H-Band
d. A-Band
38. Young male comes to you with the histry of problem in errection
and is emberessed by his
enlarged breasts.
a. Prolactinoma
b. Melanocytic tumour???
c. Increased CRH
d. Incresed Gonadotrophic hormones.
39. Type 2 heart Block
40. Nerve supply of dorsum of hand just near thumb
a. Median nerve
b. Radial nerve
c. Axillary nerve
41. Cystic artery is branch of
a. Gastroduodenal
b. Left gastroepiploic
c. Right hepatic

42. How much pressure is required for the opening of aortic valve.
Disclaimer: questions are incomplete and MAY NOT INCLUDE
CoRRECT ANSWERS
a. 50
b. 100
c. 60
d. 80
43. Trigeminal ganglia
a. Anterior portion lies in trimiginal canal
b. Is along side cavernous sinus
c. Has branches from anterior and posterior parts
44. Fastest conduction in
a. A alpha
b. A beta
c. A delta
d. B Fibres
e. C Fibres
45. Isthamus of thyroid gland lies at
a. Thyroid cartilage
b. 2-3-4 tracheal ring
46. Phonation is produced by
a. Vocal cords
b. Folds of vocal cords
47. Pharyngeal part of Deglutination
a. Voluntary
b.
48. Foramen cecum represents embryological structure.
a. Thyroid
b. thymus
49. AFP is raised in
a. HCC
b.
50. Patient presented with inappropriate Speech
a. Brocas
b. Wirnickie
c.
51. Amoebic liver abscess test done to diagnose
a. X-Ray
b. CT
c. IHA
52. Trauma to Pterion will cause bleeding of
a. Middle meningeal artery
53. Female with cholelithiasis which enzyme
a. AST
b. ALT
c. Alkaline Phosphatase
d. GGT
54. Increased GFR is caused by
a. Decreased arterial pressure
b. Increase thickness of glomerular membrane
c. Aldosterone?
d. Decreased Oncotic pressure.
Disclaimer: questions are incomplete and MAY NOT INCLUDE
CoRRECT ANSWERS
55. HIV+ patient surgery done then which antiseptic to use after
surgery to clean floor
a. 1% hypochlorite
b. 4% hypochlorite
c. Gluteraldehyde
d. Soap and water
e. Water
56. Submendibular gland surgery will cause damage to
a. Lingual nerve
b. Hypoglossal nerve
57. Patient having progressive facial weakness for 3 months and
hemiplegia lesion is in.
a. Pons
b. Cereberum??
c. Supranuclear
d. Cerebello-Pontine Angle
58. Which amino acid is essential
a. Phenylalinine.
59. Renal flow is measured by.
a. Inulin
b. Para aminohippurate
60. Gustatory sweating is caused by.
61. Regarding facial nerve
62. Heparin action on DVT, it prevents

a. Emboliztion
b. Organization
c. Propagation
63. Which of the following is primary cartilaginous joint.
a. Costochondral
b. Symphyses pubis
c. Sutures of skull
64. Boy with lymphadenopathy painless, biopsy large cells with
multiple mytoses,
appropriate diagnosis by
a. Lymphocytosis on peripheral semear
b. Immunehistochemistry of leukocytes
65. Female with fever, mailase enlarged cervical lymphnodes biopsy
reveals granulamotous
lesions, appropriate diagnosis
a. Chest X-Ray
b. AFB demonstration
66. Person from sibbi Baluchistan with history of fever, anorexia and fly
bite, examination
showed hepatosplenomegaly, lymphadenopathy.
a. Kala-azar
b. Malaria
67. Lymphoma with cervical enlargement and splenic involvement
stage.
a. Stage I
b. Stage IIa
c. Stage IIb
d. Stage III
e. Stage IV
68. Regarding spleen
a. Related to Rib 5 and 6
Disclaimer: questions are incomplete and MAY NOT INCLUDE
CoRRECT ANSWERS
b. Related to right colic flexure in lower pole
c. Developed from venteral messentary
69. flexer sheath of thumb contains
a. flexor pollicis longus
b. flexor polisis longus and abductor policis longus?
c. Flexor policis brevis
d. Flexor policis longus
e. Flexor policis longus, synovial membrane and valla
something..???
70. truncal vagotomy causes
a. decreased motility
b. decreased acid secretion
c. decreased pepsin secretion
71. Histopathology grade of poorly differentiated tumour
a. 25% undefrentiated
b. 25 - 50 % undefentiated
c. 50 75 % undefrentiated
d. More than 75 % undefrentiated
72. Idiopatihc thrombocytopenic purpura lab test
a. aPTT
b. PT
c. BT
d. CT
e. Thrombin time
73. Factor IX is not available what is next best choice.
a. Cryoprecepitate
b. FFP
c. Platelets
d. Whole Blood
74. Factor VIII is not available what is next best choice.
a. Cryoprecipitate
b. Platelets
c. Whole Blood
d. PCV
75. Epiploc Appendiges are on
a. Duodenum
b. Ileum
c. Jeujenum
d. Sigmoid colon
e. Stomoch
76. Female after delivery has developed sepsis and DIC which test is
NOT appropriate
a. Fibrin degradating products
b. Bleeding time
77. Which of the following is most malignant

a. Leukoplakia
b. Erythroplakia
c.
78. Estrogin containing pills are associated with
a. Breast cancer.
b.
Disclaimer: questions are incomplete and MAY NOT INCLUDE
CoRRECT ANSWERS
79. Female with right ovarian fibrosis, surgery is done which artery is
most likely to be
damaged.
a. External iliac
b. Internal iliac
c. Uterine artery
80. Testicular tumour will first drain to
a. Exernal iliac nodes
b. Internal iliac nodes
c. Para aortic nodes
81. Regarding pempaniform plexus
a. In inguinal canal
82. Hernia medial to inferior epigastric artery.
a. Indirect
b. Direct
c. Femoral
83. Regarding kidney
a. Kidneay and surrounding fat coverd by gerrota fascia
b. Giliasons capsule
84. Spinal cord anteriorly attached to
a. Alar ligament
b. Filum terminale
85. Regarding spironolactone
a. IV dose 100mg
b. Is steroid
86. Which of the following disease in pregnancy will cause cataract
a. Measles
b. Rubella during pregnancy
c.
87. Tumour in right upper lobe will cause collape of
a. Right upper apical
b. Right lower
c.
88. Bronchogenic carcinoma
a. Smoking
b. asbestosis
89. 1 year boy with anaemia showing microcytic cells, failure therive,
elder brother regular
blood transfusion
a. Hemophelia
b. Sickle cell anemia
c. Thelesemia
d.
90. Two brothers having hemophelia and but sister does not have what
is inheritance
a. Autosomal Dominant
b. Autosomal Recessive
c. X-Linked Dominant
d. X-Linked Recessive
91. Left circumflex artery supplies
92. Diabetic Ketoacidosis patient managed but dies, infection
a. Mucur
Disclaimer: questions are incomplete and MAY NOT INCLUDE
CoRRECT ANSWERS
93. Patient developed SCC of bladder 8 years back had
granulomatous appearance of bladder
biopsy.
a. Schistosoma
b. Tuberculosis
c. Bladder stone
d. Nitrosamines
e.
94. Vien of heart which itself converts to coronary sinus.
a. Great cardiac vein
b. Middle cardiac vein
95. Female with high grade fever, anemia and hematuria
a. P.falciparum
96. Carpus Stritum
a. Lentiform + globus pallidus
b. Caudate + globus pallidus

c. Caudate + lentiform
d. Amagdaloid + globus
e.
97. Most common intracellar buffer
a. HCO3
b. Hemoglobin
c. PO4
d. Na
e. Cl
98. Oxygen storage in muscle
a. Myoglobin
99. What causes osmotic diuresis
a. Manitol
100. Pharmocodynamica Drug-Drug interaction
a. Thiazide and lithium
b. Morphine and naloxone
101. Pus of follicle contains
a. Dead bacteria
b. Dead Neutrophils
102. Basic drug binds to
a. Albumin
b. Globulin.
c. A1glycoprotein
d. Protein
103. Chromosome lines in center in
a. Metaphase
b. Prophase
c. Anaphase
d. Telophase
e. Prometaphase
104. Question regarding apoptosis
105. Male presents with dull right sided chest what is the cause
a. Pleural effusion.
b. Pneumonia
Disclaimer: questions are incomplete and MAY NOT INCLUDE
CoRRECT ANSWERS
106. Male died after adenocarcinoma, autopsy showed thousands of
polyps in colon, type
of polys.
a. Adenomatous
b. Hyperplastic
c. Hemartomous
d. Metaplastic
107. Which of the following is blood supply of head of femur
a. Medial circumflex humural
b. Lateral cirumflex humural
c. Obturator
d. Nutrient
108. Flexor of knee
a. Sartorius
b. Gastrocnemius
c. Adductor magnus
d. Biceps femoris
109. Major Breast lymphnodes, to parrelel???
a. Along Axillary vessels apical nodes.
b. Along intercostal vessels to posterior thoracic node,
c. Along internal thoracic to internal thoracic nodes???
110. Biopsy of tissue shows Lymphoid tissue with stratified squamous
epithelium
a. Palatine tonsils
b. Lymphnodes
c. Pyers patches
d. Thymus
111. Basophilia is caused by
a. Nucleus
b. Ribosomes
c. Cytoplasm
d. Endoplasmic reticulum
112. Regarding spinal nerves
a. Exit from intervertebral foramen
b. Derived from neural crest only?
113. Female having complains of dysphagia, and weakness labs show
hb 6.2 MCV 68
a. Iron deficiency anemia
114. Cervical cancer increased risk in
a. Chronic irritation
b. IUCD
c. HPV

115. Female with cushing disease came for investigations, her cortisol
level is <1 what is
most important question to ask
a. Having Headache
b. Taking any Oral prednisolone
116. Patient with crf on dialysis with anemia what should be given
a. Erythropoietin
117. Regarding trachea
a. C-shaped rings with cavity anteriorly
b. 15 cm
118. Painful defecation nerve supply
a. Pudenal nerve
119. Boy with high grade fever and tender abdomen, X-Ray shows
pneumoperitoneum,
before surgery antibiotic should be given
a. Ceftrizime?
b. Gentamycin
120. Patient with C.Botulinum food poisoning, characteristic feature will
be
a. Diarrhoeab. Vomitingc. Respiratory muscle depression
121. Following muscle does not attach to humerous
a. Pectoralis major
b. Pectoralis minor
c. Teres minor
d. Subscapularis
e. Supraspinatus
122. Patient with Tobacco chewing showing ulcer on iner lip margin
which is indurated and
pale is
a. Squamous cell carcinoma
b. Basal cell carcinoma
c. Melanoma
123. Medial longitudinal arch piller
a. Tallus
b. Calcaneum
c. Navicular
124. Structures posterior to medial malleolus except.
a. Tibial nerve
b. Post tibial artery
c. Flexor halluces longus
d. Flexor halluses bravis
e. Tibialis anterior
gynea august 2014
1. A Female during develivery looses blood and requires blood. Her
blood sample is reactive
blood agglutinates with anti sera B and anti sera D. which blood is
suitable for transfusion.
a. A+
b. B RH Positive
c. O +
d. AB +
2. Following arches over left lung root.
a. Thoracic ductb. Arch of aortac. Azygus vein
3. Patient IVC is blocked just above the emergence of azygus vein,
where blood will divert.
a. Right gastric arteryb. Hepatic veinc. Hepatic arteryd. Umbilical
arterye. Splenic artery
4. Internal Jugular vein is related
a. Anterior to Carotid arteryb. Medially to Carotid Arteryc. Laterally to
Carotid arteryd. Posteriorly to Carotid arterye. Internal Carotid Artery
posteriorly
5. Rapid adaptation receptors
a. Free nerve endingsb. Meckels diskc. Pacinian corpusclesd. Ruffinis
corpuscles
6. Hypermeagnesemia causes
a. Hyperreflexiab. Dec acetyl choloine uptake
7. Best example against single organ?
a. Hashimoto thyroiditisb. PANc. SLEd. RA
8. Hormone required for brain development
a. Thyroid hormoneb. Growth hormonec. Cortisold. Androgene. Insulin
9. Ketamine used in repeated burn dressings because
a. It does not cause hypotention
b. Causes profound analgesia
c. It can be given IV
d. It does not causes addiction

e. Its not narcotic


10. Hanging causes death by
a. Fracture of odontoid process
Disclaimer: questions are incomplete and MAY NOT INCLUDE
CoRRECT ANSWERS
b. Fracture of atlas
11. Mecanisim of action of propylthiouracil
a. Inhibition Iodine pump
b. Blocks the synthesis of thyroid hormone.
c. Blocks the synthesis of thyroglobulin.
d. Inhibits hormone release
e. Decreases TSH
12. Last thing to be returned normal after haemorrhagic shock
a. Heart rate
b. Blood pressure
c. Cardiac contractility
d. RBC in peripheral smear
e. Pulse pressure
13. Antibodies are produced by
a. Lymphocytes
b. Plasma cells
c. T-Cells
d. Neutrophils
e. Endothelium
14. 15 year old male had history of right iliac fossa pain for 18 hours
and was operated for
appendicitis, histopathology will show which cell
a. Lymphocytes
b. Macrophages
c. Neutrophils
15. Which of congenital malformation is most common
a. ASD
b. VSD
c. TS
16. Male with transmural infarction died suddenly in washroom after 8
days.
a. Atrial fibrillation
b. Ventricular Fibrilation
c. Cardiac Temponade
d. Pulmonary embolism
17. After trauma Female with sudden acute chest pain, Dyspnoea,
hemoptysis and shock and
then death.
a. Fat embolism
b. Air Embolism
c. Peunomionia
18. G1P2 female after delivery develops sudden chest pain and
difficulty in breathing and
hemoptysis, ECG shows S1 Q3 T3 pattern which is most appropriate
test.
a. LDH
b. SGO?PT
c. X-Ray
d. CPK
19. In complete Glucose metabolism number of ATP
a. 30
b. 32
c. 38
d. 46
Disclaimer: questions are incomplete and MAY NOT INCLUDE
CoRRECT ANSWERS
20. Male breathing with Tidal volume 500ml and RR of 10/m what will
be the volume.
a. 1000
b. 1200
c. 2500
d. 4800
e. 5000
21. After forced expiration residual volume will be
a. 500
b. 1000
c. 1200
d. 2400
22. A female with regular cycle of 21 to 23 days what will be the
ovulation days
a. 12 14
b. 14 18
c. 7 9

23. A 46 year old farmer with history of discharging sinus on foot,


smear shows red granule
and filaments 1-3 microns, which is most appropriate
a. Actinomyces
b. Cornibacterium
c. Medurula mycetom
24. 15 year gardner with history of chest pain, no other symptoms, XRay was done which
showed Round mass which is calcified
a. Aspirgilosis
b. Tuberculosis
c. Sarcoidosis
d. Hydatid cyst
25. Young boy with 3 week history of facial weakness and
naopharyngeal sinusitis, on
examination bilateral facial weakness, with drooling on mouth
a. H.Influenza
b. Cornibacterium
26. Patient with defect in lumen of allantois
a. Mickels diverticulum
b. Umblical Sinus/fistula?
c. Urachal Fistula
d. Urachal Cyst
e. Urachal Sinus
27. 59 female Non Insulin dependent diabetes and HTN on
antihypertensive brought to the
emergency with semi collapsed state, K+ is 6.1 what is the cause?
a. ARF
b. CRF
c. Diuretics
d. Heart Failure?
28. Male brought to the ER with critical condition, ABGs showed PH
7.25 PCO2 65 and HCO3 24.
a. Acute Respiratory Acidosis
b. Raspiratory alkalosis
c. Partially Compensated Respiratory Acidosis
d. Partially Compensated Respiratory Alcalosis
e. Respiratory and metabolic Acidosis
29. Which of tumours does not metastasize
Disclaimer: questions are incomplete and MAY NOT INCLUDE
CoRRECT ANSWERS
a. Basal cell carcinoma
b. Squamous cell carcinoma
c. Teratoma
d. Choricarcinoma
30. Common pereonial nerve is related to
a. Biceps femoris medially
b. Biceps femoris laterally
31. Drug that is contraindicated in Intracranial HTN
a. Ketamine
b. Thiopental
c. Fentanyl
32. How to defrentiate water derivation from SIADH???
a. Plasma osmlarity
b. Urine Osmolarity
c. ADH Levek
33. A patient had resection of terminal ileum, what will result
a. Decreased Amino acid absorption
b. Increased water content in stool
c. Increased enterohepatic circulation
34. A sharp object touches the foot patient withdraws.
a. Multisynaptic reflex
35. Ejaculatory duct opens at
a. Prostatic urethera
36. Regarding renal tubular absorption which of the following is
passively absorbed.
a. Na
b. Cl
c. Urea
d. Ca
37. Regarding sarcoplasm of muscle fiber is between
a. Two Z-Lines
b. Transverse tubules
c. H-Band
d. A-Band
38. Young male comes to you with the histry of problem in errection
and is emberessed by his
enlarged breasts.

a. Prolactinoma
b. Melanocytic tumour???
c. Increased CRH
d. Incresed Gonadotrophic hormones.
39. Type 2 heart Block
40. Nerve supply of dorsum of hand just near thumb
a. Median nerve
b. Radial nerve
c. Axillary nerve
41. Cystic artery is branch of
a. Gastroduodenal
b. Left gastroepiploic
c. Right hepatic
42. How much pressure is required for the opening of aortic valve.
Disclaimer: questions are incomplete and MAY NOT INCLUDE
CoRRECT ANSWERS
a. 50
b. 100
c. 60
d. 80
43. Trigeminal ganglia
a. Anterior portion lies in trimiginal canal
b. Is along side cavernous sinus
c. Has branches from anterior and posterior parts
44. Fastest conduction in
a. A alpha
b. A beta
c. A delta
d. B Fibres
e. C Fibres
45. Isthamus of thyroid gland lies at
a. Thyroid cartilage
b. 2-3-4 tracheal ring
46. Phonation is produced by
a. Vocal cords
b. Folds of vocal cords
47. Pharyngeal part of Deglutination
a. Voluntary
b.
48. Foramen cecum represents embryological structure.
a. Thyroid
b. thymus
49. AFP is raised in
a. HCC
b.
50. Patient presented with inappropriate Speech
a. Brocas
b. Wirnickie
c.
51. Amoebic liver abscess test done to diagnose
a. X-Ray
b. CT
c. IHA
52. Trauma to Pterion will cause bleeding of
a. Middle meningeal artery
53. Female with cholelithiasis which enzyme
a. AST
b. ALT
c. Alkaline Phosphatase
d. GGT
54. Increased GFR is caused by
a. Decreased arterial pressure
b. Increase thickness of glomerular membrane
c. Aldosterone?
d. Decreased Oncotic pressure.
Disclaimer: questions are incomplete and MAY NOT INCLUDE
CoRRECT ANSWERS
55. HIV+ patient surgery done then which antiseptic to use after
surgery to clean floor
a. 1% hypochlorite
b. 4% hypochlorite
c. Gluteraldehyde
d. Soap and water
e. Water
56. Submendibular gland surgery will cause damage to
a. Lingual nerve
b. Hypoglossal nerve

57. Patient having progressive facial weakness for 3 months and


hemiplegia lesion is in.
a. Pons
b. Cereberum??
c. Supranuclear
d. Cerebello-Pontine Angle
58. Which amino acid is essential
a. Phenylalinine.
59. Renal flow is measured by.
a. Inulin
b. Para aminohippurate
60. Gustatory sweating is caused by.
61. Regarding facial nerve
62. Heparin action on DVT, it prevents
a. Emboliztion
b. Organization
c. Propagation
63. Which of the following is primary cartilaginous joint.
a. Costochondral
b. Symphyses pubis
c. Sutures of skull
64. Boy with lymphadenopathy painless, biopsy large cells with
multiple mytoses,
appropriate diagnosis by
a. Lymphocytosis on peripheral semear
b. Immunehistochemistry of leukocytes
65. Female with fever, mailase enlarged cervical lymphnodes biopsy
reveals granulamotous
lesions, appropriate diagnosis
a. Chest X-Ray
b. AFB demonstration
66. Person from sibbi Baluchistan with history of fever, anorexia and fly
bite, examination
showed hepatosplenomegaly, lymphadenopathy.
a. Kala-azar
b. Malaria
67. Lymphoma with cervical enlargement and splenic involvement
stage.
a. Stage I
b. Stage IIa
c. Stage IIb
d. Stage III
e. Stage IV
68. Regarding spleen
a. Related to Rib 5 and 6
Disclaimer: questions are incomplete and MAY NOT INCLUDE
CoRRECT ANSWERS
b. Related to right colic flexure in lower pole
c. Developed from venteral messentary
69. flexer sheath of thumb contains
a. flexor pollicis longus
b. flexor polisis longus and abductor policis longus?
c. Flexor policis brevis
d. Flexor policis longus
e. Flexor policis longus, synovial membrane and valla
something..???
70. truncal vagotomy causes
a. decreased motility
b. decreased acid secretion
c. decreased pepsin secretion
71. Histopathology grade of poorly differentiated tumour
a. 25% undefrentiated
b. 25 - 50 % undefentiated
c. 50 75 % undefrentiated
d. More than 75 % undefrentiated
72. Idiopatihc thrombocytopenic purpura lab test
a. aPTT
b. PT
c. BT
d. CT
e. Thrombin time
73. Factor IX is not available what is next best choice.
a. Cryoprecepitate
b. FFP
c. Platelets
d. Whole Blood
74. Factor VIII is not available what is next best choice.
a. Cryoprecipitate

b. Platelets
c. Whole Blood
d. PCV
75. Epiploc Appendiges are on
a. Duodenum
b. Ileum
c. Jeujenum
d. Sigmoid colon
e. Stomoch
76. Female after delivery has developed sepsis and DIC which test is
NOT appropriate
a. Fibrin degradating products
b. Bleeding time
77. Which of the following is most malignant
a. Leukoplakia
b. Erythroplakia
c.
78. Estrogin containing pills are associated with
a. Breast cancer.
b.
Disclaimer: questions are incomplete and MAY NOT INCLUDE
CoRRECT ANSWERS
79. Female with right ovarian fibrosis, surgery is done which artery is
most likely to be
damaged.
a. External iliac
b. Internal iliac
c. Uterine artery
80. Testicular tumour will first drain to
a. Exernal iliac nodes
b. Internal iliac nodes
c. Para aortic nodes
81. Regarding pempaniform plexus
a. In inguinal canal
82. Hernia medial to inferior epigastric artery.
a. Indirect
b. Direct
c. Femoral
83. Regarding kidney
a. Kidneay and surrounding fat coverd by gerrota fascia
b. Giliasons capsule
84. Spinal cord anteriorly attached to
a. Alar ligament
b. Filum terminale
85. Regarding spironolactone
a. IV dose 100mg
b. Is steroid
86. Which of the following disease in pregnancy will cause cataract
a. Measles
b. Rubella during pregnancy
c.
87. Tumour in right upper lobe will cause collape of
a. Right upper apical
b. Right lower
c.
88. Bronchogenic carcinoma
a. Smoking
b. asbestosis
89. 1 year boy with anaemia showing microcytic cells, failure therive,
elder brother regular
blood transfusion
a. Hemophelia
b. Sickle cell anemia
c. Thelesemia
d.
90. Two brothers having hemophelia and but sister does not have what
is inheritance
a. Autosomal Dominant
b. Autosomal Recessive
c. X-Linked Dominant
d. X-Linked Recessive
91. Left circumflex artery supplies
92. Diabetic Ketoacidosis patient managed but dies, infection
a. Mucur
Disclaimer: questions are incomplete and MAY NOT INCLUDE
CoRRECT ANSWERS
93. Patient developed SCC of bladder 8 years back had
granulomatous appearance of bladder

biopsy.
a. Schistosoma
b. Tuberculosis
c. Bladder stone
d. Nitrosamines
e.
94. Vien of heart which itself converts to coronary sinus.
a. Great cardiac vein
b. Middle cardiac vein
95. Female with high grade fever, anemia and hematuria
a. P.falciparum
96. Carpus Stritum
a. Lentiform + globus pallidus
b. Caudate + globus pallidus
c. Caudate + lentiform
d. Amagdaloid + globus
e.
97. Most common intracellar buffer
a. HCO3
b. Hemoglobin
c. PO4
d. Na
e. Cl
98. Oxygen storage in muscle
a. Myoglobin
99. What causes osmotic diuresis
a. Manitol
100. Pharmocodynamica Drug-Drug interaction
a. Thiazide and lithium
b. Morphine and naloxone
101. Pus of follicle contains
a. Dead bacteria
b. Dead Neutrophils
102. Basic drug binds to
a. Albumin
b. Globulin.
c. A1glycoprotein
d. Protein
103. Chromosome lines in center in
a. Metaphase
b. Prophase
c. Anaphase
d. Telophase
e. Prometaphase
104. Question regarding apoptosis
105. Male presents with dull right sided chest what is the cause
a. Pleural effusion.
b. Pneumonia
Disclaimer: questions are incomplete and MAY NOT INCLUDE
CoRRECT ANSWERS
106. Male died after adenocarcinoma, autopsy showed thousands of
polyps in colon, type
of polys.
a. Adenomatous
b. Hyperplastic
c. Hemartomous
d. Metaplastic
107. Which of the following is blood supply of head of femur
a. Medial circumflex humural
b. Lateral cirumflex humural
c. Obturator
d. Nutrient
108. Flexor of knee
a. Sartorius
b. Gastrocnemius
c. Adductor magnus
d. Biceps femoris
109. Major Breast lymphnodes, to parrelel???
a. Along Axillary vessels apical nodes.
b. Along intercostal vessels to posterior thoracic node,
c. Along internal thoracic to internal thoracic nodes???
110. Biopsy of tissue shows Lymphoid tissue with stratified squamous
epithelium
a. Palatine tonsils
b. Lymphnodes
c. Pyers patches
d. Thymus
111. Basophilia is caused by

a. Nucleus
b. Ribosomes
c. Cytoplasm
d. Endoplasmic reticulum
112. Regarding spinal nerves
a. Exit from intervertebral foramen
b. Derived from neural crest only?
113. Female having complains of dysphagia, and weakness labs show
hb 6.2 MCV 68
a. Iron deficiency anemia
114. Cervical cancer increased risk in
a. Chronic irritation
b. IUCD
c. HPV
115. Female with cushing disease came for investigations, her cortisol
level is <1 what is
most important question to ask
a. Having Headache
b. Taking any Oral prednisolone
116. Patient with crf on dialysis with anemia what should be given
a. Erythropoietin
117. Regarding trachea
a. C-shaped rings with cavity anteriorly
b. 15 cm
118. Painful defecation nerve supply
Disclaimer: questions are incomplete and MAY NOT INCLUDE
CoRRECT ANSWERS
a. Pudenal nerve
119. Boy with high grade fever and tender abdomen, X-Ray shows
pneumoperitoneum,
before surgery antibiotic should be given
a. Ceftrizime?
b. Gentamycin
120. Patient with C.Botulinum food poisoning, characteristic feature will
be
a. Diarrhoea
b. Vomiting
c. Respiratory muscle depression
121. Following muscle does not attach to humerous
a. Pectoralis major
b. Pectoralis minor
c. Teres minor
d. Subscapularis
e. Supraspinatus
122. Patient with Tobacco chewing showing ulcer on iner lip margin
which is indurated and
pale is
a. Squamous cell carcinoma
b. Basal cell carcinoma
c. Melanoma
123. Medial longitudinal arch piller
a. Tallus
b. Calcaneum
c. Navicular
124. Structures posterior to medial malleolus except.
a. Tibial nerve
b. Post tibial artery
c. Flexor halluces longus
d. Flexor halluses bravis
e. Tibialis anterior
1ST JAN 2015 SURGERY AND ALLIED:
1. Most appropriate about Clavipectroral Fascia:
A. Above the clavicle connects wid Pretracheal FaciaB. Part of
Endothorac FasciaC. Covers Pectoralis MajorD.Covers Pectoralis
Minor
2. If Lateral Cord of Brachial Plexus will damage which muscle will get
paralysed most likely:
A. Pectoralis MajorB.Pectoralis MinorC. Teres MajorD. Teres Minor
3. Patient with fluid loss now show metabolic acidosis e hypokalemia.
most fluid loss will be through:
A. StomachB. Colon C. PancreasD. DuodenumE. Jujenum
4. Fracture of surgical neck of Humerus damage to:

A. Axillary Nerve
5. Inversion of foot by which muscle:
A. Peronues LongusB. Peroneus BrevisC. Peroneus tertiusD. Tibialis
Posterior
6. Pain from Gallbladder felt in Anterior Triangle of neck, this
mechanism of Reffered Pain appropraite statement is:
A. Vagus Nerve has wide distribution in thorax and most of abdominal
organsB. Accersory nerve gives fibers to GallbladderC.
Parasympathetic something
7. Lateral foot dermatome:
A. S1B. S3c. L4
8. Regarding jerks corresponding with spinal segments:
A. Biceps- C6B. Tricep- C8
All other options were wrong but didnt know what to choose among
above two.
9. MELAS syndrome histological finding:
A. Loflar bodiesb. Inclusion bodiesC. Granulomas
D. CystE. Red ragged fibers
(some says inclusion and some say LOFLAR)
10. Giant Cell Tumors histo finding:
A. Spotted calcificationsB. Soap Bubble appearance
11. Blast Cells in peripheral blood:
A. Iron deficiency anemiasB. Erythroblastosis fetalisC. Folate
deficiencyD. Bone marrow depression
12. Most common cause of Hepatocellular CA in developing countries:
A. HEP B, CB. Hep Cc. Hep B
13. Young adult with raised ALP, jaundice and decreased Hb Scenario
A. Cholesterol StonesB. Pigment StonesC. CholangioCAD. Hepatitis
14. Regarding Lymph drainage of breast except:
A. ApicalB. Supraclavicular NodesC. Pectoral NodesD. Inferior Phrenic
Nodes
15. Closure of lips involve: or unable to close lips which muscle
involved:
1. Depressor anguli oris2. Depressor labii inferioris3. Depressor labi
superioris4. Orbicularis oris ans5. Lateral pterygoid
16. Superior Thyroid artery arise from:
A. First branch of Subclavian ArteryB. External Carotid ArteryC.
Internal Carotid Artery
17. Incubation period of which disease is longest:
1. Measles2. Rubella3. Chicken pox4. Infectious mononucleosis 5.
Mumps
18. Chronic Fungal infection with rhinosinusitis causing medial erosion
and granulomas:
1. Cryptococcosis2. Mucormycosis3. Rhinophycomycosis4.
Histoplasmosis5. Aspergillus
19. Which of following has highest chances of developing endometrial
cancer
1. Simple hyperplasia2. Complex hyperplasia without atypia3. Complex
hyperplasia with atypia4. Squamous metaplasia
5. Chronic endometritis
20. Regarding CSF
1. Formed by ependymal cells2. Formed 500ml/day3. Formed only
30percent by chorioid plexus4. Arachnoid villi are visible
There were two questions about CSF.
21. Age estimation at fifth to 12th week is done through:
A. Biparietal diameterB. Crown rump length C. Crown heel lengthD.
Abdominal circumference
22.A sprinter during running had injury to his ankle. He can stand on
his toes but with severe pain.. There is a visible ecchymoses around
ankle joint:

A. Plantaris tendon
23.Otic ganglion is between mandibular nerve and tensor tympani. It's
superior border is formed by:
A. Foramen lacerumB. Foramen ovale C. Foramen rotundumD. Styloid
process
24. Amoeba causes lesions in which part of gut:
A. Terminal ileumB. CecumC. Ascending colonD. Transverse colonE.
Sigmoid and rectum
25. A young boy with inc appetite weight gain and sleep problems. On
examination he is having genitals hypoplasia:
A. HypothyroidismB. Pineal tumorC. CraniopharyngiomaD.
NarcolepsyE. Cushing disease
26. Sensory supply of uterus passes through which ligament:
1. Broad ligament2. Round ligament3. Cardinal ligament4. Uterosacral
ligament5. Uterocervical ligament
27. Highest number of alpha resceptors are present in:
1. Trigone2. Neck of bladder and proximal urethra3. Penile urethra4.
Membranous urethr5. External urethral opening
28. Regarding elbow joint:
1. Capsule deficient posteriorly2. Covered all around by muscles3.
Hinge joint4. Supination occurs here5. Joint between humerus, radius,
ulna
29. A needle when inserted 2cm below and lateral to pubic tubercle will
be closely related to on:
A. Obturator nerveB. Ilioinguinal nerveC. Sciatic nerveD. Femoral
nerve
30. Regarding Pterygopalatine Raphe:
A. BuccinatorB. Inferior constrictorC. Parts from all constrictorsD.
Middle constrictor
31. Most important characteristic of Glycogen Storage diseases are:
A. Hyperglycemia with HepatomegalyB. Hyperglycemia with
splemomegalyC. Hypoglycemia with HepatosplenomegalyD.
Hypoglycemia with Hepatomegaly
32. Buccopharyngeal Membrane:
A. Stomodium from ForegutB. Nose from Pharynx
33. PTU:
A. inhibits synthesis of thyroid hormonesB. inhibits release of thyroid
hormones
34. Most common position of appendix:
A. RetrocecalB. Pelvic
35. Pretracehal fascia infections spreads to:
A. Anterior mediastinumB. Posterior mediastinum
36. Sensations from tip of fingers by:
A. a fibers
B. a beta fibers
C. a delta fibersD. c fibers
37. Shivering center is located in:
A. Anterior hypothalamus
B. Posterior hypothalamus
38. Which tumor involves extension to nerve sheath:
1. Myxomas2. Adenomas3. Hemangiopericytoma4. Carcinoma
expleomorphic adenoma
39. Anterior epithelium of cornea is:
1. Stratified squamous keratinized2. Simple squamous3. Simple
columnar4. Stratified Squamous non keratinized
40. A 30 year old woman in azad kashmir was injured mildly in an
earthquake that killed many people apart from her distant uncle. She
would most probably be in
1. Social crisis2. Personal crisis3. Situational crisis4. Developmental
crisis
41. While a surgery on submandibular gland which has more chances
of injury:
1. Superior thyroid artery2. Lingual nerve3 Mandibular nerve4. Facial
nerve

42. Function exclusively linked with vit E:


1. Carbohydrate metabolism2. Endothelial protection3. Skin integrity
43. Which one combination is correct:
1. Thiopental reduces heart rate2. Profofol increases cerebral blood
flow3. Morphine reduces apnoeic threshold4. Ketamine causes
bronchodilation
44. A 31 year old young man has heaviness in his scrotum for six
months. On examination a solid 5cm mass is palpated. On labs his
alpha feto protein level was 81ng/ml while bHCG level was 15 IU/l.
Same side orchiectomy was done and Which of following could be the
cause
1. Leydig cell tumor2. Embryonal carcinoma3. Choriocarcinoma4
Teratoma
45. Which ligament supports odontoid process of axis on atlas( stem
not sure)
A. Posterior longitudinal ligamentB. Anterior longitudinal ligamentC.
Tranverse ligamentD. Ligamentum nuchae
46. A patient came in ER, with bleeding from i guess Femoral Artery
something. and due to hypovolemia which will be LEAST affected:
A. MyocardiumB. Brainc. RetinaD. Skeletal MusclesE. Intestines
47. Medial geniculate body
A. Receives oculomotor fibersB. Optic tract fibersC. Fibers ascend to
thalamocortical areas D. Solely to thalamUSE. Project to Visual Cortex
48. Cerebral Malaria caused by:
A. Plasmodium FalciparumB. Plasmodium Malariae
49. C7 Vertebrae has:
A. Longest spineB. Broad body
50. PT Is indicated by:
A. XIIIB. VIIIC. IXD. Prothrombin (or Thrombin) was an option
(and 7 wasnt in da options)
51. Percentage of Plasma out of ECF:
A. 30%B. 20%C. 50%
52. Direct Inguinal Hernia:
A. Medial to epigastric Artery
53. Newborn with a Erythroblastosis Fetalis having Bloodgp Bpos what
is best for management:
Any blood with Rhpostive?
cant rem options and correct ones
54. A man during fasting his urine become concentrated because of:
A. Decrease intake of water
B. Adh secretion
55. Regarding Thalesemia
A. Defect in ratio of chains
B. Defect in globin production
56. For assessment of nutritional status of a person alongwith weight
loss and something u will assess:
A. MineralsB. VitaminC. Albumin and something
(Option and stem were imp in this question which no one could recall
properly so cant really confirm the answer now and offcourse we most
of us didnt know if whatever they marked was right)
57. Most radiosenstive tumour:
A. Glioblastoma multiforme
B. Glioglioma
C. Craniopharyngioma
58. If a person is taking Protein in his diet which will be necessary:
A. RiboflavinB. Thiamine
59. Regarding ADH secretion:
A. Released in increased Plasma osmolarity

60. Scenario bif on Marfans wid mitral valve prolapse died and on
autopsy is prolapse this was due to defect in
A. Fibrillin
B. Spectrin

A. Part of pterion
B. Posteroinferior prominent part of occipital bone
C. Posterior to frontal bone
D. Internally to occipital bone

61. Group of young men bathing n beech next day develop blisters on
back shoulder limbs region cause is:
A. Mast cell destruction B. Endothelium venules destruction

81.Function exclusively linked with vit E:

62. Obstruction to RCA after giving Marginal branch will affect:


A. AV Node
63. Regarding HEART what is appropriate:
A. SA Node generates spontaneous impulses.
64. Second Heart sound produced by:
A. Closure of aortic and pulmonary valve
65. Left Atrium supplied by:
A. LCX
66. Inferior Wall MI artery involved:
A. Right Marginal Artery

A. Carbohydrate metabolism
B. Endothelial protection
C. Skin integrity
D. Germinal epithelial protection (seniors confirmed)
82.Heterophile antibody is seen in:
A. Infectious mononucleosis
B. Herpes simplex
C. Gonorrhea
83.Deep to post digastric and near palatoglossus a structure runs
obliquely upwards
A. Facial artery
B. Lingual artery
C. Maxillary artery
D. Sup thyroid

67. Aortic valve closure outsets:


A. Rapid filling
B. Isovlumic Relaxation (or Isovolumetric relaxation)
C. Isovoulmetric Contraction

84.Nucleus of general visceral efferents of tenth nerve to palate


muscles is present in:

68. Highest Cardiac Output in:


A. AnemiaB. PregnancyC. Beri Beri

85.Which tumor involves extension to nerve sheath:

69. Most suitable NSAID during Lactation:


A. IbuprofenB. PeroxicamC. Naproxen
70. Malignant tumour mesenchymal in origin:
A. Sarcoma
71. Renal Column:
A. Collecting DuctB. Interlobar arteryC. Interlobular artery
D. Minor calyces
72. 18 year old girl has fever tenderness in right ileac fossa. On labs
WBC 17000cells/mm3. Appendicectomy was preformed. Which of the
following would be seen on histology
1. T lymphocytes in wall of appendix
2. Polymorphonuclear neutrophills
3. Glissening mass at apex of appendix
73. Myxomas most common location:
A. Left Atrium
74. Regarding Bain Bridge Reflex:
A. Increases HR
B. Decreases HRC. Maintains posture and balance
(The Bainbridge reflex, also called the atrial reflex, is an increase in
heart rate due to an increase in central venous pressure.[1] Increased
blood volume is detected by stretch receptors (baroreceptors) located
in both atria at the venoatrial junctions)
75. Surgeons injects dye during surgery through:
A. Ligamentum Teres

A. Locus ceruleusB. Tractus solitariusC. Nucleus ambiguous


D. Nucleus raphe
1. Myxomas2. Adenomas3. Hemangiopericytoma
4. Carcinoma expleomorphic adenoma
86.Loss of abduction and adduction of fingers and adduction of thumb
with intact skin sensations involves:
1. Recurrent branch of median2. Deep branch of ulnar
87.A 52 year old man underwent coronary artery bypass graft two
weeks ago and now presents with purpuric patches on skin. On labs
his PT and aPTT were normal. Following is true:
A. Heparin induced thrombocytopenia
B. Drug induced thrombocytopenia
88.Regarding pneumotoxic center following is true:
A. Increases inspiratory potentialB. Fires dynamic respiratory drive
89. What is correct about thyroid:
A. Supplied by 3 paired arteries B. Surrounded by prevertebral fascia
C. Isthmus attached to cricoidD. Venous drainage in bracheocephalic
vein
90. Regarding Pituitary gland:
A. Secretes TSH
91. Color differentiation due to red and green:
A. Protonopia
92. Dental carries due to: (or infection due to)
A. BacillusB. LactobacillusC. SretptococcusD. Staph
93. Layer of the viscera from outermost to innermost:
A. Serosa subserosa muscularis mucosa submucosa mucosa

76. Appendicular Artery is a branch of:


A. Posterior Cecal ArteryB. Iliocolic Artery

(all of the options were very close and with slight shuffling)

(Confirmed by all seniors, donot confuse urself and follow seniors


advice)

94. Dysphagia due to liquids only:


A. Plummer Vinson syndromeB. Neuromuscular blockade

78. Bladder carcinoma:


A. Schistosoma hematobium

95. Cholangiocarcinoma spread by:


A. Clonorchs sinuses

79. Regarding Thymus:


A. Hassals corpuscles
B. synthesis of t lymphocytes within lobule

96. Post 1/3 of the tongue supplied by:


A. Cranial nerve 9

80.Inion is:

97. Blood group A+ transfusions reaction due to:


A. A+B. B+C. O-D. A-

98. H2O2 found in:


A. PeroxisomeB. Mitochondirac. LysosomeD. RER
99. ICAM and VCAM are for:
A. Leucocyte adhesion
100. Breast atrophy in young female is due to:
A. Estrogen
B. Estrogen and Progesterone
C. Progesterone

we had so many questions who had such vague statements, and


missing key words. so cant really decide the mostt appropriate one for
this question also.
(It supplied by deep peroneal, saphenous, sural and tibial nerves.
Occasionally, the superficial peroneal nerve also supplies the ankle
joint)
114. Regarding Lesser Omentum:
A. Contains Hepatic DUCT and artery

101. Regarding cancer mutations in:


A. Protooncogen

B. Attaches to second part of duodenum

102. Essential amino acid:


A. Phenylalanine

D. have no attachment to liver

103. Fecal incontinence after trauma damge to:


A. External anal sphincter

115. Nucleous Pulposus herniation will cause compression of:

104. Joining of ICA and Basilar artery:


A. Posterior communicating Artery

other options seemed incorrect

105. Regarding BI Agonists:


A. Increase HR

A. Respiratory failure

106. Poliomyelitis:
A. Anterior horn cell defect

A. Paraaortic LN
118. Regarding Lower Limb:
A. Short Saphenous Vein form from superficial arch something.
B. Long Saphenous ends at mid of thigh
(so hard to recall this question options, they were weird n confusing.
better to read the topic)
119. Antiseptic for skin:
A. Phenol with clorhexidine
Cant recall other statements but i guess i have seen this question
somewhere.

107. Scenario about CHF than asked about reason for edema:
A. Dec Colloid osmotic pressure
108. Regarding P Value:
A. Checks probability
B. Percentile something..
109. Regarding H2 Blockers:
A. Inhibits both gastric acid secretion and parasympathetic
110. Mechanism of Vibro cholera:
A. Cl channelB. dec HCO3C. IP3 something
Cant really recall correct statements about this questions so its better
to read the topic and save it in ur mind if question comes again you
can choose da correct one :)
(The cholera toxin (CTX or CT) is an oligomeric complex made up of
six protein subunits: a single copy of the A subunit (part A), and five
copies of the B subunit (part B), connected by a disulfide bond. The
five B subunits form a five-membered ring that binds to GM1
gangliosides on the surface of the intestinal epithelium cells. The A1
portion of the A subunit is an enzyme that ADP-ribosylates G proteins,
while the A2 chain fits into the central pore of the B subunit ring. Upon
binding, the complex is taken into the cell via receptor-mediated
endocytosis. Once inside the cell, the disulfide bond is reduced, and
the A1 subunit is freed to bind with a human partner protein called
ADP-ribosylation factor 6 (Arf6).[13] Binding exposes its active site,
allowing it to permanently ribosylate the Gs alpha subunit of the
heterotrimeric G protein. This results in constitutive cAMP production,
which in turn leads to secretion of H2O, Na+, K+, Cl, and HCO3 into
the lumen of the small intestine and rapid dehydration. The gene
encoding the cholera toxin was introduced into V. cholerae by
horizontal gene transfer. Virulent strains of V. cholerae carry a variant
of temperate bacteriophage called CTXf or CTX.)

C. is a bilayer structure

A. L5-S1 (OR L4-L5) was in a option


116. Retention of C02:
117. Lymph drainage of Testes:

120. Cardiac disease scenario wad there and they asked about the
fluid accumulation, proteins more than 3.2 g/dl:
A. Exudative
B. Transudative
123. Scenario was given about massive splenomegaly:
A. Leishmanias
B. Chornic Malaria
(Both does so choose da option on ur own risk :)
124. Repeat mcq from asim regarding gray matter foot processes:
A. Protoplasmic Astrocytes
125. Most important renal tubular buffer is:
A. hB
B. hC03
C. Protein
D. Phosphate
126. Cause of global blindness:
A. Herpes SimplexB. RubellaC. Chlamydia Trochamatis
127. Which is normal HB:
A. HbAB. HbFC. HbC
128. Thalesemia carrier percentage in Pakistan:
A. 5%B. 10%C. 15%

111. Two point discrimination: (repeat mcq)


A. ScapulaB. LipsC. Palms

129. Surgery for removal of ovary done which is damaged:


A. Internal Iliac ArteryB. External Iliac ArteryC. Uterine Artery

112. Spread of renal tumor will be through:


A. Renal Vein

130. In respiratory Alkalosis renal compensation is:


A. NaHC03 excretion

113. About ankle joint:


A. Supplied by superficial artery
B. Supplied by superficial paroneal nerve
C. Fibrocartilage on hialine surfaces
D. lateral mallaeolar ligament on medial side
E. Deltoid is on malleolar side

About 10 or 15 repeated. so do Asim n Shoaib well!

There were questions about:

Cruciate Anastamosis

Medical Geniculate Nucleus

Resp Alkalosis n compensation

Internal Thoracic Artery

Cervical Vertebra osteology

Pituitary Tumour

Foreign body in Supine Position which lobe involved

Meningitis type values were given

Development of brain which give rise to which

5th January 2015 Medicine and allie


what iss appropriate about denticultae ligament a. join dura with
vertebra b. join adjacent vertebra c. extension of dura d.
extension of filumterminale
2.
after hepatectomy what will help in regeneration in initial stages
a. IL6 b. endothelial growth factor c. hepatic growth factor aurbhe
options theendont remember
3. red purple nodules on arm oraalthrushhdiarrheaa cause of it a.
hhv 8 ( kaposi ) dontremmber other options
4. typical kallmann syndrome aya... ididnt see other options and
marked Y chromosome with kLgi gene coz drSalahuddin
Kamalsir said that its from old ganong
5. which involves in histamine release a. morphine b. tramadol
6.
best drug for gonorrhea a. cipro b. cefo something bholgae
7.
h/k blocker a. omperazole b. cimetidine
8. secondary center of oss. a. epiphysis b. epi plus epiphysial plate
9.
taking warfarin what shouldnt be taken together a. cimetidine
10. metaplasia a. functional change
11. diagastric has two belly between these two belly central tendon is
attached to a. hyoid b. styloid

12. 12. true about thymus a. lie on sternohoidsternothyroid muscles


b. arch of aorta is in front of it c. lie in front of thyroid d. extend
from lower border of thyroid to forth costal cartilage e. single
lobed
13. coronary blood flow is maintained by a. adenosine b. NO
14. 14. about calcitonin a. cannot be given in pain of padget b. will
be helpful in vitaminosis d c. absorbs ca from intestine
15. 15. conjugated bilirubin raised a. wilson b. hepatitis c. hemolytic
anemia d. crigler e. gilbert
16. 16. when will left ventricular pressure will be highest with normal
pressure a. isovolumetric contraction
17. 17. crescent in kidney elm specimen formed by a. fibrin (repeat
of few days back paper )
18. 18. paracetamol a. poor anti inflammatory b. will not effect the
kidney
19. 19. class three anti arrythmic a. lidocaine b. quinidine c.
amiodaron
20. 20. ElM granular appearance .....a. post streptococcal

21. glomerulonephritis. b. membranous


22. 21. in epidydimus non differentiated cells at the BASE a.
spermatogonia A b. spermatogonia B c. sertoli cell d. spermatids
23. 22. pt with HIV says you are not to tell anyone a. u will keeo
treating b. call the police c. will persuade him to tell her wife d.
say him to use contraception e. will u go with his will
24. 23. group with disease risk factors compared with control a. case
control b. cohart
25. 24. pt weight gain cold intolerance case of hashimotothyroditis
best investigations a. T3 T4 TSH b. TSH c.anti microsomal
antibodies and antithyroglobins antibodies
26. 25. hashimotothyroditis is a. type 1 hypersensitivity 2. type 2 3.
type 4
27. 26. pt having HIV and having infection a. due to decrease in
helper t cell decrease b. decrease cd 8 cells
28. 27. pt with sore throat for six weeks ... generalized
lymphadenopathy biopsy shows diffuse polyclonal hyperplasia a.
infectious mononucleasis b. HiV c. T cell lymphoma d. hodgkin
lymphoma
29. 28. you working in ICU what thing you need to do to prevent
iatrogenic infections a. wear glooves b. use extensive
handwashing c. wear mask
30. 29. drug for gas gangrene a. penicillin
31. 30. boy having hemarthrosis and driarrhea for last few weeks ...
none of his brother has such problem a. hemophilia b. vit k
deficiency c. hepatitis
32. 31. neonate having inclusion infection of eyes same infection
mother has in her genital canal a. chlamydia
33. 32. which is appropriate a. nipple at T2 b. lateral side of elbow C7
c. costal margin in midclavicular line T8
34. 33. total bilirubin 10 unconjugated 8 conjugated 1 a. hemolytic
anemia is caused by it b. bilirubin will appear in urine c. stone
obstruction by it c. it will lead to hepatitis ( very wierd q due to
and lead to strange english
)
35. 34. paraneoplastic syndrome a. RCC b. small cell ca
36. 35. smoker, worker in construction company reles in chest x- ray
showing lower lobe plaques a. berylliosis b. asbestosis c. silicosis
d. anthracosis

37. 36. lambda chains in kidney elm a. membranoproliferative type 2


b. membraneoproliferative type 1
38. 37. most abundant immunoglobulin a. IgG
39. 38. pubic bones together by A. symphasis
40. 39. which is secondary cartilagenous joint a. pubic symphasis
41. 40. tissue injury is due to which amino acid a. glycine b. choline
42. 41. women having non productive cough continoously went to
vacations and remained ok there for one month and then came
back again to take care of her pets and the disease came back x
ray normal .... a. histamine degranulation
43. 42. CRL at 17- 20 wks a. 13 to 17cm b. 20 to 25cm
44. 43.
DIC started with a. tissue thromboplastin
45. which factor def will lead to bleeding a. factor 5 b. factor 12
46. 45. pt came with cardiogenic shock how would u treat him a.
adrenaline b. dopamine
47. 46. circadian rythum is controlled by a. suprachiasmatic nucleus
48. 47. action of angiotensin 2 a. vasoconstriction b. na absorption
49. 48. what angiotensin 3 will do a. aldosterone secretions
50. 49. RBC production will be coz of a. erythropoietin b. Growth
hormone c. growth factors
51. 50. hypoxix hypoxia wala typical question
52. 51. strangestatment pulse 120 bp 120/50 what respone will help
in overall survival of body a. cerebral ischemic respone 2.
generalized sympathetic activity
53. 52. patient having pain in chin area he took tegrel 200 he didnt
get ok, he went to physician . physician saw eruption along
trigeminal nerve a. herpes zoster b. trigeminal neuralgia
54. 53. if main somatosensory area along with all terminal
destroyted what will be effected a. dorsal colume and trigeminal
nucleus b. dorsal colume and spinal area and trigeminal (
issitarha kay mixed options theen )
55. 54. what is true about forth part of duodenum a. completely
retroperitoneum b. mesentry starts at duodenojeujinal junction
56. 55. destruction of right optic track a. left homo hemi
57. 56. optic tract goes to a. lateral geniculate
58. 57. which are parasympathetic nerves a. 3, 7, 9, 10
59. 58. max absorption of which substance a. urea b. glucose

1.

60. 59. what is present in umbilical cord at birth a. two umbilical


arteries b. two umbilical veins c. allantions d. ductus venosus
61. 60. which virus causes ca a. retinoblastoma b. small cell ca c.
prostatic ca d. colonic ca
62. 61. digoxin is recommanded for a. atrial fibrillation b. heart
blocks
63. 62. with cold , cold recptors are stimulated when body will be
cold which one will be stimulated a. nerve free endings b. krause
bulbs
64. 63. what will be the epithelium of pre- terminal bronchioles a.
pseudostrtifiedcolumner b. simple cuboidal
65. 64. what is true about pseudocolumnar epithelium a. all cells are
at base b. all cells reaches apex c all have nuclei at same level
66. 65. supracondylar fracture pt unable to extend her lateral fingers
and sensorey loss of lateral two third of palm nerve damage a.
median b. radial
67. 66. right coronory artery arises from aortic sinus and it will give
branch Posterior ventricular waht is true abt it a. two in number b.
Anaesthesia MCQs Dr.Asad Senior Doctor(FCPS,MCPS)
Q1 Which of the following anaesthetic agents can induce
epilepsy? A. Sevoflurane B. Isoflurane C. Desflurane D.
Methoxyflurane Correct answer: A. Sevoflurane

Q2 Which of the following is not a definitive airway? A.


Tracheostomy B. Orotracheal tube C. Nasotracheal tube D.
Laryngeal mask airway Correct answer : D. Laryngeal mask
airway
Q3 naesthetic agent contraindicated in hypertensives is? A.
Propofol B. Thiopentone sodium C. Diazepam D. Ketamine
Correct answer : D. Ketamine
Q4 ontraindications to vasoconstrictors in local
anaesthesia? A. Spinal anaesthesia B. Epidural
anaesthesia C. Digital nerve block D. Regional anaesthesia
Correct answer : C. Digital nerve block
Q5 Which of the following is correct regarding colour coding
of anaesthetic gases? A. Oxygen Black with grey
shoulders B. Nitric oxide Blue C. Cyclopropane Red D.
Helium Orange Correct answer : B. Nitric oxide Blue
Q6 Which anaesthetic agent is preferred in a patient taken
for surgery with bilirubin of 8.6 mg/dl and serum creatinine of
2.1 mg/dl ? A. Atracurium B. Vecuronium
C. Pancuronium D. Rocuronium Correct answer : A.
Atracurium
Q7 A Lower Segment Caesarean section (LSCS) can be
carried out under all the following techniques of anaesthesia
except: A. General anaesthesia B. Spinal anaesthesia C.
Caudal anaesthesia D. Combined Spinal Epidural
anaesthesia Correct answer : C. Caudal anaesthesia
Q8 The following modes of ventilation may be used for
weaning off patients from mechanical ventilation except: A.
Controlled Mechanical ventilation (CMV) B. Synchronized
intermittent mandatory ventilation (SIMV) C. Pressure
support ventilation (PSV) D. Assist-control ventilation (ACV)
Correct answer : A. Controlled Mechanical ventilation (CMV)
Q9 The most appropriate circuit for ventilating spontaneously
breathing infant during anaesthesia A. Jackson Rees
modification of Ayre T piece B. Mapleson A or MgiIls circuit
C. Mapleson C or Waters to and fro canister D. Bains circuit
Correct answer : A. Jackson Rees modification of Ayre T
piece

three in number c. supply to anterior aspect of right atria d.


supply to pulmonary conus
68. 67. trachea surrounded by a. pretrachealfasica
69. 68. physiological dead space a. reduce in excercise b. increase
in lung injury
70. 69. inguinal canal a. absent at birth b. formed by conjoint tendon
c. from ant sup iliac spine to pubic tubercle
71. 70. How many bronchopulmary segments are in upper middle
and lower lobe of right lung respectively a. 3, 2, 5
72. 71. In which congenital anomaly pul trunk and aorta will remain
attached a. Trunchus arteriosus
73. 72. Inversion and eversion takes place at a. Subtalar joint
74. Eversion is done by a. Peronouslongus
75. 74. Horner syndrome a. Mydriasis b. Ptosis
76. 75. What is true a. Cricoid is half ring b. Cricoid attached to vocal
cords c. Cricoid attached to phyrangeal?\laryngeal? Muscles
77. 76. Appropriate about cerebellum a. Purkinji present only in
cortex b. Present throughout cerebellum
Q10 The laryngeal mask airway used for securing the airway
of a patient in all of the following conditions except: A. In a
difficult intubation B. In cardiopulmonary resuscitation C. In a
child undergoing an elective/routine eye surgery D. ln a
patient with a large tumour in the oral cavity Correct answer
: D. ln a patient with a large tumour in the oral cavity
Q12 The following are used for treatment of postoperative
nausea and vomiting following squint surgery in children
except: A. Ketamine B. Ondansetron C. Propofol D.
Dexamethasone Correct answer : A. Ketamine
Q13 All of the following are the disadvantages of anesthetic
ether, except:
A. Induction is slow B. Irritant nature of ether increases
salivary and bronchial secretions C. Cautery cannot be used
D. Affects blood pressure and is liable to produce
arrhythmias Correct answer : D. Affects blood pressure and
is liable to produce arrhythmias
Q14 Which one of the following anaesthetic agents causes a
rise in the Intracranial pressure: A. Sevoflurane B.
Thiopentone sodium C. Lignocaine D. Propofol Correct
answer : A. Sevoflurane
Q15 Which of the following agents is not used to provide
induced hypotension during surgery? A. Sodium
nitroprusside B. Hydralazine C. Mephentermine D. Esmolol
Correct answer : C. Mephentermine
Q16 Which of the following agents is used for the treatment
of postoperative shivering? A. Thiopentone B.
Suxamethonium C. Atropine D. Pethidine Correct answer :
D. Pethidine
Q17 Which of the following intravenous induction agents is
the most suitable for day care surgery? A. Morphine B.
Ketamine C. Propofol D. Diazepam Correct answer : C.
Propofol Q18 Which of the following anesthetic agents does
not trigger malignant hyperthermia? A. Halothane B.
Isoflurane C. Suxamethonium D. Thiopentone Correct
answer : D. Thiopentone
Q19 Which of the following statements is not true about
etomidate?
A. It is an intravenous anesthetic B. It precipitates
coronary insufficiency C. It inhibits cortisol
synthesis D. It causes pain at site of injection
Correct answer : B. It precipitates coronary
insufficiency
B.
Q20 Which one of the following agents sensitizes the
myocardium to catecholamines? A. Isoflurane B. Ether C.
Halothane D. Propofol Correct answer : C. Halothane

Q21 Which of the following inhalational agents is the


induction agent of choice in children: A. Methoxyflurane B.
Sevoflurane C. Desflurane D. Isoflurane Correct answer : B.
Sevoflurane
Q22 Which one of the following local anesthetics belongs to
the ester group? A. Procaine B. Bupivacaine C. Lignocaine
D. Mepivacaine Correct answer : A. Procaine
Q23 Which one of the following antibacterials should not be
used with dtubocurarine? A Norfloxacin B Streptomycin C.
Doxycycline D Cefotaxime Correct Answer B. Streptomycin
Q24 Which of the following is the muscle relaxant of choice
in renal failure? A. Rapacurium B. Pancuronium C
Atracurium D. Rocuronium Correct answer C Atracurium
Q25 Which one of the following muscle relaxant has the
maximum duration of action? A. Atracurium B. Vecuronium
C. Rocuronium D. Doxacurium Correct answer : D.
Doxacurium
Q26 Which of the following is not an indication for
endotracheal intubation? A. Maintenance of a patent airway
B. To provide positive pressure ventilation C. Pulmonary
toilet D. Pneumothorax Correct answer : D. Pneumothorax
Q27 Which one of the following is the description used for
the term allodynia during pain management? a) Absence of
pain perception b) Complete lack of pain sensation c)
Unpleasant sensation with or without a stimulus d)
Perception of an ordinarily non noxious sitmulus as severe
pain. Correct answer : d) Perception of an ordinarily non
noxious sitmulus as severe pain
Q28 Which side effect is commonly seen with fentanyl?
a.Chest wall rigidity b.Tachycardia c.Pain in abdomen
d.Hypertension Correct answer : a) Chest wall rigidly
Q29 Which of the following drugs is contraindicated in a
patient with raised intracranial pressure? a) Thiopentone b)
Propofol c) Midazolarn d) Ketamine Correct answer : d)
Ketamine
Q30 The following anaesthetic drug causes pain on
intravenous adminstration: a) Midazolam b) Propofol c)
Ketamine d) Thiopentone sodium Correct answer : b)
Propofol
Q31 Which of the following fluorinated anesthetics corrodes
metal in vaporizers and breathing systems? a) Sevoflurane
b) Enflurane c) lsoflurane d) Halothane
Correct answer : d) Halothane (Factual question)
Q32 Which of the following is the neuromuscular blocking
agent with the shortest onset of action? a) Mivacurium b)
Vecuronium c) Rapacuronium d) Succinylcholine Correct
answer : d) Succinylcholine
Q33 In general, the last muscle to be rendered akinetic with
a retrobulbar anesthetic block is: a) Superior rectus b)
Superior oblique c) Inferior oblique d) Levator palpebral
superioris Correct answer : b) Superior oblique

Q34 A 20-year old patient presented with early pregnancy


was admitted for Medical Termination of Pregnancy (MTP) in
day care facility. What will be the anesthetic induction agent
of choice? a) Thiopentone b) Ketamine c) Propofol d)
Diazepam Correct answer : c) Propofol
Q35 Which of the following produces dissociative
anaesthesia a) Ketamine b) Propofol c) Thiopentone d)
Enflurane Correct answer : a) Ketamine
Q36 Which of the following inhalational agents has the
minimum blood gas solubility coefficient? a) Isoflurane b)
Sevoflurane c) Desflurane d) Nitrous oxide Correct answer :
c) Desflurane (factual question)
Q37 Which of the following in anaesthesia will produce
decreased EEG activities a) Hypothermia b) Early hypoxia c)
Ketamine d) N2O
Correct answer : a) Hypothermia Q38 Which of the
following is not true about Xenon anaesthesia a) Non
explosive b) Minimal cardiovascular side effects c) Slow
induction and slow recovery d) Low blood gas solubility
Correct answer : c) Slow induction and slow recovery
Q39 Merits of nasotracheal intubation is a) Good oral
hygiene b) Less infection c) Less mucosal damage and
bleeding d) More movement or displacement of endotracheal
tube Correct answer : a) Good oral hygiene
Q40 Most common nerve used for monitoring during
anaesthesia a) Ulnar nerve b) Facial nerve c) Radial nerve
d) Median nerve Correct answer : a) Ulnar nerve
Q41 All are seen in Malignant hyperthermia except : a)
Bradycardia b) Hyperkalemia c) Metabolic acidosis d)
Hypertension Correct answer : a) Bradycardia Q42 Side
effects of oxygen therapy are all except : a) Absorption
atelactasis b) Increased pulmonary compliance c)
Decreased vital capacity d) Endothelial damage Correct
answer : b) Increased pulmonary compliance
Q43 Muscle relaxant of choice in hepatic and renal failure a)
Cisatracurium b) Rocuronium c) Vecuronium d)
Rapacuronium Correct answer : a) Cisatracurium
Q44
A young boy undergoes eye surgery under day care
anesthesia with succinyl choline and propofol and sfter 8
hours he starts walking and develops muscle pain. What is
the likely cause? A. Early mobilisation B. Due to the effects
of eye surgery C. Succinyl choline D. Propofol Correct
answer : Succinyl choline, Muscle pain is a common adverse
effect
Q45 Which of the following is not an amide linked local
anaesthetic? A. Lignocaine B. Procaine C. Bupivacaine D.
Dibucaine Correct answer : Procaine, It is an ester linked
anesthetic

1..Development from 4th n 6 th arch


a..thyroid.b..uppr part of tracheac.cricoid
GYNAE N OBS PAPER 4TH FEBUARY 2015

2..Shortest half life


Aldosteron,..20minThyroxinTestosteronegnrh2minOxytoxin..1 t0
5min

3.. A man with 70 cardiac output 60mmhg po2 8mg/dl hb


o2 bind1.21ml/l
2..10ml/l
3.15l/lt

20.. parathyroid develops frm


A.not frm any pharyngeal pouch..??B.frm pouch 1C.pouch 2D.pouch
3E.pouch4
21.. Hasselbach triangle related to???Direct inguinal hernia

4.. 1st line of defence in malignant melanoma excision


a macrophagesb cytotoxic T cellsc..natural killer cells
5.. Common cause of infertility
a.failure of ovulation,,,b..lack of gonadotrophinsc.hypothalamic
dysfunction
6.. Baby born edematous aftr 2 days shows signs of renal failure on
examination rounded mass in pelvic area on autopsy cystic mass is
seen
a..failure of mesonephros development
b..failure of metanephros development
c..failure of union of mesonephros n ureteric bud
d..renal a genesis
(As da kidney develops from metanephric mesoderm whch forms
glumeruli nd collecting tubuls and ureteric bud from mesonephric
mesoderm..so da right ans wd b failure of union of metanephric nd
ureteric bud..)
7.. A tall man with normal external genitalia married but had no kids he
had normal ejaculation petiod but semen analysis showed azospermia
cause may b.asim gynae
A epidydymis
B sertoli cellC scrotumD seminiferous tubules8.. Tumour marker of CA
colonA. CEA

22.. lowest oxy concentration in which part of fetal


circulation??svc,ivc,umbilical vein.umbilical artey
23..breast which is inappropriatea..supply by internal thoraxic
artryb..retromemory space devoid of bloodc..related posteriorly by
pectoralis major
d.. axillary tail drains into post. group of lymphnodes
24..MMR
maternal mortality rate
A 5/1000 live birthsb 5/100000 populationc50/10000 live births
25..inferior fascial layer of urogenital diaphragm is...perineal
membrane
26.. pleomorphism increase nuclear to cytoplasmic ratio increase
mitosis anaplasi or keratin pearls
27.. cytokeratin pe b tha..lmbu stamtement k sath
28.. Hypocalcemia in CRF
A. Decreased erythopoitin production
29..which one is secreted after sweating

9..Another name o interureteric ridge is?

a..incraes adhb..increas na,,,c..decreas water

Rice's ridgemercier?? Ridge...Gonadal ridge

30.. muscle is stretched,relaxation will occur d/2:


muscle spindle..golgi tendon organ

10.. Preterm labour due to..


a..candidab..chlamydiac..trichom
11. Pt comes in to the emergency department with seizures. His BP is
90/60. His serum calcium level is 6, serum cortical level is 11 microg/dl
, blood sugar level is 110 mg/dl. Serum Sodium level ?, Serum
potassium level ?. What is the diagnosis?
A. Addisonian CrisisB. Cushings diseaesC. Tetany
12.. The carcinoma wch doesnot cause distant mets.
Basal cel ca.DysgerminomaPhyloids tumourSq.cell ca.
13.. Which one act as second messenger in many body reactions
a..adenosine monophosphateb..adeno diphosphatec..adeno
triphosphated..adenyl cyclase
14.. )Not Included In Blood Coagulation Profile
.Bt.Pt. Plttlc..pttanti thrombogin 3
15.. The specific feature of Apoptosis
A cell shrinkingB karyolysis.i did thisC hyperchromasiD cell swrlling
16.. Medullary rays containA. Collecting ducts
17.. Premature delivery related to anomaly
A . foramen ovaleB. Truncus arteriosisc..patent ductus arteriosis
18.. cancer pe tha k etiology does not depend on??
a..ageb..traumac..heredityd..infection
.19.. ONE QUESTION OF AFTER TRAUMA pt speak only few words
WHICH AREA OF BRAIN INVOLVED ?
BROCAsWERNIKEsPYRAMIDIAL GYRUSTEMPORAL REGION

31.. 12yrs old girl..hw to assess age by ossification centre of which part
of humerus
A.medial epiB.trochleaC.capitulum...lateral epi
32.. abt quadrate lobe,true is
A.lies bw ivc and lig venosum
B.functionally part of rt lobe
C.drains by left hepatic duct
33.. Safe mother hood
a..component of reproductiv health
b..related to mother health only
C..related to child n mother health only
d..dec meternal mortality
34.. 4ml of 2% lignocaine vail contain??
80mg
35.. Jugler vein coresponding on ecg
A a wave for atrial depolarization
B a c wave coresponds to pr interval on ecg
36.. primigravida 28 weeks gestation came with puffy face and
hypertensive n hypoalbuminuria
a acute glomerulonephritis
b lipoid nephritis
c..pre eclamsia
37.. epidural anestheisa failed in second stage..n dose of anesthetic
was repeated.. the cause of failure is due to which segment not

undergone anesthesia
t10l1,l2s1,s2.s3,s4
38.. stress hormone not in surgery procedure
a acth..b Adhc Glucagon;;;d Insulin
39.. Occulomotor nerve supplies
A all extta occiler muscles
B trigeminal ganglion
C sup n inf fibers pass thru sup n inf orbital fissures????
D compltely parasympathetic to cilliary muscles
40.. Pain on deffecation..
a..pudendal nerveb..inferior rectal nerve
41.. Heavy smoker small noduls on vocal cord may include all except
A.hyperplasiaB.neoplasiaB.atrophy
42.. Regarding spermatogenesis
A. Starts at birth..B. Capicitation occur in epidydmus
c..take directional movement in seminiferous tubules
43.. Lymphatic drainage of labia majora
B. Superficial inguinal nodes
44.. Parturition caused by
A. Fetal cortisol
45.. Female exposed to bomb blast radiations..whch effected most
A. Haematopoitic tissue
B. Kidney
46.. Possitiv predictive value affected by
a..prevelance of disease
b..incidence of diseas
c..mortality
47.. Patient wid fever,joint pain..receive diphtheria vaccine 2 weeks
ago..which type of hypersenstivity?
A.type1B.type2Ctype3Dtype4
48.. All cause of uti exceptasim gynae
a..ecolib..dudercin bassilusc..tuberculous
49.. Cervical cancer common causea..hpvb.hsvc..chlamydia
50..hsv 2
acervical carcionoma
b..endometrial cac..ovarian ca
51.. Pudendal nerve supply all exceptasim gynae
a..ischiocavernosis..b..levator ani;;;c..post fornix of vagina
52.. While doing LP what structure is damaged
A dura matter
B pia matter
C vertebral ligament
d ant longitudnal ligament
53.. newborn meningitis
a nesseria meningitis.b ecolic cryptococus neoformans
54Following has high clearance value
A inulin..B pahaC glucoseD Na

55.. .para sympathetic stimulation cause ...


1decrease heart rate.
..2.periphral vaso dilation
3.thick and viscus saliva sectetion
56.. Uterus n breast aftr pregnancy regress by
a..lysosomes
b..golgi apratus
c..endoplasmic
57.. A newly married female came with pain in rt illiac fossa her TlC
12000 she has fever n vomiting her lmp was 3 weeks back what is diag
A ac appendix..B salpingitis;;C uterine causeD fallopian tubes
58.. How good a test at corectly diagnosing people with the condition it
is.asim gynae
a..sensitivity..b..specifityc..true positive..d..false positivee..confidens
interval
59.. Sacroiliac joint
a..plane variety of sinovial joint
b..sacral part coverd with hyaline cartilage
c..sacral part coverd with elastic cartilage
60.. Microtubule presents in
a..centrioleb..gollgi apratus.c..mitrocondria
61.. Human pkacenta
a..covered b amnion on its fetal surface
b..is hemochorial
c..is termed as chorioalontoic placenta
d..seperate from uterine wall along plane of stratu spongiosum
e..seperate from uterine wall due to rupture os many uterine vessels
62.. X ray of pregnant women chest
a..irradiate with radiation of 45mil rads
b..can not b dangerous for women if abdomen coverd with lead sheet
63..Cloacal memb failure to proliferate cranialy whaat happen
a...exostrophy of bladder
b..anorectal fistula
c..recto vesical fistula
d..imperforate anus
64.. In quiet inspiration the muscle wch increases the transverse and
ap diameter of thorax is ..
Diaphragm
Ex.intercostals
Innermost intrcostls
Intrnal intrcostal
65.. These was question regarding a man in room temp 21 degree
humidity 80%
How ll heat b lost from body...
Conduction n radiation
Sweating
By prespiration
66.. slow growing tumor of thyroid
papillary ca (ans)
follicular ca
67.. There was one question the end product o f purine ?
Urea
Ammonia
Uric acid (ans)

68..anteroposterior view Right border of heart formed by:


svcRaLv.Ivc+ra

84..upper outer quadrent of breast drain in to


a..axillary lymph nodes

69.. Confidence interval


a..mean n its standard error of mean
b..mode n its standard deviation
c...median n its range
70.. Feco oral rout
Hep EHep bHep cHep d
71.. Steriods act by
a..ip3
B..by activating genes
c..camp
72.. Polyarteritis Nodosa
A. Fibrinoid necrosis
73.. One ques was
Pregnant lady anaemic iron level seen by increase in
:
Cbc.Sr ferritin.Tibc
74.. Rupivacaine preffered to bupuvacaine due to
a..cardiac toxicity ocours is reversible
b..less toxic
c..short duration
75.. most common cause of dysplasia
hpV
Multiparity
76.. Least peneterance
a..lead.b..copper.c..murcur
77.. Diabetic pt wd neuropathy nd foot ulcer
A. Angiopathy nd neuropathy
78.. pelvic ureter NOT
a. behind mesen colon
b terminal part is uterine artery
c..pierced parietal peritoneum before entering bladder
79.. CO 5.6
HR 70
EDV 160 ml
EF????
a 0.40
b. 0.50
c. 0.55

85left ovarian artry arise from


Abdominal aorta
86..anterior relation of kidney
a.pancreasb..left colic flexure..c..leser sac
87..effect of drug on body
Pharmacodynamic
88..aim of drug therapyasim gynae
Non pharmacological methods prefer 1st

89..common cause of hepatocellular ca


a..hep b..b..hep cI mark this
c..both hep b n c.but mostly people mark this
90..cause of hepatocellular ca
a..aflatoxin diet
b..hep c
91..starling s lawganogn mcq
a..shows incresin cardiac out put with incr venous return
b..increas c.o with excersice
92..cardiac output mainly depends
a..on stroke volume..b..venous return
93..basal gangliaa..caudate with corpus striatum..???...b..tegment
tempeni
94..premalignant lesion of vulva
Pagets diseas
95..pain in medial of thigh on medial rotation of thigh
Pelvic appendix

80.. esmolol
not a b seletive beta blocker
short acting???

96..endometrial biopsy on 18th day of lmp shows

81.. Tubular organ nd lymphoid tissue in mucosa nd submucosa


A. Appendix
B. Cecum

97..supra pubic incision.Inf epigastric artry damage

82.. femoral nerve


its root value
L2 L3 L4

99...HLA DR4

83..ovaries lymphatic drainage

100..baby at station 2 during delivery needle pirces

Para aortic nodes

Ant fontanele

a..secretory phase,,,.b..ovulatory ohasec..follicular phase

98..oxytoxic n adh arises from\....Hypothalamusnurohypophysis

Rheumatoid arthritis

Posterior fontanale

dec urine osmolarity

101.lyphoid.structure in mouth coverd with stratified squamous


epithelium

118..breast feeding started


With in half hourWith in hour2 hrs

Palatine tonsilTongue..Palate
119true about spermatogenesisasim gynae
102..true about supra renal gland
Supplied by inf pherenic artry

a..completed in 20 daysb..started at birthc..1 spermatid give four


sperms

103..after c.section there is dribbling of urine thru vagina after 15 days

120..which anti emetic hav maximum bio availability

Vesico vaginal fistula

Metoclopramidebut maximum mark this

104..polycystic kidney .autosomal dominant

Odansetron I mark this

105..bleeding aftr c.section ruptr of

121..local anesthetic effectasim ques

Uterine artry

a..effect change by change in ph

106..bitemporal hemianopia

b..not depend on route of administration

Uppr fibers of chiasm

c..independent of adding epinephrine

Middle fibers of chiasmLower fibers of chiasm


107..superior mesenteric artry thrombus intestine black

122..not transported by plasma proteinsa..o2b..co2..c..bilirubin..i


mark this pta ni brain ghom gia tha but smj ni a rae thi coz sb hi
transport krte

Wet gangrene

123..pregnancy antithyriod drugPropylthiouracil

108..alcohopl inhibits ;;;Adh..Oxytoxin

124in preg gallbladder stones investigation

109..parasympathatic cut down mostly affected

GGTALT

a..git musclesb..heart musclesc..skeletel muscles

125..hb 8 retic 10% red cells decreas ..Hemolytic anemia

110..gas gangrene.Toxic shock

126plasmodium falciparum.Black water fever

111..1st indicator of hypovolumic shock is

127..preg lady on dental chair aftr some time unconscious due to


pressing which structure;;;;;Inferior vena cava

Olygouriam.i.hypotension
112..vitamin b12 deficiency with mouth ulcers n perineal ulcers

128..HELLP SYNDROME.Hemolysis..elivated liver


enzyme..thrombocytopenia

a..apthous ulcers.b..leukemiaI mark this.c..sle

129..not a feature of malnutrition.Thrombocytopenia

113..after gastrectomy which should add in diet

130..female with iud best management.Evacuation of uterusBlood


transfusion.Pltlt transfuse

a..vit b12.b..iron
114..hb 6 dysphagia platlet normal
Iron def anemia
115..spleenomegaly splinter hemorrhages fever
Blood culture
Blood cp

131..reaction of blood transfusion of A POSITIV GROUP


a..A+b..A-..c..AB1321 carbon transfer byFolic ascidBiotinI mark this
133..carboxylation of fatty acids byBiotin
134..essential fatty acidsLinolenic ascid

Urine culture

135..insulin deficiencyIncrease activity of lipoprotein


lipase..Ketogenesis in liver

116..mcq of free redicals not remember

136..glucagon increase by

117..adh causes

ExcersciseGlucose

Contraction of vesels

137..osteogenesis

Decreas plasma osmolarity

Increase growth by increase interstitial growth

2..10ml/l
3.15l/lt

Osteosytes have many nucleiI mark this by tukka


Density increase aftr deposition
138..primary cause of lymphadema
Dec oncotic pressure due to protein dec
Problem with lymphatics
139..acute inflammation exudates
Increase lymphocytes
Protein more than 3gm

150.. Fio2 max dose which does not cause fetolental adrenoplasia:)
0.5
1
2.5
.26
Fraction of inspired oxygen (FiO2) is the fraction or percentage of
oxygen in the space being measured. Medical patients experiencing
difficulty breathing are provided with oxygen-enriched air, which means
a higher-than-atmospheric FiO2. Natural air includes 20.9% oxygen,
which is equivalent toFiO2 of 0.21. Oxygen-enriched air has a
higherFiO2 than 0.21, up to 1.00, which means 100% oxygen. FiO2 is
typically maintained below 0.5 even with mechanical ventilation, to
avoidoxygen toxicity

140..most dilated part of spinal cord

151..verteberal coloum attachment

L2..L4 .Below FORAMEN MAGNUM

a..rectus abdominusb..extensors of thigh.c..external oblique


d..internal oblique

141..medullary rays
By collecting ducts
142..not supplying scalp
Occipital arteries
Maxillary artery
Posterior cerebral
143..pregnancy with antituberculous drugs instead of taking ocps
Ethambutol
Pyrazinamide
Rifampicin
144..epileptic women on antiepileptics become pregnant what should u
advise
Add valproic asid
Change to phenol barbitone
145..highest protein content
HDL
146.. In emergency department
a..physian should discourge the anger of pt n atendents againt
departmentI mark this
b...only pt should allow to come inaftr discussion
c..phycological therapy should b imp for pt and atendents n family
147.. Post synaptic sympathatic mainly secretes
a..nor epinephrine
b...Acetylecholine
c..adrenalline
148.. Prevelence is,,,,a..new cases.b..all cases..c..group
149.. A man with 70 cardiac output 60mmhg po2 8mg/dl hb
o2 bind
1.21ml/l

152..optic chiasm.a..anteriorly related to pituitary.b..drain in dural


venous sinuses
153..barr bodies inappropriate.a..seen in some cases of turner
syndromeb..completely inactive
154a boy with rash sneezing n rhinorhea
Type 1 hypersencitivity
155.. antioxidant vit E
156.. GFR increase by
,decrese resistance of afferent arteriole
Dec resistance of efferent arteriole
157..comon cause of uti in old age
urinary obstruction
indeweling catheters
hosp acquired
158.. resting membrane potiential, due to
K efflux
NA K pump
159.. ECM+cytoskseton,cadherinsIntegrins
160.. wound healing,myofibroblast contractionfibroblast contraction
161..anion gap.difference b/w measured anions n measured cations
162.. -cardiac reserve,
Blood remain in ventricles after max contraction??
163.. mucosal glands absent in,terminal bronchioles
164.. produced by intestinal bacteria,vit K

165.. rubella infection,5th to 10th week


166.. broder's classification,degree of differentiation
167.. -arterial n venous occlusion,velocity of blood
168..post operativ wound greenish pus
,pseudomonas
169.. S1 heart sound,isovolumetric contraction
170.. all germ layers,teratoma
171..median of diff values..27.5
172.. down synd,increased maternal age
173.. tension in muscles , golgi tendon organs
174..relaxation of muscles,golgi tendons
175.. apex of heart,8 cm from midline
176.. -inflammation,macrophages r basic scavengers
177-neutrophills r basic phagocytes in blood
178..decrease helper T cells in AIDS
179..cimetidine,decrease hepatic clearance
180.. ischemia,coagulation necrosis
181..pain on lying , pericardium
182.. definition of FRC
Volume of air remain after tidal volume
183.. -major buffer in blood, Bicarb
184.. loss of ankle jerk,S1
185.. -intrinsic pathway activation,collagen exposure
186.. -local metabolites of myocardium
187-not a basic tissue,blood
188.. ant relation of kidney,diaphragm
189-opsonization,C3b
190.. CRL becomes half of bitemporal diameter , 3rd month
191.. -shift of O2 dissociation curve to right
192thyroid gland,drainage in deep cervical lymph nodes
193.. metaplasia definition

January 1st 2015 by mitral valve..

isthmus attached to cricoid 4) veous drainage in


bracheocephalic vein ans 5) forgot the last option

1.A 31 year old young man has heaviness in his scrotum


for six months. On examination a solid 5cm mass is

#10.A needle when inserted 2cm below and lateral to pubic

palpated. On labs his alpha feto protein level was 81ng/ml

tubercle will be on

while bHCG level was 15 IU/l. Same side orchiectomy was

1. Obturator nerve ans

done and smooth glistening mass was seen. Which of

2. Ilioinguinal nerve

following could be the cause

11.A 52 year old man underwent coronary artery bypass

1. Leydig cell tumor2. Embryonal carcinoma ans3.

graft two weeks ago and now presents with purpuric

Choriocarcinoma4 Teratoma

patches on skin. On labs his PT and aPTT were normal.

#####2.Ligament between dens of axis and process of


atlas
Ant. Longitudinal
Post. Longitudinal
Transverse
Laigament flavum

Following is true
1.

Heparin induced thrombocytopenia


2. Drug induced thrombocytopenia ans

12.Regarding pneumotoxic center following is true


1

3.Most common position of appendix

Increases inspiratory potential 2. Fires dynamic


respiratory drive ans

1. Paracecal

13.About ankl joint

2. Retrocecal ans

1. Supplied by superficial artery

3. Pelvic appendix

2. Supplied by superficial paroneal nerve

##4.Regarding elbow joint

3. Fibrocartilage on hialine surfaces

1.

Capsule deficient posteriorly


2. Covered all arpund by muscles
3. Hinge joint ans
4. Supination occurs here
5. Joimt between humerus, radius, ulna

5.Lymph drainage of lower medial quadrant of breast will


NOT drain into
1. Pectoral nodes ans
2. Medial sternal nodes

4. lateral malaeolar ligament on medial side


5deltoid is in maleolar side ans
#14.Which one combination is correct
1. Thiopental reduces heart rate
2. Profofol increases cerebral blood flow
3. Morphine reduces apnoeic threshold ans
4. Ketamine relaxes bronchial smooth muscles
#15.Medial geniculate body

3. Supraclavicular nodes

1. Receives oculomotor fibers

4. Inf phrenic modes

2. Optic tract fibers

5. Apical nodes

3. Fibers ascend to thalamocortical areas ans


4. Solely to thalamis
5. Progect to frontal lobe

6.Appendicular artery is branch of


Post cecal
Iliocolic ans
7.most suitable nsaid in lactation

#16.Loss of abduction and adduction of fingers and


adduction of thumb with intact skin sensations involves
1. Recurrent branch of median2. Deep branch of ulnar
ans

, brufen ans;;, ketorolac, .sulindac

#17.Heterophile antibody is seen in

8.Which tumor involves extension to nerve sheath

1. Infectious mononucleosisans2. Herpes simplex..3.


Gonorrhea

1. Myxomas
2. Adenomas
3. Hemangiomas

#18.Lesser omentum
1.

Encloses hepatic artery and hepatic duct2.


Does not attach to liver..3. Attaches to second

4. Carcinoma expleomorphic adenoma

part of duodenum4. Is a bilayer ans


#9.what is correct about thyroid gland 1)supplied by 3
paired arteries 2) surrounded by prevertebral fascia 3)

#19.Sensory supply of uterus passes through which


ligament

1. Broad ligament ans..2. Round ligament..3. Cardinal

#30.A sprinter during running had injury to his ankle. He

ligament;;4. Uterosacral ligament..5. Uterocervical ligament

can stand on his toes but with severe pain.. There is a

#20.Amoeba causes lesions in which part of gut


1. Terminal ileum and cecum ans..2. Ascending colon3.

visible ecchymoses around ankle joint


1.

Torn tendoculcaneus2. Torn plantaris tendon ans

Transverse colon

#31.Thrombosis of right coronary atery after right marginal

4. Descending colon and sigmoid

artery has branched off will result in infarction of

5. Sigmoid and rectum

1.

SA node..2. AV bundle..3. Apex of heart4.

#21.Deep to post digastric and near palatoglossus a

Infundibulum of pulmonary artery..5. AV node

structure runs obliquely upwards

ans

1. Facial artery2. Lingual artery ans3. Maxillary artery4.


Sup thyroid

#32.Age estimation at fifth to 12th week is done through


1.

Biparietal diameter

#22.Nucleus of general visceral efferents of tenth nerve to

2. Crown rump length ans

palate muscles is present in

3. Crown heel length

1. Locus ceruleus
2. Tractus solitarius
3. Nucleus ambiguous ans
4. Nucleus raphe
#23.Function exclusively linked with vit E
1. Carbohydrate metabolism
2. Endothelial protection
3. Skin integrity
4. Germinal epithelial protection ans
#24.While a surgery on submandibular gland which has
more chances of injury
1.

Superior thyroid artery


2. Lingual nerve
3 Mandibular nerve
4. Facial nerve ans

@25.Which ligament supports odontoid process of axis on

4. Abdominal circumference
#33.MELAS syndrome
LoflarInclusion..GranulomaCystraggede reddd fibers
#34.Highest number of alpha resceptors are present in
1. Trigone
2. Neck of bladder and proximal urethra ans
3. Penile urethra
4. Membranous urethra
5. External urethral opening
###35.Loss of inversion
1. Paroneus longus
2. Paroneus brevis
3. Flexor digitorum
4. Tibialis posterior ans

atlas

####.A 31 year old young man has heaviness in his

1. Posterior longitudinal ligament2. Anterior longitudinal

scrotum for six months. On examination a solid 5cm mass

ligament3. Tranverse ligament ans4. Ligamentum

is palpated. On labs his alpha feto protein level was

nuchae

81ng/ml while bHCG level was 15 IU/l. Same side

#26.Cardiax myxomas are most common in


1. Left atrium ans2. Right ventricle3. Infundibulum of
pulmonary artery
#27.Otic ganglion is between mandibular nerve and tensor
tympani. It's superior border is formed by
1. Foramen lacerum..2. Foramen ovale ans3. Foramen
rotundum4. Styloid process

orchiectomy was done and smooth glistening mass was


seen. Which of following could be the cause
1. Leydig cell tumor2. Embryonal carcinoma ans3.
Choriocarcinoma4 Teratoma
####37.A 30 year old woman in azad kashmir was injured
mildly in an earthquake that killed many people apart from
her distant uncle. She would most probably be in
1. Social crisis

#28.Inion is

2. Personal crisis

1. Part of pterion..2. Posteroinferior prominent part of

3. Situational crisis ans

occipital bone ans..3. Posterior to frontal bone

4. Developmental crisis

4. Internally to occipital bone

####38.Anterior epithelium of cornea is

#29.Closure of lips involve

1. Stratified squamous keratinized

1. Depressor anguli oris..2. Depressor labii inferioris..3.

2. Simple squamous

Depressor labi superioris4. Orbicularis oris ans5. Lateral

3. Simple columnar

pterygoid

4. Stratified squamous non keratinized ans


5. Simple cuboidal

####39.Incubation period of which disease is longest


1. Measles2. Rubella3. Chicken pox4. Infectious
mononucleosisans5. Mumps
###40.Clavipectoral fascia
A. Part of endothoracic fascia reflecting anteriorly
B. Encloses pectoralis major
C. Encloses pectoralis minor ans
D. Extends superiorly and encloses scalenous anterior
E. Option i forgot
#####41.18 year old girl has fever tenderness in right ileac
fossa. On labs WBC 17000cells/mm3. Appendicectomy

6.glucose transport...carrier mediated


7.50 yr old e lt hoarsense...mediastnal lymph node e Lt
recurrent laryngeal involvmnt
8.daily production of csf... 500 mg/dl
9.group of young men bathing n beech next day develppe
blebs on back shoulder region cause is...mast cell
destruction
10.abt meningitis...
11.for nutritional status measurement ...vitamin
12.renal column...a)arcuate artey b)minor calcyes
c)collecting tubule d) interlobar n interlobular art
13.pt e fluid loss now show metabolic acidosis e
hypokalemia...a)stomach b) colon c)panrease d) duodenum
14.1st cervical process...odontoid process
15.retention of CO2...resp failur

was preformed. Which of the following would be seen on


histology
1. T lymphocytes in wall of appendix
2. Polymorphonuclear neutrophills ans

IMPORTANT POINTS::

3. Glissening mass at apex of appendix


####42.Which of following has highest chances of
developing endometrial cancer

1. Simple hyperplasia
2. Complex hyperplasia without atypia

3. Complex hyperplasia with atypia ans


4. Squamous metaplasia
5. Chronic endometritis
####43.CSF
1. Formed by ependymal cells
2. Formed 500ml/dayans
3. Formed only 30percent by chorioid plexus
4. Arachnoid villi are visible
####44.A young boy with inc appetite weight gain and
sleep problems. On examination he is having small genitals
1. Hypothyroidism2. Pineal tumor ans3.
Craniopharyngioma4. Narcolepsy5. Cushing disease
##########45.Chronic Fungal infection with rhinosinusitis
causing medial erosion and granulomas1. Cryptococcosis2.
Mucormycosis3. Rhinophycomycosis4. Histoplasmosis5.
Blastomycosis6,aspergillus (ans
#######Simpler bcqs about
bladder cancer schistosoma
Cholangioca clinorchis
Pubic symohysis secondary cartilaginous joint
Malignancy metastasis
Exudate
Itp labs
Brachial plexus multiple twisted questions and same for
heart blood supply
VWF scenario
1.pretracheal fascia infection spread to...ant mediastinum
2.pituatry tumour cause... hemianopia of bilateral nasal half
of visual field defect
3.cardiac infection e protein > 3.2 g/ DL...exudative pattern
4.protein required... ascorbic acid
5.n resp alkalosis renal compansation...nahco3 excretion

U need to memorize some of the important


values..like:
Serum
ca,k,cl,na,hco3,phosphorus,magnesium..anionga
p,ph.
ABGs(arterial blood gases):Po2,Pco2,O2
saturation.
Gh,serumosmolarity,urineosmolarity.
BUN,Uricacid,creatinine,albumin,bilirubin(direct,id
irect).
Protein in urine.
Alt,Ast,amylase.cholestrol,glucose.
Esr,pt,apt,fibrinogen,platelet count.
Mch,mchc,mcv.
Hb,reticcount,hct and erythrocyte count.
CSF R/E

.
Important topics:
-Opening in diaphragm.
-Respiratory capacities.
-celiac truk and branches
-type of hyaline cartilage
-cnsrecepters
-epithelium
-pharyngeal pouches,clefts
-epithelium derivatives
-skull foramina
-cytochrome inducers and inhibitors.
-heart blood supply
-page 600 azimshaikh

-dna and rna viruses

Radial nerve:

-bones of hand

-Descends Infront of lateral epicondyle.

-purines and pyrimidines

-supplies extensors of arm and forearm.

-ulcerative colitis and crohns

-lesion causes wrisr drop.

-tumor markers

Axillary nerve:

-type of errorsbiostat

-passes through Quadrangular space.

-reflexes values

-lies against surgical neck of humerus.

-age related type of meningitis

-lesion wil cause paralyzed deltoid and loss of cutaneous


sensation over lower 1/3 of deltoid region.

-cardiac enzymes
-chlorpromazine can cause parkinsonism
-carotid sheath content
-parkinsonism is aggrevated by Haloperidol
-chemorecepter and baroreceptor
-Digoxin toxicity can increase by chlorothiazide
-hormones second messengers
-centraly acting antihypertensive is methyldopa
-T and B cells CD numbers
-Amino acids derivatives

-Serotonin is secreted from carcinoid tumor which is


responsible for sign and symptoms.

-Glycogen storage diseases

FOOT everters and inverters:

-joint classifications.

Inverters:

-cartilage types(hyaline,fibrocartilage,elastic

-tibialis anterior

---examples)

-tibialis posterior

-bones classifications

Everters:

-thirst is stimulated by:

-peroneus longus,brevis and tertius

1. increased ECF osmolarity.

-Granulation tissue in MI appears within 9-10 days and


becomes prominent within 2-3 weeks.

2. decreaded volume of ECF.


Conducting system blood supply:
3. Angiotensin III
-SA node__RCA(sometimes LCA,but mainly RCA)
-Median nerve:
-AV node__RCA
>Descends on Lateral side of axillary and brachial arteries.
-AV bundle__RCA
>Enters palm Behind the flexor retinaculum.
-RBB__LCA
>supplies most of the flexors of the forearm(arm flexors
supplied by musculocutaneous nerve)
>supracondylar fracture cause median nerve injury--------apelike hand deformity

-LBB__RCA+LCA
ECG diagnosis of MI:
A.Anteriorwall(LAD):v1-v4

-Ulnar nerve:
1.anteroseptal__LAD (v1-v2)
>Descends on Medial side of axillary artery and brachial
artery.

2.anteriolateral__LAD or LCX(v4-v6)

>passes behind Medial epicondyle of humerous.

B.Lateralwall(LCX)__I,aVL

>enters palm anterior to the flexor retinaculum.

C.Inferiorwall(RCA)__II,III,aVF

>injury leads to claw deformity.

-posterior interventricular artery accompanies middle


cardiac vein

-corticotropic (basophilic) adenoma__cushing disease


-prolactinoma---acidophil(30%)

-Anterior interventricular artery is accompanied by great


cardiac vein.

-GH adenomaacidophil

_heparin____monitoraptt

-ACTH adenoma---basophil

_warfarin___monitorpt

-TSH adenomabasophil(chromophobe)
*in axilacords of brachial plexus

-on contraction of diaphragm the intrathoracic pressure


decreases.

*base of neck----roots
*posterior triangle of neck---trunk and division
*detoxification of drugs---ESR

_Lateral horn is present in T1-L2


-Boat shaped cystpneumocystis carnii
_Dorsal colums present only above T5
-Epidural __middle meningeal artery
_Large ventral horn is present in
-Subdural__superior cerebral vein
(C5-T1+L2-S2)
TYPHOID fever investigations:
-Eosinophilia present in:
-1stweek____blood culture
hodgkin lymphoma and polyarteritisnodosa.
-2ndweek___antibody/widal
-PGI2__by endothelial cells
-3rdweek___stool
-TXA2__by platelets
-4thweek___urine culture
Blood transfusion induced electrolyeembalance is as
follows:

-vit b12 absorbed from terminal ileum

-hypocalcemia

-middle rectal artery remains in true pelvis

-hyperkalemia

-renal medullary rayscollecting ducts

-hypothermia

-Anxiolytic effect by GABA

-metabolic acidosis

-lipid which does not contain glycerol is sphingomyelin

-left shift of oxy-hem curve

-taste fibers from anterior 2/3 tongue passes thru internal


capsule

*common peroneal nerve injury__loss of everion and


dorsiflexion

-for controlling tachycardia;;

*Tibial nerve injury__loss of inversion and planteflexion.

Amiodarone>lidocaine>procainamide

-pelvic parietal peritoneum is supplied mainly by Obturater


nerve.

-glucose transported mainly by facilitated diffusion

Lymph drainage:
*labia majora and glans penis---superior inguinal L.N
*cervixinternal and external iliac L.N
*prostate__internal iliac L.N
*Urinary bladder__internal and external iliac L.N
-allantois__Urachus__median umblical ligament
-Somatotropic (acidophilic) adenoma__gigantisim

-amino acidsNa cotransport


-sarcomahematogenous
-carcinomathru lympatic
-night terrors happens in NON-REM
-nightmares happens in REM
-glutamate for fast pain fibers
-substance p for slow fibres
-glutamate released by rods and cons

-blood brain barriers made by astrocytes

-increased plasma osmolarity stimulates osmorecepters in


anterior hypothalamaus.

-fungiformsalty and sweet


-foliate---sour taste

-intestinal cells of kajal are the pacemakers of GI smooth


muscles.

-circumvillatebitter taste

-vommiting and swallowing centres is in the medulla

Stretch reflex:

-most colonic water absorption occur in proximal colon

-monosynaptic

-in multiple myeloma,the alkaline phosphatase is not


increases.

-Ia fibers affected


-tight junction in colon
- alphamotorneurons
-leaky junction in small intestine and gall bladder
Inverse muscle stretch reflex:
-cryptsecretion
-Disynaptic
-villiabsorption
-Ibfibres
-ABO incompatability in infants:
-gamma motorneurons
*O motherAorB baby
-Golgi tendon organs
*A motherB or AB baby
-Horners syndrome happens if lesion above T1
*B motherA or AB baby
-in the cerebellum..granule cells is the only excitatory
neurons while all others are inhibitory

-progesteron is reason behind increased temperature


during ovulation

-trouble going downstairs==Trochlearnerve damage


-Fibroadenomanot precursor of CAbelow 25 yr
-trigeminal neuralgiav2+v3
-Fibrocystic changesprecursor fro CA20-50 yr
-Glossopharyngeal nerve supplies carotid body and parotid
gland

-Granulosa cell tumorcall exner bodies

-loss of accommodation causd by damage to midbrain

-Dysgerminomashrilerduval bodies

-lesion to pretectum leads to loss of reaction to liht but


intact accommodation

--in renal failure:


*early stagedilute urine

-liquid dysphagia is due to neuromuscular incordination


*late stage____concentrate urine
-study brain stem lesions very well..for example medial
medullary syndrome..etc
-nuclei which lies beneath the fourth ventricular floor:

-fat absorption occur primarily in jejunum


-oblique fissure extends from T36th costal cartilage on
both lungs

Vestibular,vagal,abducent,hypoglossal.
-groove for subclavian artery is at first rib.
-spinal cord ends:
-azygos vein enters thorax inlet
*in infants:L3
-right middle lobe has 2 bronchopulmonarysegment
*in adults:lower border of L1
-diaghragm is not attached to T 10
-Subarachinoid space ends at (S2-S3)
-Tendon of popletial muscle is intracapsular
-internal capsule---projection fibres
-DIFFICULTY in:
-at the level of superior colliculusocculomotornerve
nucleus-midbrainred nucleus

*flexing knee and thighsartorius

-genu+anterior 2/3 of posterior limb-----------------motor

*flexing knee and extending hipsemitendinous


*flexing hip and extending kneerectus femoris

-Conusmedullaris is pial extension to the coccyx

-narcolepsydefect in hypothalamus

-revise muscles of laryns..narroeing,widening,tensing vocal


cordetc

-U wave in papillary repplarization


Pt had ankle sprain:

-absorption of short chain FFA happens in colon


-if pt can stand on toerupture of plantaris tendon
-osteosarcoma in metaphysis
-if cannot stand on toerupture of achiles tenton
-occulocardiac reflex mediated by trigeminal
Embryoooooo
-thyroid derived from endoderm
-albumin has low molecular weight and high concentration
-amoebic abcess occur in
Cecum>ascending colon>sigmoid >rectum

Week 1-2: formation of zygote, implantation and formation


of bilaminar
Weeks 3-8: Embryological period .Weeks 9-38: Fetal
period .

-somites are present in paraxial mesoderm

1STWEEK.BLASTOCYST IMPLANTATION

-strongest layer in small bowel in circular layer

2NDWEEK..EPIBLAST AND HYPOBLAST

-embryonic primary vesicle is mesencephalon

3RDWEEKECTODERM MESODERM ENDODERM


FORMATION
HEART DEVELOPMENT START

Lymph drainage of

4TH WEEK..

-lateral handinfraclavicular

At the beginning of the 4th week, the somites (4)


are well formed and the neural tube is also
formed but it is opened at the rostral and caudal
neuropores .

Upper limb buds become recognizable during


week 4 and the lower limb buds become
present by the end of week 4 (day 28 ).The
patterning of the limb development is regulated
by Homeobox-containing (Hox) genes.

The upper limb buds appear low on the embryo


due to the dominant development of the head
and neck.

The upper limb buds form opposite the caudal


cervical segments and lower limb buds form
opposite the lumbar and upper sacral segments.

STOMACH AND FORGUT ORGAN FORMED

PRONEPHROS DEVELOP EARLIER AND


MESONEPHROS DEVELOP LATE.

PRIMODIAL GERM CELL IN 4TH WEEK

-liver makesfactors 1,2,7,9,10 but not 4(calcium)

RESPIRATORY DIVERTICULUM FORMED

-connections of cerebulum

PHARNGEAL APPARATUS FORMED

*cerebellum to midbrain through superior cerebellar


peduncle<SCP>

OTIC PLACODE FORMED

---medial handepitrochlear
-lateral ventricle roof formed by body of corpus callosum
-dermatome of perineal region is S3S4
-CVAheart sounds in JVP
C wave first sound
V wave 2nd
A wave 4th

-in cell cycle:


Replicationinterphase
Divideprohase
A lign 2 chromosomesmetaphase
-noradrenaline and serotonin metabolized by MAO
-adrenaline by COMT

*cerebellum to pons through MCP<middle>

5TH WEEK

*cerebellum to medulla through ICP<inferior>

METANEPHROS BEGIN TO WORK

GONADS PRONEPHROS REGRESS

Weeks 34-38

DEVELOPS

Secondary ossification centers appear in the epiphyses .


The first ones to appear are in the distal end of the femur
and the proximal end of the tibia, at the knee joint.

TRACHEOESOPHEGEAL FOLD FORMED


BRONCHIAL BUD ENLARGE
Bones appear during week 5 as mesenchymal
condensations in the limb buds .

The epiphyseal cartilage plate intervenes between the


diaphysis and epiphysis. When it is replaced around age
25, growth of the bones ends.
1-6 WEEK.EMBRYO REMAINS UNDIFFERENTIATED

Upper limbs show regional differentiation with developing


hand plates

7WEEK.BEGINS TO DIFFERENTIATE

6TH WEEK

12WEEK.MALE AND FEMALE EXTERNAL GENITALIA


RECOGNIZED

MIDGUT HERNIATE THROUGH UMBLICAL CORD


.

20WEEK.PHENOTYPIC DIFFERENTIATION COMPLETE.

PRIMODIAL GERM CELL MIGRATES INTO GONADS


TRACHEA AND ESOPHAGUS SEPERATES FROM EACH
OTHER

FOR LUNGSSS;
7-16 WEEK.PSUDOGLANDULAR PERIOD
16-24WEEK.CANALICULAR PERIOD

Mesenchymal models of the bones in the limbs undergo


chondrification to form hyaline cartilage.
The clavicle develops by intramembranous ossification and
later develops articular cartilages.

24 WEEK TO BIRTH.TERMINAL SAC PERIOD


32WEEK-8YEAR.ALVEOLAR PERIOD
OPTIC VESICLE APPEAR.22DAY

7TH WEEK;
FCPS Part 1 Gynae/Obs Questions (Feb 12th, 2014)
LIMB ROTATIONS BEGINS
8TH

WEEK.

Week 8 (Last week of embryonic life;


At the beginning of week 8,

The digits of the hand are short and webbed.

Notches develop between the digital rays of the


feet.

At the end of week 8, there are distinct regions in the limbs,


with long fingers and distinct toes.
Weeks 9-12

The fetus has short legs and small thighs at the


beginning of week 9.

By the end of week 12, the upper limbs have


reached their final relative length but the lower
limbs have not.

Primary ossification centers are present in all


long bones

Order of ossification: Clavicle, femora, etc...

1Turner syndrome karyotype; a.


47 XO b. 44
XXYc.
44 XYY
2.Glucose tolerance (OGTT) test was done on a lady who
is fasting. RBS 122,after 1hr 198 after 2hr 194. Whats your
diagnosis;a.
Diabetes mellitus b.
Abormal
glucose tolerance c.
Abnormal glucose tolerance
3.Langerhan giant cells are found in a. Tuberculosis
b.
Sarcoidosis c.
Wegners granulomatosis d.
Syphlis
4.Bp 180/110, with headache, favourable treatment;
a. Methyl dopa b.
Hydralazine
5.Factor that delays wound healing locally is;
a. Foreign body b. Infection
6.Which one is not an epithelial tumor;
a. Adenocarcinoma b. Squamous Cell Carcinoma
c. Liposarcoma
7.Atonic bladder caused by
a. Parasympathetic supply cut down b.
Somatic
efferent
8.Fibrous dysplasia more common in;
a. Temporal b.
Maxillary
c.
Sphenoid d.
Frontal
9.True about External Carotida.
Crossed by lingual
artery and divi des into facial and temporal terminal b.
Lateral to retromandibular vein
10.
Microtubules found in a.
Centriole
b.
Cell membrane c. RER
11.Local anesthesia crosses the placenta by; a. Diffusion
b.
Facilitaed diffusion c.
Active transport d.
Bulk flow e.
Pinocytosis
12.Ionisng radiation in bomb blast causes;
a. Aplastic anemia b. Iron deficiency anemia
c. Megaloblstc anemia
13.Gram -ve infection causes septic shock in which of the
following tracts;
a. Biliray tract b.
Colon c. Smal bowel
d.

Urinary tract
14.Epidural block 3% chlorpromazine is more rapid than
2% lidocaine. Reason
a. Lipid solubility b. Ester vs amide c. PKa d.
Concentrati
15.Non carcinogen
a. Methylsulfate b.
Radiation
c.
Cyclophosphamide d.
Alcohol
16.Post op wound causes greenish pus discharge . Which
organism is involved;
a. Pseudomonas b. Klebsiella c.
E. Coli
17.Abortion performed by dai, hematoma formed by
a. Ovarian Artery b. Ovarian venous plexus
c.
Uterine Artery d. Broad ligament
18.Atonic bladder is caused by; a.
Parasympathetic
cut down supply b. Somatic efferent c. visceral afferent
19.Characteristic of Graves disease;
a. TSH decrease
b. T4 decrease
c. T3 decrease
d. Increase in the above
20.Insulin is inhibited by;
a. Glucagon
b. Beta adrenergic blockers
c. Acetylcholine
21.Compression of S1 nerve root results in ;
a. Loss of ankle jerk
b. Postive Babinski sign
c. Sensory loss on sole and medial aspect of foot
22.Uterus remains in position by;
a. Broad ligamen
b. Transverse cervical ligament
c. Urogenital diaphragm
23.Reaction after diphtheria vaccine;
a.
Anaphylaxi b.
Type 1 c. Type2
d.
Type3 e. Type4
24.Baby after delivery;
a. RBC increase in number
b. lung compliance increases
c. Billirubin level increases in 7-14days after birthd.
Weight increases in 1st week
25.A 50 yr old smokers works in a tyre factory, diagnosed
as transtional cell CA, what is the cause of this disease;
a. Aromatic amines
b. Smoking
c. Nitrosomines
26.Regarding Breast the following is not true;
a. coopers ligament attaches it firmly to the
underlying chest wall b.
extends from mid
sternum to midclavicular line
27.Uterus prolapsed but anus in position which
intactligament is intact;
a. Cardinal ligament b.
Broad Ligament c.
Pelvic diaphragm d.
Round
ligament e.
Outer anal sphyncter
28.Percentage can be calculated as
a. By dividing the number by the total number of
items and multiplying by 100
29.After the birth of a baby the perinatal mortality rate and
Still birth can be calculated in;
a. Death within 1 mon b.
1 yr
c. 1 week d.
1 day
30.10% Formaldehyde is used for sending Biopsy
specimen, Why? a. Sterilisation b.
To
prevent
autolysis c.
To stain cells d. To
denature
proteins e.To prevent autolysis of cell
31.Anal canal extent; a.
Between levator anni and
Anus
32.Collagen; a.
Formed by chondroblast
b. Present in all types of connective tissue
33.Stroke Volume increase by;
a. Increase in HR at constant Cardiac Output
b. No change
34.Growth hormone secretion is increased ;
a. Exercise b.
Hperglycemia
35.Severe transfusion reaction occurs when A+ blood is

given to ;
a. A- b.
AB- c. O36.Hormonal therapy against malignant condition acts by;
a. Apoptosis b.
Degeneration
c.
liquefactive necrosis
37.After 48 hours starvation, the source of energy for the
body is;
a. Muscle glycogen
b. Muscle protein
c. Liver glycogen
d. Triglycerides
38.The following is true about acute Inflammation;
a. Local response to injury
b. Systemic response to injury
39.Pressure in aorta least during;
a. T wave
b. Isovolumic relax
c. Isovolume contraction
40.A woman with chest pain and history of fever, that
becomes worse on lying down
a. Myocarditis
b. Pericarditis
c. Pleuritis
41.Regarding septic shock does not include;
a. high dose steroids get benefit
42.Local anesthesia crosses placenta through;
a. Active transport
b. Bulk fl0w
c. Facilitated diffusion
d. Passive diffusion
43.Edema is caused by;
a. hydrostatic pressure
b. osmotic pressure
c. Lymphatic obstruction
44.Preganglionic fibers release ;
a. Acetylcholine b.
Norepinephrine
c.
Dopamine
45.Clinically GFR is measured by;
a. Creatinine b.
Inulin
46.During 2nd week which is appropriate test for typhoid
fever ;a. Widal+Blood culture b.
Blood c. culture
only d. Stool culture e.
Bone marrow
47.Stilbestrol in first trimester of pregnancy;a.
Vaginal
cyst b. Clear cell carcinoma of vagina
48.Glucose transport across a membrane is directly
proportional to concentration graddient . Type of transport
is a.
Simple diffusion b. Facilitated Diffusion
c.
Active Diffusion d. Na co-transport
49.Barr bodies;
a. Always inactive b. Heterochromatin c. Have Xchromosome d.
It can be seen under light
microscope
50.Hormonal therapy causes regression of tumour by;
a. Atropy b. Degeneration c. Apoptosis
51.In neonate the spinal cord terminates at the level of;
a. L1 b.
L2 c.
L3 d.
L4 e.
T12
52.Turner Syndrome a.
45XO
53.Klienfelter Syndrome a. 47XXY
54.Which antib0tc safe in pregnancy in 8th month
a. Tetracyclin b.
Augmentin
c.
Coamoxiclave d. Nitrofurantoin
e.
Cotrimaxazole
55.Amniocentesis vs trans-cervical chorionic villous
sampaling before 14 weeks
a. 1% Decrease chance of miscarriage
b. 10% increase risk of fetal talipes
56.Amniocentesis done in a pregnant lady at term
pregnancy and the baby's head at -2 station. The needle
will pierce;
a. Anterior fontenelle
b. Posterior fontenelle
c. Sagital suture
57.After 48 hr of starvation, wht will be broken down to
provide energy; a. Carbohydrate b. Fat
58.
45 yr female patient presented with hirsutism on
the body and is shy to show her hands to the physician.

Whats the cause;


a.
Adrenal tumor
b.
Congenital adrenal hyperplasia
c.
Polycystic Ovaries
d.
Drug induced
59.Which tumor is derived from all the 3 germ layers;
a.
Adenoma
b.
Teratoma
c.
Rhabdomyosarcoma
d.
Chancroid
60.A 20% patients received HRT, 40% did not. The best
way to present such a data is;
a.
Pie chart
61.Least site of tumor metastasis
a.
Spleen
b.
Liver
62.The enzyme most sensitive to the damange of heart
muscles after MI is;
a.Trop T
b.
LDH c. Myoglobolin
63.
A scenerio of normocytic hypochromic anemia
was there. Values of PCV, MCH and MCHC were given.
(paper 1)
64.Regarding aminocentesis;
a.
Risk inc of talipes if performed before 14 weeks
b.
Perforation of placenta
65.Hamartoma is;
a.
Totally benign
66.A man died 5 days after MI and large blood clots was
found in pericardial sac . He died due to ;
a.
Pericardial tamponade b.
Viral infection
c.
Kawasaki disease
67.The blood supply of right adrenal gland is;
a.
IVC b. Right renal artery c.
Superior
Vena cava
68.A patient taking oral contraceptive pills and
antitubercular drugs and still got pregnant the drug
responsible?
a.
Rifampicin b.
Isoniazid
69.A scenario about Odd's ratio was given. We had to
calculate the Odds ratio
70.Criteria for screening test in diagn0sis of cancer
a.
It should be 100 % sensitive and specific
b.
Early diagnosis cures early c.
It should
be acceptable to the general population
71.Which of folowing causes least respiratory depression;
a.
Fantany b.
Ketamin
c.
Midazola d.
Propofo
e.
Thiopentene
72.A 20yr old male has dyspnea on lying down. The
structure most likely to cause this is;
a.
Enlarged isthmus of thyroid b.
Puberty
c.
Retrosternal goiter
73.The cause of RBC fragility;
a.
Some drugs cause RBC hemolysis
b.
By enzyme deficiency
74.Microtubules
are
present
in;
a.Lysosomesb.
Centrioles
75.Bronchopulmonary segment;
a.
Each has its own blood supply b.
Different
no on both sides
76.Most effective gluconeogenesis is from;
a.
Protien b.
Fats c. Triglycerides
77.Increase in interstitial fluid presure in:
a.
Decrease in capillary permeability
b.
Decreased hydrostatic pressure
c.
Increase in coloid osmotic pressure
d.
Increase in capillary permeabilty
78.30 yr female presented with secondary amenorhea,
diagnosis
of
premature
ovarian
failure
made.
Ultrasonography being normal. Most likely cause:a.
Ovarian tumor b. Autoimun
79.
16 yr child with cola color urine since one day ,
Hb 6 , high grade fever , was given antimalarial 1 day back.
The reason for his black color urine is; a.
G6PD
b.
Falciparum (blackwater fever) c.
PNH
80.Glycolysis is; a. Reversible b.
Hexose phosphate
is converted to lactate anerobicaly and pyruvate aerobicaly

81.A person naked with 80% humidity in room. The mode


of loss of heat from his body is by; a. Radiation
and
conduction
82.Which of the following causes increased interstitial
pressure; a.
capillary permeability b.
colloid
osmotic pressure
83.Jaundice in a pregnant lady. Diagnosis made by
a.
ALT b. AST c. GGT
84.motor supply of SCM
a.
Ascending
pharyngeal
nerve
b.
Occulomotor Nerve
c.
Motor branch of facial Nerve
85.The overdose decreased of which drug can bby
alkalanization of urine to increase its excretion; a Morphin
b.
Phenobarbitone
86.Fast pain fibers; a.
Delta
87.Fibrous dysplasia site a. Temporal b b.
Maxilla
c.
Sphenoid bone
88.Person lying supine. Chest pain increases due to;
a.
Pleura b.
Myocardium
c.
Pericardium
89.1st heart sond is related to which phase of cardiac cycle;
a.
Isovolumatric contraction
90.S1 lesion a.
Ankle jerk loss
91.Person naked in room, temperature 21 degree
centigrade, humidity present, loss of heat by ;
a.
Insensible perspiration b.
Sweating
c.
Conduction and radiation
92.Type of necrosis in brain infarction ;
a.
Coagulationb.
Liquefaction
93.The medial quadrants of breast are supplied by;
a.
Interthorascic nodes b.
Posterior group
c.
Axillary group of nodes
94.The labia majora lymphatic supply;
a.
Vertical group of lymph nodes b.
lateral
group of lymph nodes c.
Superior group of lymph
nodes
95.Severely dehydrated fluid to be given
a.0.9% Nacl
96.During an open surgery of heart the doctor saw a vein
bleed. he vein bleeding most probably may be;
a.
Great cardiac vien b.
Middle cardiac c.
Smal cardic vein
97.Drug action wont be affected if bound to Alpha
glycoprotein ina.
Neoplasm b.
Trauma
c.
Hepatic disease d. Infection
98.Genital tubercle from;
a.
Clioris b. Labia minora c.
Labia majora
99.Fat necrosis occurs in ;
a.
Acute pancreatitis b.
Heart c. Liver
100. Faty acid oxidation is helped by a Biotin b. Vitamin
C
c.
Vitamin D d.
Vitamin B-12
101. Tocoplasmosis is; a.
Helminth b.
Parasite
102. Man runs loses 2 litre sweat , and replaces it with
isotonic plain water of 2 litre, what will happen after the
marathon;
a.
ICF/ECF osmolarity inreasd b.
ICF
volume increased
103. Most common complication of dibetes mellitis ? (help
me correct the stem and the other options fellows?
a.
Dry gangrene b. Wet
gangrene
c.
Raynaud's phenomenon
104. Antibodies are formed from a.
Plasma cells
105. The most potent opsonin; a.
C3B
106. Tumor caused by EBV a.
Nasophyrngeal Ca
107. Female of middle class has cervcal lesion
a.
Cytomegalo virus b.
Herpes
simplex
type 1 c. Human papiloma virus d.
IUCD
108. Toxoplasma; a.
Parasitic infection
109.
What is not present in dermis
a .
Melanocytes b. reticulocyte
110. Dysarthria is the result of damage to;
a.
Cerebellum b.
Broca's area 44
111. Which of following acts at central and peripheral
chemorecrptors for respiration; a.
Inc arterial po2

112. 3rd ventrical ki boundaries important hain. Don't


remember the mcq completely. But it needs to be learnt.
113. Fatal disease of patient;
a.
Shouldn't be told to the patient ad not the family
b.
Crisp clear information should be provided to the
patient according to his needs
c. Should be told to the relatives of the patient.
114. Corona radiata thickness. Cells responsible;
a.
Granulosa cells
115. Simple squamous epithelium wid clefts and smooth
muscles;
a. Uterine cervix b. Uterine body c. Fallopian tubes
116. Langerhan Cells are seen in;
a.
TB b.
Sarcoidosis
116. Female presented with shock with history of abortion
by dai ,on laprotomy large haematoma of broad ligament
formed by;
a. Uterine artery b. Ovarian artery c. Ovarian venous
plexus
117. The cause of death in gas gangrene is due to ;
a. Bacteremia b. Toxic shock c. Pneumonia
118. Uterus prolapse but anus is intact. Which structure not
damaged;
a. Pelvic diaphragm b. Brogenital diaphragm c. Sphincter
urethre ani d. Levator ani peineal body
119. Not a carcinogen; a. Alcohol b. Cyclophosphamide
120. About parasympathetic supply; a. Increased salivary
secretion
121. Insulin is inhibitted by
a. Glucagon b. Beta blockers
122. RBC osmotic fragility Test;
a. Rupture in 0.35 Nacl solution b. Shrink in hypertonic
solution
c. Swell in hypotonic solution
123. Glycogen is not formed in the muscle due to
a. Glucose 6 phosphatase is absent in the muscles
124 . Aldosterone release increased in a. Hyperkalemia
b. Hypernatremia c. Hypercalcemia
124. Preganglionic sympathetic nerve fibers release ;
a. ACH b. Norepinephrine c. Dopamine
125. Regarding the boundaries of inguinal canal;
a. Posterior wall is strengthened in its medial third by
inguinal falx (conjoint tendon)
b. The anterior wall is strenghthened in its lateral third by
inguinal falx
126. Graves' disease;
a. TSH decrease b. T4 dec c. T3 dec d. Increase in the
above mentioned
127. Alpha feto protein is not raised in;
a. Oligohydramnios
b. Hepatitis
c. Hepatocellular
carcinoma d. Heratoma
128. Lesion of s2 s3 s4 nerve causes;
a. Rectal incontinence b. Painless deliver
129. Tubular organ with characteristic aggregates of
lymphoid tissue a. Appendix b. Jejunum c. Ileum
130. Glucose croses placenta by;
a. Active transport b. Co-transprt c. Difussion gradient
Aortic pressure lowest during which phase of cardiac
cycle??
a.atrial systole b.isovolumic contraction c.isovolumic
relaxation
132. 10% Formaldehyde is used for sending Biopsy
specimens, Why?
a. Sterilisation b. To prevent autolysis c. To help he
specimen absorb stain d. To denature proteins e. To cause
lysis of cell
133. After rapid loss of 2 liter of fluid person drinks plain
water it will lead to;
a. Increase in interstitial fluid only b. Increase in intracellular
fluid
c. ECF volume increase
134. A femal presented with history of car accident. She
has extravasation of urine. The interna liliac artery was
ligated to stop bleeding. Which organ is ruptured?
a. Ureter b. Urinary bladder
135. Female at gestational age 34 having symphsio-fundal

height of 28 weeks. The cause may be;


a. Anencephaly b. Esopagial atresia c. Renal agenesis
d. Tracheo-esophageal fistula
135. Stratum Functionalis includes
a. Stratum spongiosum plus startum compactum
136. Regarding surfactant inappropriate one
a. ratio > 4 at 32 week
137. Urinary bladder parasympathetic supply;
a. Pelvic nerves
138. Most comon carcinogen;
a. Virus b. Chemicals
139. Which tumor is malignant;
a. Glioma b. Meningioma
140. Heavy smoker has a small nodule on vocal cord;
a. Atrophy b. Hypertrophy c. Hyperplasia d. Metaplasia
141. Compression of S1 nerve root results in:
a. Loss of ankle jerk b. Positive Babinski sign c. Sensory
loss on sole d. medial aspect of foot
142. The cells forming the corona radiata are;
a. Theca interna b. Theca externa c. Granulosa cells d. A
and b
143. Juandice in pregnany confirmed by;
a. Alkaline phosphate b. ALT
144. Lateral wall of third ventricle is formed by ;
a. Medial 2/3 of thalamus b. Lateral 1/3 of thalamus
c. Medial 1/3 of thalamus d. Lateral 2/3 of thalamus
145. The diagnostic lab test for the infection by
streptococuss beta haemolyticus is;
a. ASO titre b. Blood culture c. ESR d. Leukocytes culture
e. Urine for bile pigment
145. Fast, sharp pain from mechanical origin is mediated by
which fibers;
a. A-alpha b. A delta c. C fiber d. High myelinated
146. Fatty acid oxidation helped by;
a. Biotin b. Vitamin C c. Vitamin D d. Vitamin B12
147. Remanant of vitelline duct;
a.Urachus b.
Mikel's diverticulum
148. Most common site of ectopic pregnancy?
a. Fallopian Tube
149. Condition with increased ESR;
a. Polycythemia b. Pregnancy c. Anemia d. Infection
150. Young lady with MCV 76m MCH 25 the type of
anemia;
a. Normocytic Hypochromic
151. Genital tubrcle form;
a. Clioris b. Labia minora c. Labia majora
152. In the resting stage the binding sites of actin mysin
fibers are covered by; a. Actin b. Myosin c. Tropomyosin
153. Mesonephric duct functionally persists as;
a. Euctus deference b. Epididymus
154. Most common cause of premature ovarian failure;
a. Autoimmune b. Adrenal carcinoma c. Drug induced
d. Old age
155. A pregnant lady with the baby's head at -2 , occipitoanterior position ,where does the needle insert;
a. Anterior fontanelle b. Posterior fontanelle c. Sagital
suture d. Fronal suture
156. Which cranial nerves are parasympathetic;
a. III, VII, IX, X
b. V, VIII, II, XII
157. Most common complication of dibetes mellitis;
a. Dry gangrene b. Wet gangrene
158. Medial quadrant of breast drains into;
a. Internal thorascic group of lymph nodes
159. Pleural pain reffered to the shoulder by;
a. c1, c2, c3 b. c2, c3, c4 c. c3, c4, c5
159. Epidural block 3% 2 chlorpromazine is more rapid than
2% lidocaine??
a. Lipid solubility b. Ester vs amide
c. PKa d. Concentration
160. Tocoplasmosis is;
a. Helminth b. Parasite
161. Local anesthetic pass placenta by;
a. Active Transport b. Bulk effect c. Simpl diffusion d.
Faccilited diffusion
162. Gram -ve infection cause septic shock in which of the
foll0wing tracts;

a. Biliray tract b. Colon c. Smal bowel d. Urinary tract


163. Paramedian approach in spinal anesthesia must
pierce on of ligaments;
a. Anterior spinous ligament
b. Posterior logitudnal
ligament c. ligamentum falvum d. interspinous ligament e.
Supra spinous ligament
164. During 2nd week which is appropriate test for Typhoid
fever;
a. Widal+Blood cultureb. Blood c. culture only d. Stool
culture onl e.Bone marrow
165. In neonates spinal cord ends at?
a. L1 b. L2 c. L3 d. L4 e. T12
166. Sebacous glands are absent in;
a. Corner of lips b. Palms and soles c. Glans of penis
167. Histopathology showing pleomorphism, rete ridges,
loss of polarity and increasd nuclear to cytoplasmic ratio;
a. Squamous cell CA b. verrucous CA
168. Right Adrenal gland drains into;
a.
IVC b. Right renal vein
169. Secondary cartiligineous joint;
a. Pubic symphysis
169. An mcq about great cardiac vein. Ant interventricular
artery ke along jo vien drain karti uska pucha tha, great
cardiac vein tha ans
170. Amniocentesis vs transcervical chorionic villous
sympalling befre 14 weeks;
a. 1% Decrease chance of miscarriage
b. 10% increase risk of fetal talipes
c. Misscarriage rate is 1/2000
d. Rupture of placenta
171. Preganglionic sympathetic nerve fibers release ;
a. ACH b. Norepinephrine c. Dopamine
172. Langerhans giant cells found in;
a. Tuberculosis b. Sarcoidosis c. Wegners granulomatosis
d. Syphlis
173. Antibodies are formed from; a. Plasma cells
174. The following combination of hormones regulate
lactation;
a. oxytocin, prolactin, ACTH b. Oxytocin, prolactin and
cortisol
175. Langerhans giant cells found in;
a. Tuberculosi b. Sarcoidosis c. Wegners granulomatosis
d. Syphlis
176. Which one of them is responsible when a muscle is
stretched it leads to the relaxation of muscle;
a. Golgi tendons b. Muscle spindles
177. Lateral attachment of urogenital diaphragm;
a. Sarum n coccyx b. Obturator internus membrane c.
Ischial tuberosity and pubic ra
178. Type of reaction which occurs after diphtheria vaccine;
a. Anaphylaxis b. Type 1 c. Type 2 d. Type 3 e. Type 4
179. Right kidney is not related to;
a. Descnding colon
180. Fastest control of BP control is by;
a. Baroreceptors b. Cardiac ischemic response
c. Chemoreceptiors
181. Local anesthetic mechanism of action;
a. Decreasing action potential by Blocking Na+ channels
182. AFI index <2 possibily due to;
a. Renal agenesis b. Gastriesophageal fistula
183. Vulval premalgnant lesion is;
a. Vulvar nevib. Condyloma
185. Premalignant lesion on vulva
a. Pagets disease
186. A woman with chest pain,worse on lying down .she
gives History of fever for 10 days ? What's the Diagnosis;
a. Myocarditis b .Pericarditis c.Pleuritis
187. The secret regarding the disease of a patient;
a. Should inform the patient and not the family b. Shouldn't
inform the patient and the family c. Should inform the
relatives and not the family d. Should be breached if the
patient authorizes
188. Patient taking OCP and ATD still got pregnant. The
drug responsible; a. Rifampicinb. Isoniazid
189. Slide showing simple columnar epithelium with
indistinct smooth muscles and clefts;

a. Uterus b. Cervix c. Uterine tube d. Ovaries


190. Condition with increased ESR;
a. polycythemia
191. Dysarthria is due to a lesion in;
a.Brocas
areab.Cerebelluc.Thalamusd.Sensory
motor
cortex e.Hypothalamus
192. 3 % one drugg n other 2 % ANESTHTIC CORSES
placenta dua to;
a. In concentration b. lipid solubility c. Active transport d.
Facillliated
193. A patient who is from a middle class family and has 3
children. She has cervical dysplasia. The reason may be;
a. HPVb. IUCD
194. After 48 hours starvation,energy source;
a. Muscle glycogen b. Muscle protein c. Liver glycogen d.
Triglycerides
195. The cause of death in gas gangrene is due to ;
a. Bacteremia b. Toxic shock c. Pneumonia
196. A female presented with shock with history of abortion
by dai ,on laprotomy large haematoma of broad ligament.
The structure responsible is;
a. Uterine artery b. Ovarian artery c. Ovarian venous
plexus
197. A femal presented with history of car accident, has
extravasation of urine interna liliac artery ligated tostop
bleeding which organ ruptured?
a. Ureter b. Urinary bladder
198. Parasympathetics reduce heart rate by;
a. AV node delay b. Potasium eflux through SA Node.
198. ACTH effectively controls ?
a. Aldosterone b. Adrenal androgens c. Adrenal estrogen
d. Hydrocortisone
199. Oxygen dissociation curve is shifted to the left when;
a. Temperature increases b. Acidosis c. Alkalosis
200. Regarding the extent of Uterine tube
a. It extends
from lateral end of uterus to medial end of ovary
201. Compact bone is characterized by;
a. Concentric
lamelle b. Lacunae containing osteocytes
201. The cause of death in gas gangrene is due to ;
a. Bacteremi b. Toxic shock c. Pneumonia
202. Sphyincter urethrae is the content of;a. Superficial
perinepouchb. Deep perineal pouchc. Anal canal
RADIOLOGY PAPER-B FEB 2014
Qs. 1. Heinz bodies are found in
A.
B.
C.

G6PD deficiency
HS
Sickle cell disease

Qs 2. During a surgery a surgeon nick hepatoduodenal


ligament. Which structure is likely to be damaged
A.
B.
C.
D.

Portal vein
Bile duct
IVC
Hepatic Duct

Qs 3. Gall Bladder
A.
B.
C.

Produces bile
Submucosa is present
Mucosa is thrown in extensive folds

Qs 4. Bare area of liver is limited by


A.
B.

Right and left traigular ligament


Falciform ligament

C.
D.

Ligament teres
Ligament venosum

Qs 5. Medial side of arm is supplied by


A.
B.
C.
D.

Medial cord (answer)?


Lateral cord
Musculocutaneous nerve
Median nerve? (dont rem)

Qs 6. Pendular knee jerk


A.
B.
C.
D.
E.

UMN lesion
LMN lesion
Hypothyroidsm
Parkinson
Hyptonia ( dont rem if this option was given)

Qs 7. Intercostal spaces
A.
B.
C.

Have three msucles in between


Neurovascular bundle present
Options were lengthy and tough. Not from any
MCQ book. So better to give this topic a read
from SNELLs or RJ Last

Qs 8. Neurovascular bundle

A.
B.
C.

Lies deep In ribs


Lied on upper border of ribs?
Lower border of ribs

Qs 9. Mesonephric duct functional remnant or unit is

A.
B.
C.
D.

Epididymis
Ductus deferens
Vagina
Uterus

Qs 10. Metastatic Calcification of kidney most commonly


due to
A.
B.
C.

Hypervitaminosis D
Hyperparathyroidism
Hypercalcemia

Qs 11. Main supply of head of humerus


A.
B.
C.
D.
E.

Ant circumflex artery


Post circumflex artery
Arteries around rotator cuff
Axillary artery
Subscapular artery

Qs 12. Deep cervical artery is a branch of


A.
B.
C.
D.

Costocervical trunk
Subscapular artery
Aorta 1st part?
Int thoracic artery?

Qs 13. Capsule of left kidney prevents the spread of


infection to

A.
B.
C.
D.
E.

Other kidney
Lesser sac
Upper part of greater sac
Lower part of greater sac
Deep pelvis

Qs 14. Proximal part of CBD is supplied by


A.
B.
C.
D.
E.

Inf mesenteric artery


Sup mesenteric artery
Cystic artery
Right hepatic art
Left hep artery

Qs 15. A pt got spleen rupture and she is going to


spleenectomy..whats her peripheral blood picture right now
A.
B.

Howeljowel bodies
Thrombocytopenia

Qs 16. Infants spinal cord ends at


A.
B.
C.

Btw L1 and L2
Lower border of L3
L2

Qs 17. Esophagogastric junction is at


A.
B.
C.
D.

T11
T7
L1
L3

Qs 18. Portal HTN


A.
B.
C.

Left colic (answer)


Inf mesenteric
Infepigastric

Qs 19. A young 1yr old has hydrocele


A.
B.
C.

Scrotal swelling
Peritoneal fluid accumulation
Testis inflammation

Qs 20. During hysterectomy which structure is damaged


A.
B.
C.

uterine artery (answer?)


ovarian artery
? some nerves?

Qs 21. Deep ring is

A.
B.
C.

Fascia transversalis
Aponeurosis of transvers abdominus
Aponuerosis of ext oblique

Qs. 22. Duodenal ulcer which artery is involved


A.
B.
C.

Gastroduodenal artery
Splenic artery
Gastroepoploic artery

Qs 23. Head of pancrease related anteriorly to all except

A.
B.
C.

Rt suprarenal gland (ans)


Left suprarenal gland
Duodenum?

Qs 24. Sigmoid and descending colon drain into


A.
B.
C.

Left colic nodes


Inf mesenteric nodes (ans?)
Sup mesenteric nodes

Qs 25. Hip joint is


A.
B.
C.

Related post to psoas bursa


Has obtext inferiorly
Related to femoral nerve?

Qs 26. Femur bone


A.
B.

Lat ligament attached to lower epicondyle (ans?)


Plantaris muscle??

Qs 27. Rectum is
A.
B.
C.

Straight structure?
Has haustrations circular folds?
Post tp rectum is S2S4S5??

Qs 28. Infepigastric artery & deep inguinal ring (qs was only
this with no extra words or hint)
A.
B.

Medial (ans?)
Lateral

Qs 29. S1 lesion
A.
B.

Loss of ankle jerk


Loss of sensation on medial side

Qs 30. AV bundle
A.
B.

Only neuro connection btw right atria &ventricle


(ans?)
Present in right atria

Qs 31. Post interventricular artery supplies


A.
B.
C.
D.

Both right and left ventricle


SA node
Rt atria
Some other options abt its aneurysm?

Qs 34. Lateral side of breast drains into


A.

Pectoral LN

Qs 35. Esophagus constricted at


A.
B.
C.

Where crossed by arch of aorta (answer)


Post mediastinum + left ventricle
Rt bronchus

Qs 36. Site of radial artery pulse is


A.
B.

Ext pollicis and abdpollicis


Dnt rem options learn it from snell review

Qs 37. Most imp anatomic functional lobe or largest


subdivision of prostate is
A.
B.
C.
D.

Median lobe
Peripheral lobe
Transitional lobe (ans???)
?

Qs 38. Pain of vagina mediated medially due to


A.
B.
C.

Obturator nerve (ans)


Femoral nerve
Sciatic nerve

Qs 39. Female pelvis with short AP diameter and long


transverse diameter is
A.
B.
C.

Plateploid
Gynaecoid
Anthropoid

Qs 40. Rectum nerve supply by


A.
B.
C.

Infhypogastric
Sup hypogastric
Pelvic splanchinic

Qs 41. Symphysis pubis is


A.
B.

Prim cartilaginous joint


Sec cartilaginous joint

Qs 42. Intrinsic factor is secreted from


A.
B.

Gastric fundus
Gastric antrum

Qs 32. Most medial nucleus of cerebellum


Qs 43. Most common site of ectopic pregnancy
A.
B.

Fastgial
Dentate

Qs 33. Defect in interventricular septum causes or affects


which valve most

A.
B.
C.

Uterine tubes
Ovaries
Mesentry of abdomen

Qs. 44. Sickle cell disease is a defect in


A.
B.
C.
D.
E.

Aortic
Pulmonary
Tricuspid
Mitral
Coronary septum?? Dont rem the option and
neither understand the qs

A.
B.

Beta globulin (ans)


Albumin

Qs 45. Common peroneal nerve damage scenario in a


footballer and it was asked where it is damaged

A.
B.
C.

Neck of fibular (ans)


Head of fibular
Ant to tibia

Qs 46. No of divisions of lower airways??


A.
B.
C.
D.

17
21 (ans???)
23
26??

C.

Cervical part??? Dont remember complete


options. But thisqs was NOT FROM ANY BCQ
BOOK.

Qs. 56. Atherosclerosis mainly compress or is present in


A.
B.

Abdominal aorta (dont rem if it was given in


options)
Sup mesenteric artery (ans)

Qs 57. Deep pain around orbit and nose


Qs 47. Reflux of cecum is prevented by
A.
B.

Ileocecal valve
Ileocecal sphincter (ans)

Qs 48. Right testis cancer scenario and asked abt its


drainage
A.
B.

Para-aortic lymph node (ans)


Int iliac lymph nodes

Qs 49. Prostate some scenario was given drains in


A.
B.

Int iliac nodes(ans)


Para aortic nodes

Qs 50. Superficial inguinal lymph nodes doesnt drain


A.
B.
C.
D.

Testis (ans)
Scrotum?
Skin of gonads
Vagina

Qs 51. Left supra renal gland


A.
B.

Separated from left kidney by perirenal fascia


Drains in left renal vein (ans?)

Qs 52. Posteriorly lesser sac has all except?


A.
B.
C.
D.

Right supra renal gland (ans)


Left supra renal gland
Rt kidney
Left kidney

A.
B.

Maxillary nerve (ans)


Supraorbital nerve

Qs 58. Post curvature is lost in an infant. This condition is


called
A.
B.

Kyphosis
Scoliosis

Qs 59. A girl has problem in opening mouth. Which muscle


is defected
A.
B.
C.
D.

Lateral pterygoid
Medial pterygoid
Temporalis
Buccinators

Qs 60. Annulus fibrosis


A.
B.

Attached med and lat ligaments/?


No sensory supply??

Qs 61. Phenylalanine converts into


A.
B.

Tyrosine (ans)
Tyrmaine

Qs 62. Alkaptonuria all except:


A.
B.
C.

Child has dark urine


Child has dark ears
No failure to survive/failure to survive (ans??)

Qs 63. Amykoidosis all except


Qs 53. Parasympathetic supply to submandibular gland is
from
A.
B.
C.
D.
E.

Sup salivary nucleus


Inf salivary nucleus
Auriculotemp nerve
Lesser petrosal
Greater petrosal

A.
B.

Not present in heart (ans??)


Nephrotic syndrome

Qs 64. Most common cause of PDA


A.
B.

Prematurity (ans)
Infections

Qs 54. Post belly of digastric is supplied by


Qs 65. Human gonadotropins hormone main function is to
A.
B.

Facial nerve
Accessory nerve

Qs 55. Vertebrae
A.
B.

Post and inf spine??


Nerve leave sup???

A.
B.
C.

Maintain pregnancy (ans?)


Enhance breast
Ovulation

Qs 66. Which goes to cortex some visual pathway qs


A.
B.

LGB
Geniculocalcarine tract (ans?)

Qs 67. ESR is raised in


A.
B.

A.

Qs 79. Left renal vein

A girl with periods


In infections (ans)

Qs 68. Right atrium of heart. It was not abt venous


drainage. Abt structure of right atrium
A.

Sinus venosus
rt atria (ans)

b Coronry sinus drains in

Qs 69. Stab wound injury in intercostal space..surgeon


needs to go to endothoracic fascia by reaching
A.
B.
C.
D.

no options remembered

Ext intercostal and internal intercostal muscles


Neurovascular bundle??
Some weird options not given in any MCQ book.
Parietal pleura??

A.
B.

some relation options


Drains in IVC directly (I chose that)

Qs 80. Right renal vein


A.
B.

Dont remember qs exactly


Marked drains in IVC directly

Qs 81. Adult derivative of umblical vein


A.
B.

Ligamentumvenosum (ans)
Ligarteriosum

Qs 82. Pancrease related anteriorly to all except: (dont rem


if it had except or not)

Qs 70. Area of visual field supplied by


A.
B.

PCA
MCA

A.
B.
C.
D.

Lesser sac
Spleen
Splenic vein
SMA

Qs 71. Homonymous hemianopia caused by


Medicine Online 28th August,2014 with ANSWERS !!
A.
B.

Optic tract
Optic chiasma

Qs 72. Intrauterine midtrimester hematopoiesis occurs in


A.
B.

Liver (ans)
Spleen

Qs 73. Central and peripheral chemoreceptors are sensitive


to
A.
B.
C.

Dec P02
Inc CO2
IncPh

1. A 40 year old teacher while writing on blackboard is


suffering from shaking of his hands. His cousin has same
problemPipe lead rigidity and other sign not reportedall
other signs are unremarkableDrug should be given is
ROPINIROLE.
2. Which has got soft keratin with having no nucleus
A. St. Corneum (ANSWER)
B. St. Basalis
C. St. basalis & st. spinosum
D. Hairs follicle
E. base of the nail
3. In anemia , blood becomes turbulent due to
Decrease blood viscosity (ANSWER)

Qs 74. Premalignant condition is caused by all except:


A.
B.
C.
D.
E.

Pagets disease
Cirrhosis of liver
AIDS
Osteoporosis??
Some breast disease too

4. HIV drug causes pacreatitis and peripheral neuropathy


is
Didanosine (Answer)
5. Anaphylactic shock comparison with Hypovolumic shock
has
Increase cardiac output (answer)

Qs 75. Hodgkin disease


A.
B.

More in male
Lymphocyte depletion has some prognosis??

Qs 76. Autosomal dominant is


A.

Marfan syndrome

Qs 77. Pulmonary TB
A.
B.

Most in post upper lobe (ans)?


Foul smelling sputum

Qs 78. Superior thoracic aperture

6. Adenoma has
Glandular cells ( answer)
7. Anterior SURFACE of the heart is formed by Right
Ventrilcle (anwser)
( remember the question was about surface NOT about
borderanterior border of heart is formed by Right atrium)
8. A scenario of VMA found in yrine Pheochrocytoma
(ANSWER)
9. Ovarian cancer markerCA 125
10. A scnerio of 5 hydroxyindoleacetic acid Carcinoid
tumor
11. A young man having dysphagia, horness of voice
.case of medullary cell carcinomamarker is
CALCITONIN

12. Increase carbohydrate consumption will increase the


demand of
A. Thaimine (Answer)
B. Riboflavin
C. Niacin
13. Scnerio about tremors and rigidity..are
affectedSubstantia Nigra
14. Protooncogene is converted to oncogene by
PONT mutation
15. The most common cause of mental retardation is
A. Cri du chat syndrome
B. Fragile X chromosome ( Answer)
C. Edward syndrome
D. Turner syndrome
16. The length of right bronchus is
2.5 cm (answer)
17. Creatinine is important in assessing renal failureit
arises maily from
A. Liver
B. Brain
C. Kidney
D. Skeletal muscle ( answer)
18. Muscle carrying capacity with hemoglobin is
A. Actin
B. Troponin
C. Myoglobon (Answer)
B. Tropomyoglobin
19. Increase GFR due to
Dilation of afferent arterioles
20. Hamartoma is defined as ?
A. mass of mature disorganized tissue indigenous to that
organ (Answer)
B. ectopic rest of normal tissue
C. a tumor that can metastasize
D. a malignant tumor that can cause serious complications
E. Totipotent cells
21. Flare due to arteriolar dilation is due to AXONAL
REFLEX
22. A new born baby is having complain of passing urine
from umbilicus to urachus
A. ectopic ureter
B. Ectopic urethera
C. persistent allantois (ANSWER)
23. A pateient having CVP line in subclavian vein & on
parental nutrition. He develop fever.Blood CP shows cluster
of gram +ve cocci. The drug of chaoice for this organism is
A. Amoxicillin + Clavulanic acid (ANSWER.Rabbia ali
mcqs PAGE 443..qs # 246..2ND EDITIOIN)
B. Vancomycin
C. Ceftazidim
D. Imepenam
E. Piperacillin + Tanzobactam
24. True about cerebellum
A. Connected to medulla through middle cerebellar
peduncle
B. Resting tremors occur after injury
C. Lies below tentorium cerebellum (ANSWER)
D. Ataxia not occur after this lesion
25. Melanocytes are derived from
A. Endoderm
B. Ectoderm
C. Neural crest cells ( Answer)
D. Pharyngeal arch
E. Lateral plate
26. Primordial germ cell of yolk sac are derived from
A. Ectoderm
B. Mesoderm
C. Endoderm (answer) .Rabbia ali BCQ page 140
27. Which of the following would produce maximum
excitation of the hair cells in the right horizontal semicircular
canal?
A. Hyperpolarization of the hair cells
B. Bending the stereocilia away from the kinocilia
C. Rapid ascent in an elevator
D. Slow ascent in an elevator
E. Rotating the head to the right (ANSWER

MOCK Test With Key


Q1 All of the following result in detoxification of drugs
except:
a. NADPH cytochome P450 reductase b. Cytochome
P450 c. Cytochome Oxidase d. Monoxygenases Key C
Q2 Phase II reactions of a drug biotransformation:
A Decreases its water solubility.
B Includes activity of cytochrom P450.
C Usually leads to inactivation of the drug.
D Does not include acetylation.
E Occur at the same rate in adults and the new born.
Key: c
Q3 Half life of a drug may be helpful to determine:
A Dosage schedule of the drug.
B Level of absorption.
C Distribution into different body systems.
D Rate of absorption through GIT.
E Time to get the steady state.
Key: e
Q4 Epinephrine does not cause increase concentrations of:
A Glucose in blood.
B Lactate in blood.
C Free fatty acids in blood.
D cAMP in heart muscle.
E Triglycerides in fat cells.
Key: e
Q5 Which of the following drugs given preoperatively is
used to
prevent postoperative pain caused by succinylcholine?
A Dantrolene. B Tubocurarine. C Diazepam. D Lidocaine. E
Baclofen. Key: b
Q6 The steady state concentration of a drug can be double
by:
A Doubling the both rate of infusion and concentration of
drug.
B Doubling the rate of infusion only.
C Doubling the loading dose but maintaining the infusion
rate.
D Tripling the rate of infusion.
E Quadripling the rate of infusion.
Key: b
Q7 A 3 year old child has been admitted to emergency with
suspicious of atropine overdose as there are:
A Abdominal cramps.
B Increased gastric secretion.
C Increased cardiac rate.
D Papillary constriction.
E Increased urinary frequency.
Key: c
Q8 A 50 year old patient is having propranolol due to his
cardiac
problems but his physician now wants to stop this drug;
which of the following is the most important reason for this
step?
A Familial tremor. B Partial AV heart block. C Mild
hypertension. D Angina pectoris. E Supraventricular
tachycardias. Key: b
Q9 Which of the following drugs when used for prolonged
period in the maintenance treatment of tonic clonic seizures
can lead to increased metabolism of warfarin like drugs?
A Phenobarbital.
B Meprobamate.
C Chlordiazepoxide.
D Triazolam.
E Zolpidum.
Key: a
Q10 Characteristics of non depolarizing neuromuscular
blockade include which of the following?
A Block of post tetanic potentiation.
B Histamine blocking action.
C Poorly sustained tetanic tension.
D Significant muscle fasciculations during onset of block.
E Stimulation of autonomic ganglia.
Key: c

Q11 A 35 year old lady who is on anticancer therapy is now


suffering from severe nausea and vomiting which of the
following drugs is having most effective antiemetic actionsin
this case?
A Dexamethasone. B Levodopa. C Apomorphine. D
Sucrulfate. E Omeprazole. Key: a
Q12 Which of the following is a prophylactic antiasthmatic
agent
that stabilizes mast cells:
A Ipratropium. B Prednisone. C Terbutaline. D
Cromoyln. E Aminophyllin. Key: d
Q13 A 30 year old male suffering from cerebral edema will
be best
treated with:
A Furosemide. B Amiloride. C Ethacrynic acid. D Mannitol.
E Acetazolamide. Key: d
Q14 A 45 year old patient developed severe
thrombocytopenia due to unfractionated heparin so the best
alternative anticoagulant will be:
A Lepirudin. B Abciximab.C Urokinase. D Plasminogen. E
Vit. K
Key: a
Q15 Acute hemorrhage cystitis is a common toxic effect
seen with: A Vincristine. B Tamoxifen. C Doxorubicin. D
Cyclophosphamide. E Fluorouracil. Key: d
Q16 The toxic effect of amphotericin B which definitely
require reduction of dose is:
A Myelosuppression.
B Hepatitis. C Renal Toxicity. D Hypotension. E Infusion
related adverse effects. Key: c
Q17 Doxycycline is:
A Bactericidal.
B Not excreted in the feces.
C Having a short elimination half-life.
D Not effective in lyme disease.
E Not as effective as tetracycline against H.pylori.
Key: e
Q18 Neomycin, an aminoglycoside is:
A Least nephrotoxic.
B Metabolized by hepatic enzyme.
C Used in hepatic coma.
D A drug of choice in Rockey Mountain Spotted fever.
E An adjunct in treatment of tuberculosis.
Key: c
Q19 Indicate the local anesthetic, which is more lipidsoluble:
A Bupivacaine B Lidocaine C Mepivacaine D Procaine
E cocaine key: A
Q20 Indicate the function, which the last to be blocked in
anesthesia:
A Pain, temperature B Muscle spindles C Motor
function D Touch, pressure E All block at same time
Key: C
Q21 The symptoms of excessive stimulation of muscarinic
receptors include all of the following EXCEPT:
A Abdominal cramps, diarrhea
B Increased salivation, excessive bronchial secretion
C Miosis, bradycardia
D Weakness of all skeletal muscles
Key D
Q22 Which of the following drugs is both a muscarinic and
nicotinic blocker?
A Atropine B Benztropine C Hexamethonium
D Succinylcholine E Tubocuraine
Key B
Q23 Epinephrine is used in the treatment of all of the
following disorders EXCEPT:
A Bronchospasm
B Anaphylactic shock
C Cardiac arrhythmias
D Open-angle glaucoma
E All of above
Key C
Q24 Valproate is very effective against:
A Absence seizures
B Myoclonic seizures

C Generalized tonic-clonic seizures


D All of the above
Key D
Q25 The mechanism of both topiramate and felbamate
action is:
A Reduction of excitatory glutamatergic neurotransmission
B Inhibition of voltage sensitive Na channels
C Potentiation of GABAergic neuronal transmission
D All of the above
E none of above
Key D
Q26 The pathophysiologic basis for antiparkinsonism
therapy is:
A A selective loss of dopaminergic neurons
B The loss of some cholinergic neurons
C The loss of the GABAergic cells
D The loss of glutamatergic neurons
E all of above
Key A
Q27 A long-acting drug against both absence and
myoclonic seizures is:
A Primidone B Carbamazepine C Clonazepam D
Phenytoin E Ethionamide Key C
Q28 The rate of secretion of thyrotropin is controlled by:
A The amount of iodine in the thyroid gland
B The amount of thyroid hormones in the thyroid gland
C The concentration of thyroid hormones in blood
D The concentration of catecholamines in blood
E The concentration of cortisol in blood
Key C
Q29 If an agonist can produce maximal effects and has
high efficacy its called:
A Partial agonist B Antagonist C Agonist-antagonist D
Full agonist E non competitive antagonist
Key D
Q30 Idiosyncratic reaction of a drug is:
A A type of hypersensitivity reaction
B A type of drug antagonism
C Unpredictable, inherent, qualitatively abnormal reaction
to a drug
D Quantitatively exaggerated response
E A type of drug agonism
Ans:C
31)papez circuit in limbic system involve
A) anterior thalamic nuclei
B) pulvinar nuclei
C) anterior hypothalamic nuclei
D) interlaminar nuclei
Answer A
32) site of rbc formation in adult is
A) long bones B) flat bone C) liver D) spleen Answer B
33) botulinum toxin involve all of the following except
A) neuromuscular junction B) post ganglionic
parasympathetic ganglia C) peripheral ganglia D) CNS
Answer D
34) which of the following hormone is not secreted by
kidney
A) renin
B) angiotensin 1 C) 1,25 dihydroxycholecalceferol
D) erythropoeitin
Answer B
35 ) enzyme not stable at acidic ph
A) trypsin B) chymotrypsin C) pepsin D)
carboxypeptidase
Answer C
36)Iron is stored in the body in the following except
A) Spleen B) reticuloendothelial system C) gall bladder D)
bone marrow Answer C
37) absolute refractory period in the heart
A) corresponds to the duration of relaxation
B) last till half of cardiac contraction
C) shorter than refractory periods in skeletal muscle
D) last till cardiac contraction
Answer D
38) increased vagal tone causes
A) hypertension

B) tachycardia
C) bradycardia
D) increase in cardiac out put
Answer C
39) the extrapyramidal system is not concerned with
A) stretch reflex B) righting reflex C) spasticity D)
sensation of viscera Answer D
40) epiphyseal closure is regulated by
A) calcitonin B) somatomedians C) 1,25
dihydroxycholecalceferol D) thyroxine Answer B
41)..A 43 years old Diabetic suffering from Stroke,His
desire for drinking water is obtunded since he
recoverd.Which of the following is appropriate in this
patient.
a.watershad Infarcts of Post Hypothalamus.
b.Lesions involving posterior commuinating
artery
c.Lesions involving Middle ceebral artery
d.Lesions involving anterior Communicating
artery.
e.0.9%NaCl Infusion therapy Intravenously
(Answer is Dlesion involving Anterior
communicating artery Inhibits or Obtund the intake of
drinking water since it supplies area concered with
ThirstGanong Page no 310..24th Ed)
42.All of the following events in Humans are initiated by
Heat Except.a.Anorexia b.Increase in Respiration
c.Cutaneous Vasodilatation d.Apathy e.Incesed levels of
Epinephrine in Blood
(Ans is E.Referance Ganong Page No 319 24th
Ed)
Q No 43.The insensible loss of water in humans is
a.10mL/h b.20mL/h c.30mL/h d.40mL/h e.50mL/h
(Answer is Ereference is Ganong Pg no 318 24th
ed)
Q No 44.Most Common metabolic finding in Humans with
Hypothalmaic disease is.
a.Anorexia b.Bulemia c.Diabetes insipidus d.Precocious
puberty
e.Hypogonadism
(Ans is DRef Ganong page no 316 24th Ed)
Q No 45.A drug abuser of Heroin is presented with
Hypertension,What is appropriate.
a.His Serum TSH levels are normal to decreased.
b.His serum TSH levels are normal to increased
c.His serum T3 levels are decreased
d.His serum T3 levels are increased
e.He is clinically Euthyroid
(Ans is E..Ref Ganong page No 344 24th Ed)
46.Delta waves in ECG are charecterstically found in
a.AV junctional rhythum
b.Wolf- Parkinson-white syndrome
c.Lown-ganong-Levine syndrome
d.Brugada Sybdrome
e.Hyperkelemia
(Answer is B Reference essential revision Notes
Karla 3rd ed )
47. Metabolic cause of Proarrythmic Abnormal
repolarization (ST-T changes) is.
a.Hpercalcemia b.Hypernatremia C.Hyperthermia
d.Hyperkaelemia e.Hpothyroidism
(Ans is EReference essential revision Notes
Karla page no 29 3rd ed)
48.Gastrin secretion is inhibited by all Except
a.increased Serum Somatostatin concentration
b.increased Luminal Somatostatin concentration
c.Incresed Serum (VIP) vasoactive intestinal peptide
concentration
d.Increased Luminal gastric acid concentration.
e.Increased serum concentration of Glucagon
(Answer is Areference Ganong Page No 471 24th
Ed)
49.Regrading Secretion of Renin,which is most appropriate.
a.Intra renal Baroreceptors mechanism Increases it

b.Its secretion is Directly proportional to Na and Cl entering


DCT
c.Its secretions is Inversely proportional to Na and Cl
entering DCT.
d.Thirst increases it
e.Thirst decreases it.
(Ans is CReference Ganong page no 705
.24th Ed)
50.Regarding Pressures in blood vessles.which one is
appropriate
a.Pressure in carotid is higher compared with
pressures in Brachials.
b Gravity has no effect in Pressure dynamics in
vessels.
c.Presure in any vessel below heart is increased
d.Standing decreases cerebral pressure.
e.Pressures in any vessel below heart is
Decreased.
(Ans is C.Page No 578 Ganong.effect of
gravity24th Ed)
51.Paroxysmal
Nocturnal Hemoglobinuria is best
diagnosed by Which of the following.
A.Blood Compelte picture
B.HAM acidified serum Test
C.Sugar Water Test.
D.Flow Cytometery
E.Hypocomplementemia
(Ans is D.Flow cytometery.Refernce
Goljan page no 306 4th Ed)
52.Which of the following has Increase MCHC.
a.Iron deficiency anemia
B.Sickle cell disease
c.Heridatery Spherocytosis
d.Heriditary Eliptocytosis
e.Immune Hemolytic anemia.
(Ans is D.Heriditary Spherocytosis.Refrence
Goljan Page no 304 4th Ed)
53.Regarding Hypersegmented neutrophils which is
appropriate.
A.Associated with Iron deficency anemia
B.They are used charecterstically as marker of
Pernicious anemia.
C.Almost always present with pancytopenia.
D.Used to describe when Neutrophils have more
than five nuclear lobes.
E.Commonly seen in acute inflammatory
response.
(Ans is DReference Goljan Pathology ..page no
299 4th Ed)
54.Reticulocytosis is Usually evident in acute Hemorrhage.
a.3-4 hours after the acute event.
b.5-10 hours after the acute event
c.1-2 days after the acute event
D.5-7 days after the acute event
E.14-28 days after the acute event.
(Ans is D.Reference Goljan Pathology page no 301 4th
Edition)
55.Aplastic Crisis of Sickle cell Anemia is characterized by
a.Lymphocytosis
B.Lymphoctyopenia
c.No change in Red blood cell count
D.Reticulocytosis
E.Reticulocytopenia
(Ans is EReferenace Goljan page no 309 4th ed)
56.Intravascular Hemolysis is Best characterized by.
a.Increased Unconjugated bilirubin levels in blood.
b.Increased unconjugated bilirubin levels in Urine
c.Normal Levels of Unconjugated bilirubin in blood
d.No change in urinary execration of Hemoglobin.
e.Decreased Unconjugated Bilirubin levels in blood
(Ans is A..Reference Goljan 4th Ed)
57.Sterile Pyuria is usually diagnose by presence of which
of the following in Urine
A.Myoglobin b.hemoglobin c.urobilinogen d.Leukocyte
esterase

e.Bens jones proteins


(Ans is D..Ref Goljan..page no 505
4th edition)
58.Hypoalbunemia in Nephrotic syndrome accelerates the
synthesis of which of the following.
a.Albumin itself as a replenishment measure.
b.Antithrombin III
c.Erythropoietin
d.Cholestrol
e.acute phase proteins
(Ans is D..reference Goljan Page No 512.4th
Edition)
59.A 7 years old girl is noticed by her mother for a palpable
lesions on her Buttocks that does not blanch when pressure
is applied, along with pain in both knees and small joints of
hands. She had an episode of Sore throat 1 week
earlier.What could be the Cause.
a.A sequelae of staphylococcal Infection. b.Infective
arthritis c.Churg-Strauss Syndrome d.Reiters Syndrome
E.Henoch Schonlein Purpura.
(Ans is E Henoch Schonlein Purpura..most
common vasculitis in children Reference is Goljan
Pathology Page Number 234 4th ed)
60.Regarding Euthyroid Sick syndrome which is most
appropriate.
a.it is a associated malignancy of Adrenal Gland..
B.serum TSH levels is specifically remain normal to
Increased.
C. serum TSH levels is specifically remain normal to
Decreased.
D.Symptoms usually require Thyroidectomy.
e.Levothyroxine is drug of choice in the treatment options
(Ans is C ..Reference Goljan Pathologypage No
602 4th Edition)
61.Bithermal antibodies are found in which of the following
conditions
a.Immune hemolytic anemia b.Paroxsymal nocturnal
hemoglobinuria
c.Heriditary Eilptocytosis d.Sickle cell disease.
e.Paroxysmal cold hemoglobinuria
(Ans is ERef is Goljan Page No 307 4th Ed)
PHARMACOLOGY BCQs
62.A patient of epilepsy is using Phenytoin for the last 1
year,His Blood complete picture shows Megaloblastic
anemia.it is due to.fact that.
a.it decreases the vit B 12 stores in body.
B.it increases the osmotic volume of RBCs.
C.it increases the folic acid consumption in the body.
d.it inhibits the intestinal Conjugase
e.it increases the activity of luminal Conjugases
(Ans is DReference Katzung review)
63.Regarding Sulfasalzine which of the following is
appropriate.
a.5-ASA is the active component which is absorbed to have
systemic anti-inflamatory activity.
b.can cause Ireversible Azoospermia.
C.can safely be given in patients with Aspirin allergy.
D.its metabolisim is impotant clinically for Acetylator status.
E.Black females are at risk of Hemolysis.
(Ans
is
D..Ref
is
from
Katzung
and
Davidson,Sulfasalazine is composed of 5-ASA and
sulfapyridine which are cleaved by bacterial enzymes in the
bowl.5-ASA is not absorbed but sulfapyridine is .5-ASA is
active in IBD,whereas sulphapyridine is active in
Rheumatoid disorders.Reversible Azoospermia is a side
effect.in patient with aspirin allergy will react to
sulfasalazine.Black males are at risk of Hemolysis owing to
G6PD deficiency, which is X linked. Acetylator status is
important Clinically in the metabolism.)
64.Which of the following Drug increases the formation of
Hb F.
a.Busulfan b.Cyclophosphamide C.Hydroxyurea
D.Tyrosine kinase inhibitors E.5-flourouracil.
(Ans is Creference is Goljan)

65.Pneumonia in Patients with sickle cell disease can be


prevented by Prophylactic use of which of the following
agent.
a.Amoxicillin calvulnate. B.Gentamicin c.Cefotaxime
d.Penicillin e.Cefuroxime
(Ans is DKatzung review,Goljan pathology )
66.Non Ionized drugs are
a.lipid soluble more than water soluble
b.water soluble more than lipid soluble
c.Neither soluble in water Nor in Lipids
d.are strong acids
e.are strong bases
(Ans is A..reference Kaplan Pharma)
67. Chronic,accelerated and blast phases of CML are
treated by which of the following agent.
a.Dasatinib b.Nilotiib c.Eculizumab d.Bosutinib E.Imitinib
(Ans is EReference is Medscape)
Tyrosine kinase inhibitorsused in treatment of CML
Imatinib mesylate (Gleevec): For chronic, accelerated,
and blastic phases; standard treatment of choice
Dasatinib (Sprycel): For chronic phase
Nilotinib (Tasigna): For chronic phase
Bosutinib (Bosulif): For chronic, accelerated, and blast
phases
Ponatinib (Iclusig): For chronic or blast phase T315I positive cases, or in appropriate patients in whom no other
TKI therapy is tolerated or indicated
67) All are motor nerves except: A : Accessory B :
Abducent C : Trochlear D : Trigeminal Ans: D
68) Middle meningeal artery is transmitted through:
A : Foramen rotundum B : Foramen ovale C : Foramen
spinosum D : Foramen lacerum Ans: C
69) pleural tapping in mid axillary line, muscle not pierced is
:
A : Int. intercostal B : Ext. intercostal C : Innermost
intercostal D : Transversus thoracis Ans: D
70|) In epidural Lumbar puncture, structures pierced A/E :
A : Post. longitudinal ligament B : Ligamentous flavum C
: Interspinous Lig. D : Supraspinous Lig. Ans: A
71) Fibrous Joint A/E:
A : Skull Sutures B : Tooth Socket C : Inf. tibiofibular D :
Symphisis pubis Ans: D
72) SUBSTANCES THAT DO NOT CROSS THE
PLACENTAL MEMBRANE ARE A/E : A : IgG B : IgA C :
IgM D : IgE Ans: A
Q.73 : Double barr body is seen in :
A : Turners B : Klinefelters C : XXX D : Downs Ans: C
Q.74 : Which of the following disappear in umbilical cord:
A : Left art. B : Left vein C : Right art. D : Rt. vein Ans:
D
Q.75 : Implantation occurs at the stage of :
A : Zygote B : Morula C : Blastocyst D : Primary villi
Ans: C
Q.76 : A midline cleft lip is due to failure of fusion of:
A : Maxillary processes B : Medial nasal processes C :
Medial & lateral nasal process D : Medial nasal &
maxillary process Ans: B
Q.77 : which of the following is a feature of Y chromosome
?
A : Acrocentric B : Telocentric C : Submetacentric D :
Metacentric Ans: A
Q.78 : All of the foll. are pneumatic bones Except :
A : Maxilla B : Parietal C : Ethmoid D : Mastoid
Ans: B
Q.79 : All of the following are composite muscle Except:
A : Pectineus B : Rectus femoris C : Adductor magnus
D : Biceps femoris
Ans: B
Q.80 : The skin overlying the region where a venous cut
down is made to access the great saphenous vein is
supplied by :
A : Femoral N. B : Sural N. C : Tibial N. D : Superficial
Peroneal N. Ans: A
Q.81 : Porto systemic shunt is not seen in :
A : Liver B : Spleen C : Anorectum D : Gastro
esophageal

Ans: B
Q.82 : aortic opening transmits A/E : A : aorta B : thoracic
duct C : vagus nerves D : azygos vein Ans: C
Q.83 : The urogenital diaphragm is formed by the following
structures except : A : Deep transverse perineal muscle B
: Perineal membrane C : Sphincter urethrae D : Colles
fascia Ans: D
Q.84 : The following structures pass posterior to the flexor
retinaculum except : A : Flexor digitorum superficialis
tendons B : Median nerve C : Ulnar nerve D : Flexor
pollicis longus tendon Ans: Cs
85)Pain of ovarian carcinoma is referred to
a) Back of thigh b) Gluteal region c) Anterior surface of
thigh d) Medial surface of thigh Ans: b
86)Causes of primary amenorrhea are A/E a) Rokitansky
syndrome b) Sheehan syndrome c) Kallman syndrome
d) Turner syndrome Ans: b
87) Which of the following treatments for menorrhagia is not
supported by evidence?
a) Tranexemic acid b) Ethamsylate c) Combined OCP
d) Progesterone Ans:b
87)Lady undergoes total radical hysterectomy for Stage Ib
Ca Cervix. It is found that Ca extends to lower part of body
of uterus and upper part of cervix. Next step of
management
a) Chemotherapy b) Radiotherapy c) Chemoradiotherapy
d) Follow-up
Ans:c
88)Ideal contraceptive for lactating mothers
a) Barrier method b) Combined OC c) Lactational
amenorrhoea d) Progesterone only pills Ans:d
89) A pregnant lady had no complaints but mild cervical
lymphadenopathy in 1st trimester. She was prescribed
spiramycin but she was non compliant. Baby was born with
hydrocephalus &
intracerebral calcification. Which of these is likely cause
a) Toxoplasmosis b) CMV c) Cryptococcu d) Rubella
Ans:a
90) The presence of increased levels of which of the
following in amniotic fluid is an indicator of open neural tube
defect in the fetus?
a) Phosphatidylesterase b) Acetylcholinesterase
c) Pseudocholinesteras d) Butyrylcholinesterase Ans:b
91) Vaginal delivery can be allowed in A/E
a) Monochorionic monoamniotic twins b) Mento-anterior
c) Extended breech d) Dichorionic twins First vertex,
second breech Ans:a
92)A lady with placenta previa delivered a baby. She had
excessive bleeding. After resuscitation most likely
complication is
a) Galactorrhoea b) Diabetes insipidus c) Loss of
menstruation d) Cushings syndrome Ans:c
93) Which organ is the primary site of hematopoiesis in the
fetus before midpregnancy ?
A) Bone
B) Liver
C) Spleen
D) Lung
Ans:b
94) Choline esterase in fetus most specific for ?
A) Open spina bifida
B) Gastrochisis
C)
Exomphalos
Ans:a
95) Perinatal transmission of Hepatitis B is maximum when
infection in mother occurs in ?
A) At implantation

B) 1st trimester

C) 2nd

trimester
D) 3rd trimester
Ans:d
96) The shape of cervical canal leading to preterm delivery
is ?
A) T shaped
shaped
Ans:c

B) Y shaped

C) U

97) In Bishop's score all are included except ?


Effacement of Cervix

A)

B) Dilatation of Cervix

C)

Station of head
D) Interspinous diameter
Ans:d
98) External cephalic version is contraindicated in ?
A) Anemia
B) Flexed Breech
C) PIH
Ans:c
99) An ovarian cyst is identified in the immediate postpartum period. What should be the timing of surgery ?
A) 2 weeks
B) 6 weeks
C) 8 weeks
D)
Immediately
Ans:d
100) The type of pelvis which is associated with increased
incidence of 'face to pubis' delivery is ?
A) Android pelvis
Gynaecoid pelvi

B) Anthropoid pelvis

D) Platypelloid pelvis

C)

Ans:b

M0ck test 2015


1.Lady with infertility with bilateral tubal block at cornua.
Best method ofmanagement is :
Laparoscopy&Hysteroscopy Hydrotubation IVF
Tuboplasty
Answer : A; Laparoscopy is done forproper
assessment & to exclude active infection or TB.
2Women with postmenopausal bleeding need endometrial
sampling if endometrial on US is thicker than
1mm 2mm
5mm
8mm
10mm
Answer : C ; endometrialthickness in menopause > 5
mm is suspicious forhyperplasia
3. Which of the following change in puberty is influenced by
the estrogen:
Growth ofthe acinarbudsof the breast
Epiphysealfusion
Proliferatvephase
Allof theabove
Noneoftheabove
Answer : D
: Estrogen is critical for epiphyseal fusion
inboth young men and women .Proliferative phase
also continues under the effect of ovarian estrogen
produced by thematurating follicles till ovulation occurs

. Estrogen also can affects both ductal andglandular


acinar system of breast .
4. Sub urethral diverticula may occur as a sequelae to
infection of:
Bartholinsgland
Skenesgland
Clitoralgland
Vulvovaginalgland
Answer : B
; Periurethral glands (Skene's glands) are
tubuloalveolar structures along thedorsolateral aspect
that drain into the distal two thirds of the urethra.
Repeated infectionand obstruction of these glands
lead to formation of suburethral cysts or abscesses
thatcan rupture into theurethral lumen.
5.any loss or abnormality of psychological , physiological or
anatomical structure or function is ?
a. impairment b. disability c. handicap d. disease
6.a data is arranged as satisfied , very satisfied ,
dissatisfied . this is a ?

D) O shaped
a. nominal data b. ordinal data c. interval data d. ratio

7.all can be incenirated except ?

14.Muscles supplied by anterior division of mandibular


nerve are all except?

a. cytotoxic waste b. sharps c. anatomical waste


infectious waste

Medial pterygoid B. Lateral pterygoid C. Temporalis

8.herd immunity all are true except ?

15.Not an action of bradykinin?

a. herd structure is constant


b. it is mostly due to subclinical infection
c. can be aquired by immunization
d. spread of epidemic is influenced by it

A.Bronchodilation B. Vasodilatation C. Pain

D.

Increase in vascular permeability


16.A seven year old boy presented to the OPD with

9.Areas involved in posterior cerebral artery infarct are all

complaints of pain anddischarge from left ear. On

except?

examination the patient was febrile with neck rigidity and

A. Thalamus B. Temporal lobe


D.Choroid plexus

D.

Masseter

positive Kernigs sign. Gram positive cocci were visualised

C.Anterior cortex

in the gram stained specimen of the discharge.Most likely

Correct answer
: C.Anterior cortex

10.Decreased energy production in thiamine deficiency is

pathogen is? a.Streptococcus pneumonia B.

due to?

Staphylococcus aureus C. Haemophilus influenza D.

A. It is a cofactor for oxidative reduction

Staphylococcus epidermidis

B. It is required for transamination reactions


C. It is a coenzyme for alpha ketoglutarate dehydrogenase
and pyruvate dehydrogenase in citric acid cycle

17.Which of the following does not have polysaccharide


capsule related antigen antibody response?
Haemophilus influenza B. Neisseria meningitides C.

D. It is a coenzyme for transketolase in hexose

Streptococcus pneumonia D. Bordetella pertussis

monophosphate shunt
Correct answer
: C. It is a coenzyme for alpha

18.Latent varicella zoster infection is found in?

ketoglutarate dehydrogenase and pyruvate

T cells B cells

dehydrogenase in citric acid cycle


11.40 year old female came to the OPD with complaints of
with depressed mood, insomnia, loss of appetite and lack of
interest in surroundings for the past 1 year. These
symptoms appeared after loss in a business deal 1 year

Macrophages Trigeminal ganglion

19.Draughtsman appearance is seen with colonies


of:Pneumococcus

Streptococcus Staphylococcus

None of the above

ago. Which is correct regarding the management?

20.Which of the following is false regardingintraocular

A. Selective Serotonin Reuptake Inhibitors (SSRIs) are the

retinoblastoma?

most effective drugs

94% cases are sporadic

B. Antidepressants should be given based on the side

B. Individualswith sporadic retinoblastoma do not pass

effect profile

their genes to their children

C. No treatment is required as it is due to loss in a business

C. Reese Ellsworth classification is used for predicting

deal

visual prognosis following radiotherapy

D. Combination therapy with 2 or more antidepressants

D. Tumour calcificationcan be detected by anultrasound

Correct answer
: B. Antidepressants should be given

scan

based on the side effect profile


The history is characteristic of depressive disorder.

21.Bitemporal hemianopiais a characteristic feature of?

Antidepressant drugs are to be given. But there are no

Glaucomab) Optic neuritis c) Pituitary tumour d)Retinitis

ideal antidepressants. The treatment should be based

pigmentosa

on the side effect profile of the drugs.


22.All of the following are involved in endophthalmitis
12.Most common cause of premature death in

except?

Retina Vitreous Sclera

Uvea

schizophrenia is?
Drug toxicity
Suicide

B. Nosocomial infection

C. Homicide

D.

Visual evoked response Electrooculogram

Correct answer
: D. Suicide

13.Which of the following is not a pneumatic bone?


A. Frontal bone B. Sphenoid C. Maxilla
Correct answer
: D. Mandible

23.Arden index is used for interpretation of?


ElectroretinogramVisual field charting

D. Mandible
24.False about Argyll Robertson Pupil is?
A. Accommodation reflex normal

B. Direct pupillary reflex absent

33.Rupture of Charcot Bouchard aneurysm usually causes?

C. Indirect pupillary reflex normal

A. Extradural haemorrhage B. Subdural haemorrhage C.

D. Visual acuity normal

Subarachnoid haemorrhage D. Intracerebral haemorrhage

Correct answer
: C. Indirect

pupillary reflex normal

Correct answer
: D. Intracerebral haemorrhage
ETIOLOGY OF INTRACRANIAL HAEMORRHAGE

25.Cycle which is absent in humans is?


Menstrual cycle B. Ovarian cycle
Estrous cycle

Extradural haemorrhage Torn meningeal artery

C. Hormonal cycle

D.

Correct answer:D. Estrous cycle

26.Which among the following is the primary action of nitric


oxide in the GIT?
A. Contraction of gastrointestinal smooth muscle
B. Relaxation of gastrointestinal smooth muscle
C. Vasodilation

Subdural haemorrhage Rupture of bridging cortical


veins due to trauma.
Subarachnoid haemorrhage Rupture of berry
aneurysm.
Intracerebral haemorrhage Rupture of Charcot
Bouchard aneurysm,associated with chronic
hypertension.

D. Vasoconstriction
Correct answer
: B. Relaxation of gastrointestinal
smooth muscle
27.Reabsorption of water in the GIT is maximum in?
Duodenum B. Jejunum C. Ileum

D. Colon Correct

answer
: B. Jejunum

34.Anti LKM antibodies are found in?


Inflammatory myopathies B. Behcets syndrome
infection

C. HCV

D. Primary biliary cirrhosis Correct answer


: C.

HCV infection
35..Anti LKM (Liver Kidney Microsomal) antibodiesare of 3

28.Pulmonary compliance is not decreased in?


COPD B. Decreased surfactant production
Pulmonary congestion

C.

D. Pulmonary fibrosis Correct

answer
: A. COPD

types:
Anti LKM 1 Autoimmune hepatitis type II and
Chronic hepatitis C
Anti LKM 2 Drug induced hepatitis
Anti LKM 3 Chronic Hepatitis D

29.Which of the following inhibits thrombin activity?


Heparin cofactor II

Alpha 2 antitrypsin Chymotrypsin

Alpha 2 macroglobulin

Both a & d are correct but mostttt

correct is D.

Antibodies present in the other options: Inflammatory


myopathies Anti Jo1 antibody
Anti alpha enolaseantibody

Behcets syndrome
Primary biliary

cirrhosis Anti mitochondrial antibody against E2


subunit of pyruvate dehydrogenase complex

30.Which of the following are vitamin K dependant clotting


factors?II and IV

(especially middle meningeal artery) due to trauma.

IX and X

III and V

VI and VII

31.The force of muscle contraction can be increased by all


A. Increasing the frequency of activation of motor units
B. Increasing the number of motor units activated
C. Increasing the amplitude of action potentials in the motor
neurons
D. Recruiting larger motor units Correct answer : C.
Increasing the amplitude of action potentials in the motor
neurons
32.The main excitatory neurotransmitter in the CNS is:
A. Glycine B. Acetylcholine C. Aspartate D. Glutamate
GABA is the main

inhibitory neurotransmitter.
Excitatory neurotransmitters Glutamate, Aspartate
Inhibitory neurotransmitters Glycine, GABA (Gamma
Amino Butyric Acid)

A.Bronchodilation B. Vasodilatation C. Pain D.


Increase in vascular permeability Correct answer

: A.Bronchodilation

of the following except :

Correct answer : D. Glutamate

36..Not an action of bradykinin?

37.Triad of skin lesions, asymmetric mononeuritis multiplex


and eosinophilia are seen in?
A. Cryoglobulinemic vasculitis B. Polyarteritis nodos C.
Churg Strauss Syndrome D. Giant cell arteritis
39. Which of the following genetic mutation has been
described in aortic stenosis?
a) KCNH2 b) KCNQ1 c) NOTCH1 d) SCN5As
40.Mitral annular area in systole: a) Decreases by 10% b)
Decreases by 25% c) Increases by 10% d) Increases
by 25%
41.The most common genetic make up in Gastation
trophoblastic neoplasm is ?a)69xxx b.69xxy c.46xy
46.xx
42.the most common cause of mastitis??

a.strep pyogen b.E.coli


d.pseudomonas

C.staph aureus

4.

43.ovaran tumor with psammoma bodies? a. cystic


teratoma b.granulosa theca
c.krukenbergs d.yolk
sac e.serous cystadenocarcinoma
44.all related to CCOP, except??
a.thromboembolism b.coronary disease c.stroke
d.demetia in old women e.endomatrial ca
45. most common complain of pco?? a.hirsutisim
b.infertility c.obesity d.oligomennhorea
46.best serum marker of menopause
C.ESTROGEN D.CORTISOL

49.breast cancer with wrst prognosis?? a. pagets disease


b. tubular carcinoma c.infiltrating ductal ca
d.inflammatory carcinoma e.ductal ca in situ
50. gold standard sign of virilization?? A,acne
muscle mass c.hirsutism d.enlarged clitoris

11.

13.
14.

b.inc

51. common site of fertilization? a.ovaries b.isthmus


c.ampulla d.cervix
52.features of normal change in pregnancy.. except a.inc
GFR b.inc plasma volume c.inc RBC mass
d.inc
thyroxin e.res acidosis
53. examples of pathological gynecomastasia.. except
a.cirrhotic pt b.kelinfelters syn
c.on spironolactone
d.on digoxin for ccf e. 85 yrs old age man with no
appearent disease.
54. risk factr for ovarian ca .. except Mutation of BERCA
SUPPRESSOR
Turners syn
peutz jegarhans synd
histry of breast ca
multiparity

15.
16.

17.

18.

19.

a. abd enlargement is

b.palpable ovaries in post menopausal women is ovarian


cancer until proven otherwise. c.CA125 inv in all ovarian
cancers d.prognosis is better if age of pt is less th 65 yrs
18 april medicine test mcqstried to remember all but
few remain

3.

9.

12.

a. beta hcg b. urinary beta hcg c.ultrasound d.laproscopy

2.

8.

10.

48. most accurate test for ectopic pregnancy??

1.

6.
7.

a.LH B.FSH

47.The most common diagnosed breast cancer is??\


a.phyllods tumor b.fibroadenoma c.intraductal
pappiloadenoma d.lobulr ca in situ e.ductal ca in situ

55. true for ovarian ca.. except?


most commn sign

5.

Cervical rib attached to transverse process of


cevical rib will compress:A.C5B. C8C. T1
. Resting membrane potential acieved by:A.
Diffusion of potassium outside the cell ..B.Na K
pump C.is same for all types of cell
Diagnostic tumor marker for ovarian CA:A. CEA
B. CA 125 C. AFP

20.
21.

22.
23.
24.
25.
26.
27.
28.

Which of the folowing prolong use of drug will


cause persistent inc in gastrin
secretion:a.antacids b.antichollenergic c.b
blockers d.H 2 antagonist e.PPI
Compression effects on esophagus by which
organ enlargement:A. Aorta B. Left atrium C.
Pulmonary artery D. Right atrium E. Left ventricle
Normal FEV/FVC1 Ratio:A. 0.8B. 0.95C. 0.6
Prolong coma is caused by lesion of which
nucleus:A. Nucleus cerelous B. Periaquedactal of
gray of mid brain
Stem about diphtheria vaccine reaction:A:type 1
B:type 2 C:type 3: d type 4
Which of the following increases lower
eosophageal tone n speeds up gastric emptying
time:A. Aluminium hydroxide B. Metoclopramide
Scanty bar bodies: A. Klienfelter B. Downs C.
Turner
A pt presented to his dentist due to inability to put
his dentures, dentist referred back to physician
saying there is no dental probe, where is the
lesion: (STEM NOT PROPER)A. substantia nigra
Cause of Mental retardation:A. Fragile X
Syndrome (Downs wasnt in options)
Rt optic tract damage
30 yr male newly diagnosed wth HTN, on ECG lt
ventricular hypertrophy, serum cholesterol 200,
FBS 88 mg%, urinary sugar ++ RFTs were
normal, K 2.5 , Na 140, what is cause of his HTN:
A. Diabetic nephropathyB. Essential
hypertensionC. PheochromocytomaD. Primary
hyperaldosteronism (asim gynea page 328 )
Soft Keratin with no nucleus found in:A. Cortex of
hair B. Matrix of nail C. Stratum corneum
Monitoring of HEP B virus can be done via: A. S.
Bilirubin B. SGPT (or ALT) C. PTD. Serum
Protein E. Serum Albumin(PT is to monitor Liver
failure)
Patient with Gastrectomy presents with Anemia
after One and half year, what will you give:A. Vit
B12B. Vit CC. IronD. Blood transfusion
Dorsal Column Medial Leminsical carries:A.
ProprioceptionB. PainC. Temperature(Pain and
Temp is by Anterolateral System)
Contralateral loss of pain and temp at T9 and
ipsilateral loss of proprioception below T8,
bilateral normal reflexes:
A. Lesion of spinothalamic tract at T8 B.
Complete spinal cord sectionC. Hemisection at
T12D. Hemisection at L1 or 2 (asim surgery page
21 4th edition)
Downs Syndrome:A. Increase incidence with
increasing mother age B. Extra chromosome
Aphasia (Non-fluent) after stroke or Trauma
which area of Brain is involved:A. Brocas B
wernik
Maximum absorption of salt and water:A.
Jejunum B. Ileum
Parasympathetic effect on SA node is due to 1 K
2 Na 3 Na and K 4 calcium 5 cal and k
Mcqs on Gap junction
Metaplasiaa change in function
Dysplasia ,,sixe and shape
Pallegra ..which vitamin def..nicotinic acid
Aids pt wd treatment side effect pancreatitis
neuropsthydidonosine

29. Poorly cross bbb..levodopa dopamine


diazepam..phenytoin
30. Wrist injury deep ulnar nerve injury (asim and
shoab gynea)
31. Primary hyperparathyroidismasim and shoab
gynea mcq no 79 page 320..very imp mcq
32. Contration of smooth musle anp aldosterone.
adh
33. Parasyphetic effect ..dec intestinal
motility..pupillary muscle contraction..
34. Tachyphylaxix means
35. Insulin s stimulated by glucogan
36. Pubertyestrogen..hypothalamic sensitivity to
estriol
37. Chorea..
38. Tremor of teacher tx..proprnolol
39. S4 due to ventrilur fiilling ganong mcq
40. B cell cd19 and cd 20 asim and shoab
41. Activated t cell in mononucleases asim and
shoab
42. B cell damage in mononucleases asim and
shoab
43. Young pt wd Atypical cell wd
lymphadenopathy..invest monospot
44. chrons and UC defrntitng lesioins Perianal
lesion,.,,, asim
45. if 100 cm ileum is resectd wht will happenInc
water loss / inc bile salts loss.. asim
46. urine osmlarity 600mosm
urea 15
Na 110
K 3.4
options:
SIADH
dehydration
addisons
47. muscles for foot eversion..peroneus longus
48. young female wd recurrent abortion and dvt that
is nt corrected even aftr giving. FFPs
mutated factr 5 laden
49. musle spindle or golgi tendon asim surgery
mcqs..
50. myxoma.left atrium.asim
51. bells palsy lesion at cerebelopontinelesion
just befre stapidius
52. testicular ca markers
53. gas cangrene is due to RAbia plus asim
54. membranous coilitis is cuased by
55. dilated pupils occulomotr nerve damage
56. trochealor nerve damage..
57. drug causes acidosis ulcers salycylate
58. treated wd naloxone not
responding..phenobarbitol
59. .in pregnent womn whch is incrsd
TIBc
60. pO2 will be equal to 100mmhg in
left to rt shunt
rt to left shnt
fibrosis
high altitude
61. whch spreads via orofcal trnsmsn
hep b
c
CMV
hep e
62. qstn abt secretin pancreatic hco3

63. pain ovry reaching medial side of thigh ..whch


nrve is involvedObturator nerve..pudendal
nerve
64. qstn abt SMA brnchs Middle colic artry
65. progestrne is precusr to whch hrmnes asim
gynea
66. sjogrens is assocated wd RA (ans)
67. neutrophils 98% lymphocytes 2%
cloudy plerual effusion of an old man wd h/o
cough, no fvr
options were
chronic inflmtn serpous effusion
abscess,,,, purulent effusion./..
fibrinoid inflmtn
68. following splnectmy immediate finding will be >
thrombocytosis Howel jolly bodies
69. left adrenal vein drain
70. dif b/w anaphylctc shock nd hypovlmc shck
71. cavrns sinus contains -> ICA and abducnt nerve
72. vit acitvated in intstines >vit k
73. CRF ->///low co2 oral intake is limited to
0.5 l/day
74. eclmptc ldy
drug given to her before GA
hydralazne
sodm nitropruside
75. puberty starts wd increase in
gondotrophins
gonadal hormones
76. malaria scenario
77. bby wd urine frm umblcs
ans.presistent urcharus
78. qstn abt aziothioprine mech of action.Inhibits
both immmune plus serological response..
79. chemotactix c5a
80. smthng abt amyloid light chain
81. axillary fascia is derived from deep
cervicalprevertebrl
82. thyroid epithelium..changes its shape wd function
83. macroprolsctinemia asim gynea page 325
84. inclusions smthing ..parkinsons/basal ganglia
invlvment
85. qstn abt carcinoid tumors
86. all anti psychotics are likely in
anti ematic effect
anti pyschtic
potency in mili grams
price
87. Pregnant women 1st trimester..develops atrial
fib..what would u do...
Iv hep(ANS)Iv hep plus oral anticoagAspirin
88. 55yr old Pt allergic to penicilin..prev h/o
rhematoid heart disease..get dental treatment..
Prophylaxis
Vanco plus
gentaVancoClindaAmpicilinAmoxil
89. Sup to inf in intercostal space
Nerve artery vein
90. Inguinal region below inguinal ligamnmedial to
lat..navel
91. Ligamentam arteriosum is between
Arch and left pul artery
92. Pt wd high bpdec afferent arteriole
resiistnce..bowmane cappilaryy high pressure

93. Gonococcus..dx gram staining


94. Rt Horinzal semicircular canal..when moves head
to left
95. Yeloow color pleural effusion..neutrophils 90
percentbasophils 2 percnt serous
effusepurulent effusion
96. Folic acid source..raw vegetable liver
97. Lp site..upper boerder od l4
98. Pheochromocytoma vma
99. Brain area 321 postcentral gyrusagranular
cortex..primary somatosensory
100. Cerebellumafferent from motor cortex
101. Supralateral Floor of 4th ventricle frmded by
medulla ponsinferior cerbllm peduncles
102. Cortisol..which has diffrnt function in adispose
tissue and liver in protein syntheses
103. Uncoupling of oxidation reaction..thyroisds
hormone insulin.. epinephrine
104. Atps 38 total
105. Product btwn glycolysis and tca..pyruvate and
acetyl coa
106. Dead space volume is not changedstanding
tracheastomydeep breathingdo cheetara
note on dead space
107. Venous return in exercise..contraction of leg
muscles
108. Primary billiary cirrhosis..antimitochondrial
antibody
109. Mean
110. Aldosterone mainly act on ..proximal distal loop
collcting
111. Sulphur containg amino acid cystine
112. Cs f..glucose lower than normal
113. Hla.grft rejection..not necessary in liver
transplant if abo compate
114. Hlaother bcq ..asim
115. High level of estrogen..endometrial hyperplasia
116. Pt wd recurenr throat infection
..diarheaaselective igA defici
117. Pancreatitis fat necrosis
118. Ankylosing spondy b27
119. Thymusno lumph nodules..rabia
120. Psudohyphea and budding..candidiasis
121. Second week..hypo epiblast..rabia
122. Metaplasia asim
123. Dysplasiawd big nuclei of cervical cell asim
124. Alpha feto protein testicular ca
125. Bipolar disorder..lithium tx
126. Mitral reg..pansystolic murmur
127. Beta blockersAv block
128. Dilated cardiomyopathy..coxsackie virus
129. Dvt..femoral vein
130. Wegener granul scenario..focal necrotizing
vasculitis
131. Inc gastric motility..gastrin
132. Vitamin k def scenario
133. Duchene dystr..invest cpk
134. Ornithine transcarbamoylase def
scenario..forgotten options of invest
135. Inc triglyceridechylomicron remnnt
136. A scenario wd increase jvp..lung translucency
etcpulmonary aneurysm..vasculitis..forgotten
the stem but tukkaa lagaya
137. Red blood cell in csf..subarachoind hemmrhage
138. Mcq about upper motor neuron lesion
139. Multiple sclerosisreactive gliosis demyelination

140.
141.
142.
143.
144.
145.
146.
147.
148.
149.
150.
151.
152.
153.

154.
155.
156.
157.
158.
159.
160.
161.
162.
163.
164.
165.
166.
167.
168.
169.
170.
171.
172.
173.
174.
175.
176.
177.
178.

179.
180.
181.
182.
183.

184.

185.
186.

Rennin release..jg cell


Insulin sec stimulated by glugogon
Succinyl..psudocholinsterase
Plasma fibrinogrn..invlved in blood clotting
Tbt cell macrophages type 4 reaction
Slescenario
Pernicious anemia..After GASTRECTOMY
Staph aureus inf after thorn prick
Apoptosiscancer
Anti oncogens p53
Vibrio cholera.
MyeloperoxidaSE.NUETROPHILS
Rtaist vasoconstriction..
Dead space
unchanged..treachostomy..standing..deep
inspiration
Female wd silicon breast implant having
inflamtory ..plasma cell nuetrophils esinophils
Deudenal ulcer..gastroduedenal artery
Sa node,,upper part of crista terminalis
Respiratory segmnta treeone atery for each
segmnt
Diabetic nephropathyurine albumin
Fastest conduction..purkinjee
Dorsal rami..extensor of trunk
Rapid adaptationpascinian
Infra orbital artery maxillaery branch
Memory areaoperculum of insulatemporal
lobe
Olfactory cellbipolar cells
Virus causes cancer by changing protein
synthesis
Feco oral route..hep E
Dvt..immobalization
Adh ..supraoptic nuc
ttypical case scnro of asprgilosis
local edema..allergy
creatinine is formed in muscles liver;;; kidney
amputation and then tumor of
nervernueroma??
Fetal life po2 is 80 % in which vesselumbilical
vein
Nasal scraping for leprocy
Atrial fib..digo,,,,atria fib ..pulse deficit
Female cant open mouth,,lat pterygoid dam
Dobutamine action ..bet 1 agonist
About skeletal muscleaponuerosis is fibrous
and membranousbelly is fleshy all along
lengthorigins are proximal and
mobileinsertions are distal and immobile..
Branch of whch artery is end
arterycoronarycentral retinal artery
Remnnt of allontiospersi urachus
Taste center???
Ionizing radical cuases cancer ,,DNA
Dif btw primary abd secondary wound healing
A. Granulation tissue
B. Wound contraction
Primary hepatocellular carcunoma caused by
A. Aflatotoxin
B. Heptatis c
C. Alpha 1 anti trypsin def
Medical ethics pillars..autonomy beneficience
male efficeience justice
Thyroid Medullary ca.marker..calcitonin

187. Pubic bone joint type in last


trimestersymphysis syndesmosis
188. Pulmonary atery ends in ..alveoli ..bronchioles
bronchi

17 april gynea by remsha sheikh


1. Cardiogenic shock due to ?
A. POST MI b. burn c.hypothermia d. hemorrhage e. gram
negative sepsis
2 . neurogenic shock due to ?
A. DEC VASOMOTOR TONE

3. hyoid bone in errect posture located at what level ?


A. AT C3
4. Corticosteroid dec what ?
A. Lymphocytes
5. left shift ?
A. hypothermia
6. left shift ?
A. co poisoning
7. co-relation is said perfect when ?
A. +1
8. Calcitonin secreting tumor are present in ?
A. THYROID b. parathyroid
9. Muscle length measured by ?
A. Muscle spindle
10. total lenght of respiratory cycle ?
A. 2 sec insp and 2 sec exp b. 3 sec insp and 2 sec exp c.
1 sec exp and 3 sec exp
11. lenght of right bronchus ?
A. 2.5cmb. 1.5cm
12. universal recipient ?
A. AB+ve b. ab -ve
13. anticholinergic drug used in the treatment of ?
A. ACUTE BRONCHOSPASM b. allergic asthma
14. mechanism of action of carbidopa and levidopa ?
A. Dec peripheral conversion
15. blood supply of uterus ?
A. ovarian and uterine artery
16. IgA percentage in genital secretion ?
A. 70% b. 55% c. 95% d. 85%
17. Amenorrhea in lactating women by ?
A.PROLECTIN b. Gnrh antagonist c. progesteron (if given
gnrh agonist or gnrh itself then that would be ans ) i dont
remember exactly !!
18. Transverse incision suprapubic structure injured?
A. INF EPIGESTRIC ARTERY
19. melanocytes originates from ?
A. NEURAL CREST CELLS b. epidermis c. dermis
20. severe diarrhea senerio ?
a. Low sodiam b. high potassium c. metabollic alkalosis
rathr than metabollic acidosis
21. structure formed at the end of 5th week ?
A. HEART b. brain c. limb bud d. stomach rotates
22. 80% taking hormne replacement while 20% not ?
A. PIE CHART
23. cervical dysplasia caused by ?
A.iucd b. multiparity c. hsv2
24. 2% bupivicain in 4ml ?
A. 80% b. 0.08% c. 20% d. 0.02%
25. one group of individual with vomitting and one without ?
type of data ?
A. NOMINAL DATA
26. increase GFR ?
A. afferent arteriolar dilatation
27. renin inhibited by ?
A. angiotensin 2

189. Diagnostic feature of folic acid defa


megaloblstic erythropoisis..b..low serum folate ..c
low rbc folate level
190. Pul artery dilation is due to ..a atherosclerosis
b..anuerysm
191. Necrosis type in pancreatitis
28. person taking carbohyderate deficient diet ? which
vitamin should be supplimented?
A. THIAMINE b. riboflavin
29. Mg stores abonduntly present in ?
A. BONES b. soft tissues c. icf d. ecf
30.pus contains ?
A. DEAD NEUTROPHILS b. dead bacteria
31. carcinoma contain which filament ?
A. KERATIN b. desmin
32. most common organism transmitted through STD ?
A. CHLAMYDIA b. gonococci c. Trichomonas d. e-coli
33. fisher man with bleeding gums and echimosys ?
deficiency of ??
A. VIT C b. vit b12 c. folate
34. cyclic GMP which hormone included ?
A. ANP
35. Female with history of c-section and blood transfusion
10 years back now presented with edema and ascites.
choice of investigation ?
A. LFT b. serum to urine osmolarity ratio
36. largest total cross sectional area in ?
A. CAPILLARIES
37. factor 9 deficiency should be treated by ?
A. FFP b. cryoprecipitate
38. lead sheet used in pregnant lady for ?
A. X RAY b. mri c. u/s
39. injestion of drug ? NaHCO3 given as antidote ?
A. PHENOBARBITONE
40. menopause ? patient with lab result ?
A. Ovarian hyperstimulation to INC FSH b. inc lh c. dec
estrogen d. dec lh
41. tamoxifen ?
A. BREAST b. brain c. uterus
42. dead space decreased by ?
A. TRACHEOSTOMY b. shallow breathing c. standing d.
deep inspiration
43. proto oncogene becomes oncogene by ?
a. point mutation b. gene over expression c. dna
transcription d. transduction by retroviruses
44. thyroxine or thyroid hormone increases what ?
A. FREE FATTY ACIDS b. triglycerides c. cholestrol
45. Which hormone regulate prolectin ?
A. DOPAMINE
46. senerio regarding how many bar bodies in XXX ?
A. 2 b. 1 c. 0 d. 3
47. pectoral and axillary lymph node drainage ?
A. upper outer quadtrant
48. there was a Q on mean, median , mode ??
49. true about kidney ?
A. LEFT RENAL VEIN ANT TO AORTA AND LEFT RENAL
ARTERY
b. rra smaller than left
50. glucose transort from mother to fetus ?
A. FACILATED DIFFUSION b. simple diffusion
51. Dopamine infusion steady state ?
A. 2min b. 5 min c. 9 min
52. common pathway between gluconeogenesis and
glycolysis ?
A. FRUCTOSE 6 PHOSPHATE to FRUCTOSE 1,6 bi
phosphate
53. diabetic patient developed sensory abnormality of left
leg and ulcer of big toe?

A. angiopathy and neuropathy


54. carotid boies ??
a. osmorecepror b. baroreceptor c. lie at the junction of
external carotid artry d. responds to dec PH e. response to
dec Ppo2
55. caroted bodies response in stimulation to ?
A. INCPCO2 b. inc po2 c. inc ph d. pco2 in csf
56. at 26 week of gestation with history of hemolytic
disease in previous child ? which investigation should be
done ?
A. INDIRECT COOMBS test b. maternal antibody titre c.
fetal blood for Rh sampling
57. nerve fibre to uterus travel through ?
A. uterosacral ligament b. broad ligament c. round ligament
58. patient with multiple fractures of pelvis having loss of
sensation in perianal area. which nerve is spared ?
A. ILLIOHYPOGASTRIC NERVE b. pudendal nerve c.
ilioinguinal nerve d. podt cutaneous nerve of thigh
59. Q on characterstics of TB?
a. caseous necrosis b. epithiloid cells c.
60. q on diagnostic criteria for TB ?
a. sputem AFB b. caseoes necrosis
61. patient treated for TB pericarditis now dies suddenly .
Autopsy shows blood in pericardiam ?
a. cardiac temponade b. constrictive pericarditis
62. senerio on single crease ?
A. down syndrome ( triasomy 21 )
63. 14 week pregnant lady had fracture and died due to ?
A. FAT EMBOLISM
64. pregnant lady with labur pains died on the way to
hospital due to ?
A. AMNIOTIC FLUID EMBOLISM b. air embolism c. fat
embolism
65. Q was also related to amniotic fluid embolism
66. fetal hydrocephalus head at station -2 from where csf is
drained ?
A. POST FRONTANALL b. ant frontanalle
67. mammary gland originate from?
A. mammary bud or milk line
68. pincipal action of insulin ?
A. move potassium inside cell
69. Q on physiological functions of growth hromone ?
a. closure of epiphyseal cartilage b. ??
70. Q on pulse pressure increases by?
A. stroke volume
71. eating alot...... mentally retarted senerio ??
A. plaider villi syndromb . angle man syndrom c. down
syndrom
72. limb short ..... large trunk senerio ??
A. AUTOSOMAL DOMINANT b. autosomal recessive c. xlinked recessive
73. fetal head at station 0 ? land mark called as ?
A. ISHIAL SPINE
74. parasympethatic stimulation ?
A. intestinal sphinter relax
75. sympethatic stimulation?
A. pupil affected
76. micturation reflex by ?
A. mechanorecepter in bladder wall b. inhibition of reflex by
centre located in pons
77. christmas disease ?
A. factor 9 deficiency
78. cardiogenic shock 1st drug ?
A. dopamine b . adrenaline
79. hypophysectomy and ovulation induction?
A. hmg and hcg
80. uterine prolapse ?
A. metaplasia and chronic cervicites
81. GB pain to shoulder ?
A. c3,c4,c5

82. pudendal nerve root ?


A. s2,s3,s4
83. Q on thirst stimulated by ?
84. urogenital sinus devided in to upper and lower part by ?
A. PARAMESONEPHRIC DUCT b. mesonephric duct
85. embryionic conter part of mullarian tubercle ?
A. SEMINAL COLLICULUS
86. Hiv associated with ?
A. invasive Ca of cervix
87. pt in sibbi balochistan ..camping and sleeps on floor ..
fever with lymphadenopathy ..history of tick bite ?
a. kalazar b. filariasis c. malaria d. lyshminiasis
88. stress hormone ?
A. CORTISOL
89. Acth increased and pt presents with hypertension and
hirsutism ?
A. cushing disease B. cushing syndrome c. adrenocortical
insufficiency
90. pregnant female ... no veggie diet ??
A. ANENCEPHALY
91. adult female breast atrophy ?
A. ESTROGEN and PROGESTERON both b. estrogen c.
progestron
92. Estrogen ?
A. dec osteaclastic activity
93. thyroid ithmus lies at ?
A. 2,3,4 tracheal ring
94. coticospinal decussation ??
A. PYRAMIDS
95. pre central gyrus and left cerebral cortex damage will
cause ??
a. right upper and left lower paralysis
b. left upper and right lower paralysis
c. right upper and right lower paralysis
d. pelvis and lower limb paralysis
e. left upper and lower paralysis
96. inguinal ligament ?
A. ant sup iliac spine to pubic tubercle
97. muscles of the back supplied by ?
A. DORSAL SPINAL RAMI
98. anterior spinal nerve give cutaneus branches by ?
A. T1
99. inscised wound ? factor aid healing ?
A. SUTURES
100. 15 year boy with megaloblastic anemia which
investigations should be done ?
a. methyl melonic acid
101 . arterial supply to bone by ?
a. metaphisial and nutrient artery b. epiphyseal and nutrient
artery
102. iucd in place ?
a. cervical metaplasia
103. renal cause of edeme ?
a. nephrotic syndrom b. hypertension
104. pt treated with myocardial infarction now developed
edemea ?causes?
options??
105. 12 year ols with multiple episodes of staph abcess n
klebsella attack ? deficiency of?
A. NADPH oxidase
106. radiation exposure injury mechanism ?
A. free radicles
107. no increase of growth hormone in pregnancy due to ?
A. placental somatotropin b. thyroxin
108. oxygen Pp when leaving placenta ?
A. 22 b. 30. c. 40 d. 50
109. beta blocker ?
A. hypoglycemia
110. alpha blocker ?
A. DOXAZOCIN b. clonidine

111. gastric Ca ?
A. h pylori
112. adenoma ?
A. glandular cells
113. brain ischemia ?
A. liquifictive necrosis
114. milk ejection ?
A. oxytocin
115. menopause ?
A. dec response from ovary
116. difference of rbc in veins and artries ?
A.inc HCO3 in veins b. inc cholride ions in veins c. small
size rbc in vein d. more viscocity in artries e. more oxygen
in veins
117. hyline cartilage ?
a. menisci b. pinna c. epiglottis d. something related to
articular surface
118. anal triangle lymph drainage ?
a. medial group of superficial inguinal lymph nodes
119. labia majora drainage ?
a. sup inguinal lymph nodes
120. middle rectal artery is a branch of ?
A. INTERNAL ILLIAC ARTERY
121.regarding bupivican ?
a. block Na channel in ionised state
b. more absorption in non ionised state
c.
122. confidence interval can be find out by ?
A. MEAN AND STANDARD ERROR OF MEAN
123. sternocostal and anterior surface of heart formed by ?
A. RIGHT VENTRICLE
124. post 1/3 of interventricle groove supplied by ?
A. RIGHT CORONARY ARTERY
125. tertiary bronchus/segmental bronchus supplies part of
lung ?
A. bronchopulmonary
126. scarlet fever toxin ?
A. exotoxin A b. enterotoxin c.
127. part of brain receive excitatory input from brain but
doesnot send back axon ?
a. ventroanterior thalmus nucleus b. vlt c. s.nigra d. globus
palidus e. caudate
128. spleen squestration of rbcs and old rbcs phagocytosis
in which part ?
a. red pulp sinosoids and marginal sinosides
129. during surgery from where can surgeon reach to
pouch of douglas by one nick ?
a. post fornix
130. vagina ?
a. post rectovaginal fascia
131. epiploic foramen ?
a. posteriorly aorta B. SUPERIORLY CAUDATE LOBE c.
gastrosplenic ligament on the right side
132. bladder control ?
A. s2, s3 , s4 b. L1 and L2
133. after prostectamy rectourethral fistual can be
prevented by ?
a. bucks fascia B. DENOVILLERS FASCIA c. scarpa fasia
d. walder fascia
134. golgi tendon organ involved in ?
a. muscle tension
135. female in labour , station 2 on pv , ishial spines
inverted , arch less triangular , sacral promontry un
reachable , which type of pelvis is this ?
a . anthropoid b. android
136. in chronic intavascular hemolysis ?
A. DEC HEPTAGLOBIN b. hemoglobinemia c.
hemoglobinuria d. reticulocytosis
137. most common cause of anemia in pregnancy ?
A. HEMODILUTION b. iron deficiency c. folate deficiency

138. most imp factor in 2nd stage of labour ?


A. MATERNAL ABDOMINAL MUSCLE CONTRACTION b.
spinal reflexes c. oxytocin d. prostaglandin
139. how rubella infection can be prevented ?
A. ANTI RUBELLA VACCINE IN PUBERTY b. anti rubella
vaccine in pregnancy c. avoid contact d. education .
vaccination to babies
140. stuctures not passed through greater sciatic foramen ?
A. TENDON OF OBTURATOR INTERNUS b. piriformis c.
pudendal nerve
141. in anemia blood becomes turbulent due to ?
A. DEC VISCOSITY
142. primordial germ cells derived from ?
A. ENDODERM
143. phrenic nerve lies on anterior surface of ?
A. scalene anterior
144. female with bi cornute uterus anomly and with that
what other anomalies can be present ?
A. UROGENITAL ANOMILIES b. renal abnormality c.
mental retardation d. limb deformity e. cardiac abnormality
145. antigens resplonsible fro graft rejection belong to ?
A. HLA
146. sub acute bacterial endocarditis caused by ?
A. VIRIDANS
147. gamma globulins formed in ?
A. plasma cells
148. essential fatty acid ?
A. linolenic acid
149. longest incubation period of which malarial parasite ?
A. plasmodiam Malaria b. ovale c. vivax d. falciperem
150. liver is connected to anterior abdomial wall by ?
A. FACIFORM LIGAMENT b. attachment of hepatic veins
to ivc
152. whip like cilia ?
A. fallopian tubes b. sperms c. organ of corti
153. most abundant immunoglobin in body is ?
A. Ig G
154. Q about superior relation of pitutary gland ??
a. body of sphenoid sinus b. body of ethmoid sinus c. body
of maxillary sinus
155. female urethra not true ?
A. 2inch from clitoris
156. functional layer of endometrium ?
A. compactum and spongiosm
157. bladder supply ?
A. pelvic splancnic and inferior hypogastric plexes
158. pharma mcq action of anesthetic require ?
A. lipid solubility
159. damage to brocas and wernickes will cause ?
A. global aphasia
160. pre malignant condition of skin ?
A. compound nevus b . simple navus c. seboric keratitis d.
psoriasis
161. cavernous sinus contains which cranial nerve ?
A. 6thb. 3rdc.4thd. 5th
162. two different genetic lines derived from a single zygote
?
a. genomic imprinting b. zygote acromegaly c. mosaicsm d.
zygote anomly
163. reagarding pelvic inlet increased transverse diametre
and decreased anteroposterior diameter ?typ eof pelvis ?
a. gynecoid b. plateplloid c. android d. anthropoid
164. conter part of labia majora in males ?
A. SCROTUM b. testes c. epididymud
165. blood supply to uterus pass by which ligament ?
A. braod ligament b. cardinal ligament c. round ligament
166. loss of sensation over perianal area ?
A. pudendal nerve b. illioinguinal nerve
167 . a child with dry cough , low grade fever , and pain in
joints . which investigation will you do ?A.BLOOD

CULTURE b.sputem culture c. both d. afb


168. regarding kidney ?A. renal fascia covers
kidney/perirenal
fat and suprarenal gland

169 . regarding blood supply of bones ?


a. scapula inf angle .... artery
b. nutrient foramen by..... artery
c . bones of hand by ..... artery
fat and suprarenal gland
FCPS Part-1 Exam Sunday 19th April, 2015.

19. Self Replicative Organelles, Peroxisomes, contain


Oxidase developed from? A. Golgi Apparatus B. Nucleus C.
Mitochondria D. S ER (Answer) E. R ER
20. Pulmonary Embolism, caused by? Ans. Deep Venous
Thrombosis of Leg.
21. Helmintic disease by caused by Mosquito bite? Ans.
Filariasis .
22. Scenario of Patau Syndrome. Microcephaly,
Polydactyly, cleft Lip and Palate. Ans. Trisomy 13. .
23. female short Vagina, no Uterus, O/E testes present,
cause? Ans. Testicular Feminizing Syndrome. .

1. Prolonged APTT, scenario. Ans. Christmas disease.

24. Fragile X Syndrome? A. Trinucleotide repeat (Answer)


B. Defect in X chromosome. .

2. Byssinosis? Ans. Textile Industries.

25. Scenario, female with Oligomenorrhea, karyotype


XO/XX, cause? Ans. Gonadal Agenisis and Hypogonadism.
.

3. Mother Blood Group A, One Child O and one child AB,


Father have? A. A B. O C. AB D. B E. Two childs have two
fathers.
4. Collagen? A. Abundant in Ligament Flavum B. Elastic C.
freely rejoin D. Yellow Colour E. Lightly Stain with Eosin
5. Diaphragmatic opening, Aortic Aneurysm at opening,
compress which structure? Ans. Thoracic Duct and
Azygous Vein. .
6. Whole Wheat contain Vitamin? A. Thiamine (Answer) B.
Riboflavin .
7. Highest Cholesterol content? A. White meat B. Red meat
(Answer, not sure) C. Milk D. Egg (some says Red Meat
and some says Egg) .
8. Tip of scapula, vertebral level? A. T10 B. T9 C. T8
(Answer)
D. T6 .
9. Breast Atrophy caused by? Ans. Lack of Estrogen and
Progesterone. .
10. Metaplasia? A. Increase number and size of cell B.
Functional change of Epithelium (Answer) .
11. Scenario, Councilmans bodies are present? Ans.
Apoptosis. .
12. Scenario, Ascites+Pleural Effusion, Alcoholic Hepatitis,
finding? Ans. Mallory Bodies .

26. Hallmark of HIV? A. Immunodeficiency. B. Proliferation


of virus inside T lymphocyte. .
27. Female have Butterfly Rash, first test performed? Ans.
Antinuclear Antibody. . 28. Esophageal Stage of
Swallowing effected by? Ans. Scleroderma. .
29. Tumor, not melagnant? Ans. Adenoma. .
30. Tumor with 3 germ Layers? Ans, Teratoma. .
31. Tumor invading Nerve Sheath? Ans.
Hemangiopericytoma. .
32. Squamous Cell Carcinoma associated with? Ans.
Marjolin Ulcer. .
33. Scenario, pateint have half/one side headache, biopsy
of Artery show granuloma formation? Ans. Giant Cell
Arteritis.
.
34. Drug which stable plaque in Acute Coronary
Syndrome? A. Atorvastatin (Answer, not Sure) B. Diuretics
C. Morphine D. Nitrates .
35. First Biological marker of Myocardial Infarction? Ans.
Troponin-T. . 36. Myocardial Infarction due to block of LCA,
which site of heart damaged? Ans, Left Atrium and Left
Ventricle. .

13. Scenario of Alcoholic Patient. Ans. Fatty Liver. .

37. Lumbago Patient have chest pain and Tenderness at


Posterior 2-4 Intercostal Space, CK and Trop-T normal, B.P
normal, no retrosternal burning, cause? A. MI B. Aneurysm
of Aorta C. Costochondritis (Answer) D. Pleurisy .

14. Opsonization. Ans. C3b. .

38. Aortic Aneurysm, cause? Ans. Medial Necrosis. .

15. In Acute Inflammation Fluid present is? A. Specific


Gravity of Fluid is 1010 B. is known as Transudate C.
Protein more than 3gram (Answer) .

39. p53, Tumor Suppressor Gene, if mutated, what occur?


Ans. Cell survival. .

16. Pleural Effusion present, Specific Gravity is low than


1002, cause? Ans. CCF. .
17. Chronic cervical adenopathy, tuberculous suspected,
diagnosis by? Ans. Caseous Necrosis. .
18. Young patient on corticosteroids therapy, Abscess on
upper outer quadrant of Rt. Arm excised, after one month
wound not healed, cause? A. decreased neutrophil
infiltration (Answer) B. decrease formation of collagen C.
Re-epithelialization defective .

40. Pneumonia not respond to Antibiotics, Boat shaped


cysts are seen? Ans. Pneumocystis carinii. .
41. Characteristic/diagnosis of Asthma? Ans. FEV1/FVC
<75% .
42. Chron's disease different from Ulcerative Colitis? Ans.
Perianal Lesions. .
43. Diabetic nephropathy diagnosed by? Ans. Albuminuria.
.

44. Goodpasture syndrome? Ans. sub-endothelial deposits


of IgG. .
45. Urothelial Carcinoma associated with? A. Bladder
exstrophy B. Horse shoe shaped kidney
C. Duplication of bladder D. Polycystic Kidney .
46. Scenario of Berry Aneurysm/hemorrhage, association?
Ans. Adult Polycystic Disease. .
47. Testes in Abdomen/ Cryptorchidism associated with?
Ans. Malignancy. .
48. ACTH Incresed, hirsutism, central Obesity, cause? Ans.
Cushing Syndrome. .
49. (another Question also related with
ACTH/Adrenocortical condition, i don't remeber exactly) .
50. Pateint suffered Head Trauma, Urine Osmolarity
decresed, symptoms Polydipsia and Polyuria, cause? Ans.
Central Diabetes Insipidus. .

67. Adenosine? A. Half life 30 seconds B. Transient


Tachycardia C. action with thephylline D. Inhibit SA and AV
node .
68. Patient, Accident, multiple Fracture, B.P 90/60, Pulse
120 b/min? Ans. Fluid Replacement. .
69. Patient transfused with 2 weeks old Bood, Blood
contain mainly?
A. RBC (Answer) B. Platelets C. Monocytes D. Neutrophills
.
70. Chemical Synapse, most likely? Ans. Ca uptake and
release of ACh into cleft. .
71. Isotonic and Isometric contraction, Isotonic contraction
is different by? A. Consume more phosphate bonds B. Less
work done C. Have more energy of recovery D. Heat
production .
72. Major Stress harmone? A. ACTH (Answer) B.
norepinephrine C. Epinephrine D. ADH .

51. Patient came to clinic with complaints of Polyuria and


poldipsia, urine Osmolarity decrased, Incresed Thirst and
Thirst not relieved by drinking water, Injection of ADH given
but symptoms persists? Ans. Nephrogenic Diabetes
Insipidus. .

73. Major Blood Buffer? A. Bicarbonate (Answer) B.


Hemoglobin C. Protein D. Phosphate .

52. Patient with tachycardia and Palpitations, TSH low,


after giving Thyrotropin TSH is Increased, cause is? A.
Problem in Pituatry B. Problem in Thyroid (Answer) C.
Thyroid Adenoma .

75. Autonomic Nervous System? Ans. Parasympathetic


Increase GIT Motility. .

52. Multiple Fractures occur in which disease? A.


Osteoporosis B. Vitamin D deficiency C.
Hyperparathyroidism D. Hypoparathyroidism E. Cushing
Synrome .
53. Meningitis, CSF shows, Gluscose 40 (less)
Lymphocytes. Ans. Tuberculous meningitis. .
54. Chorea and Athetosis, disease of? Ans. Basal Ganglia.
.
55. One gram of Protein gives? Ans. 4 calories.
56. Counselling is? Ans. Help patients themselves. .
57. A Girl of 8 years, at terminal illness, parents told to
doctor that dont anything about disease to our Child, and
now Patient is asking "am i going to die?" Doctor response
should be? Ans. first docor should ask "what Your parents
have told you?" .

74. Vasodilation cause by which Adrenergic Receptors?


Ans. Beta 2. .

76. Regarding Epithelium and Glands? Ans. Simple


Squamous occurs at sites of Gaseous Exchange. .
77. 2nd heart Sound? Ans. Closure of Semilunar valves. .
78. Humans start burning sensation of skin, when surface
exposed to Temparature is? A. 35 B. 40 C. 45 (Answer, not
sure) .
79. Tricuspid valve Auscultation.
Ans. Lower end of Sternum on Rt. Side. .
80. Mean Systemic Filling Pressure? Ans. Affects Mean
Arterial Pressure. .
81. Difference of Pulomanary and Systemic Circulation?
Ans. Pulmonary have low Resistance. .
82. Organ have no change of Blood Supply during
Exercise? Ans. Brain. .
83. In CCF, edema caused by? Ans. Increase Hydrostatic
Pessure. .

58. Individuals have the independent and Same vhance of


being selected? A. Random Sample B. Systemic Sample
(Answer) .

84. Extreme left shift of Curve? A. HbF B. CO poisoning


(Answer) .

59. CCK causes? Ans. Inhibition of Gastric Emptying. .

85. Low Affinity of Oxygen? Ans. Methemoglobin. .

60. Gastric Motility increased by? Ans. Gastrin. .

86. Increased Ph, causes? Ans. Hypo ventilation. .

61. Steatorrhea, deficincy of Pencreatic enzyme? Ans.


Lipase. . 62. Bile Salts absorbed in? Ans. Ileum. .

87. Rapid Ascent to high Altitude? Ans. Pulmonary Edema.


.

63. Anterior PItuatry Histology Features. A. Corticotropes


constitute more than 50 percent of Ant: Pitutary. B.
Chromophobes are of smallest cell type. ,

88. Increase in Renal Blood Flow and GFR? Ans. Dilatation


of afferent Arteriole. .
89. Digoxin Toxicity by? Ans. Loop Diuretics. .

64. ADH and Oxytocin produced in? Ans. Hypothalamus. .


90. Aldosterone causes? Ans. Hypkalemia. .
65. Insulin inhibits? Ans. Glycogenolysis. .
91. Osteogenesis Imperfecta? Ans. Collagen Disorder. .

92. Testosterone function? A. Thickening of laraynx B.


Darkening of areola C. Deposits of Fat on Medial side of
thigh .
93. Virus Transfer information of one cell to another? Ans.
Transformation. .
94. Respiratory Symptoms? Ans. Ascariasis. .
95. Amoebic Liver Abscess, diagnosed by? A. Liver biopsy
B. Ultrasound C. CT scan D. Serology E. Stool Antigen
Testing .
96. Typhoid at day 6, diagnosed by? Ans. Blood Culture. .
97. Typhoid in week 2, diagnosed by? Ans. Blood
culture+Widal Test. .
98. Thorn prick form Abscess, bacteria? Ans.
Staphylococcus Aureus. .
99. Charecteristic Finding in Clostridium botulinum? Ans.
Flaccid paralysis. .

117. Rt. Gastric Artery is branch of? Ans. Hepatic Artery. .


118. Subclavian Vessels groove over? Ans. First Rib. .
119. Carotid Sheath? Ans. Thickening of Investing layers of
fascia of neck. .
120. Parotid Duct open? Ans. Upper 2nd Molar between
gum and Cheek and lips.
.
121. DIabetic Old Lady, Scenario of Weakness of Vision?
Ans. Cataract. .
122. Cavernous Sinus Thrombosis, Eye swelling, block to
Vein? A. Infraorbital. B. superior ophthalmic (Answer) c.
Inferior Ophthalmic .
123. Difficulty in moving foot medially/Inversion, cause is?
Ans. Loss of Function of Tibialis Anterior and Tibialis
Posterior. .
124. Appendicitis, Tenderness in moving thigh medially and
medial rotation, appendix position? Ans. Pelvic. .
125. Artery in True Pelvis? Ans. Middle Rectal Artery. .

100. Finding of Falciparum Malaria? Ans. Black water fever.


.
101. Scenario, pap smear +ve, after 2 years? Ans. Ca
Cervix. .
102. Drug for MRSA? A. Co-trimaxazole (Answer) B. CoAmoxiclav .
103. Drug contraindicated in Chronic Renal Failure? Ans.
Gentamycin. .
104. Anti-Tuberculous Drug cause Hyperuricemia? Ans.
Pyrazinamide. .

126. Muscles of back supplied by? Ans. Dorsal Ramus. .


127. Scapula attached to Thoracic wall by muscle? A.
Quadratus lumborum B. Lattismus Dorsi C. Rhomboids
Major D. Serratus Anterior (Answer, not sure) .
128. Patient with Hemoptysis, damage to? Ans. Bronchial
Arteries. . 129. Tip of Nose, herpes Infection, Nerve? Ans.
Ophthalmic division of Trigeminal. .
130. Upper Motor neuron Lesion. (don't remember answer)
.
131. Lower Motor neuron Lesion. Ans. Wasting. .

105. Chronic Renal Failure, Anmeia, Drug will be given?


Ans. Erythropoietin. .
106. Route by which first pass Metabolism occur? Ans.
Oral. .

132. Nerves involve in Light Reflex. Ans. II and III .


133. Notochord. Ans. Induces overlying Ectoderm to form
Neural plate. .
134. Cerebellar Lesion. Ans. Dysmetria. .

107. Phase I Biotransformation, example? Ans. Oxidation. .


108. Scenario of organophophorus poisoning, Miosis+
Salivation, Antidote is? Ans. Atropine. .
109. Drug which causes release of Histamine? Ans.
Morphine. .
110. Drug causing hypoglycemia when given with
sulphonylureas? Ans. Phenylbutazone. .
111. High calcium in serum, hyperparathyroidism, drug will
b given? A. Bisphosphonates B. Loop diuretics (Answer,
not sure) .
112. ECG, shows 2 P waves to 1 QRS complex? Ans.
Mobitz Type II. .
113. In complete Heart block? Ans. Fainting may Occur
during ventricle unable to pump Blood. .
114. cause Thrombocytopenia in Child after Infection (don't
remember Options) .
115. Regarding Low molecular weight heparin? Ans.
Antagonizes factor Xa and Thrombin (not sure about
answer) .
116. Protrusion of Mandible by Muscle? Ans. Lateral
Pterygoid. .

135. Medial Leminscus formed by? Ans. Internal Arcuate


Fibers decussate. .
136. Nerve accamponies Superficail Temporal artery? Ans.
Aurico-temporal Nerve. .
137. Head tilt to left and Right Eye upward moved, muscle
effected? A. Rt. Superior Oblique (Answer, not sure) B. Lt.
Superior Oblique C. Inferior Rectus .
138. Vertical Cut of Optic Chiasma. Ans. Bitemporal
hemianopia. .
139. Regarding blood supply in the brain? A. Middle
cerebral artery supplies foot area (Answer) B. Arteries don't
anastomose on the surfac of the brain C. Arteries don't
anastomose after entering the brain substance D. Internal
capsule is supplied by anterior cerebral artery .
140. Area of the Brain most activated before performing
skilled movements? Ans. Cortical association area. .
141. Person Sitting Calm and Eyes closed, which pattern
on EEG? Ans. Alpha. .
142. Patient during eating, Increased lacrimation,
misdirection of which nerve? A. Facial B. Vagus C.
Glossopharyngeal .

143. X-Ray, Rt. border of Heart shadow formed by? Ans.


Superior Vena Cava. .
144. Nucleolus? A. Contain Euchromatin during replication
B. Ribosmal RNA formation C. Contains DNA and
nonhistone proteins .
145. Carbohydrates and proteins combined and secreted
from? A. Golgi
B. R ER C. S ER D. Lysosomes E. Mitrochondria .
146. DIC, caused by (related question, i dont remember
exact) A. Thromboplastin B. Factor X C. Factor IIV d.
Factor IX .
147. Peroxisomes originate from? A. Golgi? B. S ER C. R
ER . 148. Thymus? A. Contain Lymphocytes (Answer, not
sure) B. Developed from 4th pouch C. Glandular structure .
149. Most likely feature of thyroid? A. Goiter moves on
deglutation bcoz attach to prevertebral fascia B. Thyroid
drain to deep cervical (Answer, not sure) C. Rt. Recurrent
laryngeal nerve on isthmus D. Supplied by Internal Carotid
artery .

167. Smoking associated? Ans. Bronchogenic Carcinoma.


.
168. Lipoprotein with highest cholesterol content? Ans.
LDL. .
169. Venous Return increased by? Ans. Muscle contraction
of leg. .
170. Organ with high arteriovenous oxygen difference at
rest? Ans. Heart. .
171. Drugs causing Neuroleptic malignant sydrome? Ans.
Haloperidol. .
172. Mediator of pain? Ans. Bradykinin. .
173. Lady with dypnea at rest, cause? Ans.
Pulmonary Embolism. .
174. Regarding Thiazides? Ans. Cause Hypokalemia. .
175. Dyspnea on lying down, cause? Ans. Retrosternal
Goiter. .

150. Radioactive Strontium causes? A. Small cell Ca of


Lung B. Osteosarcoma (Answer, not sure) C. Kaposi
sarcoma .

176. Warfarin? Ans. Not given in pregnancy. .

151. Diagnosis of Leprosy at initial stages by? A. M Leprae


in Nasal Scrapings B. M Leprae in sputum (Answer, not
sure) C. Blood C/S .

178. 1st line cell in inflammation? Ans. Neutrophils. .

152. True Hermaphrodite genotype? A. XY B. XX C. XXY


(Answer, not sure) D. XY/XX .
153. Slow growing Thyroid Tumor?
A. Papillary Carcinoma (Answer) B. Follicular Carcinoma C.
Medullary Carcinoma D. Anaplastic Carcinoma .

177. IgE interacts with? Ans. Eosinophils. .

179. Scenario, Lady with decreased levels of plasma cells


and recurrent infections? Ans. Complement deficiency. .
180. Factor that maintains the lungs not to collapse? Ans.
Negative Intrapleural pressure.
.
181. Thrombocytopenia appropriate? Ans. DIC occurs due
to decrease in platelet count. .

154. Primary Center of Ossification at Epiphysis of Long


bone at Birth? A. Lower end of Femur B. Lower End of
Humerus C. Upper End of Fibula D. Upper End of Tibia .

182. Regarding Amniocentesis? Ans. Performed after 14


weeks. .

155. Question about Immunodeficiency. (don't remember


exactly) .

183. Local Cause of delayed Wound Healing. Ans.


Infection. .

156. Oblique Fissure? Ans. T3 to 6th Chostochondral


junction. .

184. Plasma volume increases alot by giving?? A. Distilled


Water B. Hypertonic Saline (Answer, not sure) C. Hypotonic
Saline .

157. Antiemetics during chemotherapy. Ans. Ondansetron.


.

185. Vagus Nerve Nucleus? Ans. Nucleus Ambigus (not


sure, not remember exact question) .

158. Which is a function of Parasympathetic Nervous


System? Ans. Increase in salivary gland secretion. .
186. Question of Biostatistics, Answer was Variability. .
159. Gamma Efferent fibers supply? A. End of intrafusal
muscle fibers (Answer) B. Central portion of muscle fibers .
160. Shortened PR Interval? Ans. WPW Syndrome. .
161. A child with recurrent infections decreased levels of
all Immunoglobulins? Ans. X linked Agamaglobulinemia. .
162. Pulmonary Artery supplies? Ans. Alveoli. .
163. Estrogen is produced by? Ans. Ovary. .

187. Discharge from muscle spindles decreases when? A.


end of the intrafusal muscle fiber contracts B. muscle
contracts C. when the muscle spindle is stretched .
188. Cause of Hypernatremia? A. Heart failure (Answer,
not sure) B. excessive ADH .
189. Injury to spinal cord does not effect Respiration? Ans.
below C5. .

164. Most common bone fractured? Ans. Tibia. .


165. Scenario, B.P 220/110, defect in? Ans. J.G Apparatus.
.
166. Patient having raised PO decreased calcium? Ans.
Acute Renal Failure. .

190. End product of Purine Metabolism in Urine? Ans. Uric


Acid

Vous aimerez peut-être aussi